Re: [obm-l] Duvidas II

2004-11-14 Por tôpico Augusto Cesar de Oliveira Morgado



max(2x+5, 8-3x)<35  se e somente se  2x+5<35 e 8-3x<35, ou seja, x<15 e x>-9. 


== 
Mensagem  enviada  pelo  CIP  WebMAIL  - Nova Geração - v. 2.1 
CentroIn Internet Provider          http://www.centroin.com.br 
Tel: (21) 2542-4849, (21) 2295-3331        Fax: (21) 2295-2978 
Empresa 100% Brasileira - Desde 1992 prestando servicos online 


-- Original Message ---
From: "aryqueirozq" <[EMAIL PROTECTED]> 
To: "obm-l" <[EMAIL PROTECTED]> 
Sent: Sat, 13 Nov 2004 19:12:03 -0200 
Subject: [obm-l] Duvidas II 

> Se max(a,b) denota o maior dentre os números reais a e b, quantas soluções inteiras admite a desigualdade max(2x+5, 8-3x)<35 ?
 
>  
 
> a) 21   b) 22 c) 23 d) 24  e) 25 
>   
>   
> Agradeço desde de já.
 
--- End of Original Message ---






Re: [obm-l] um problema do livro 'A matematica do ensino medio'

2004-10-30 Por tôpico Augusto Cesar de Oliveira Morgado
n circunferências distintas; caso contrário, não são n circunferências.
Não quero me absolver, mas esse tipo de observação é uma coisa bem paulista. 
Nos vestibulares de São Paulo, a quantidade de pegadinhas a respeito é 
absurda. Várias já apareceram pela lista, em questões de múltipla escolha que 
deixavam os candidatos sem saber o que marcar.
==
Mensagem  enviada  pelo  CIP  WebMAIL  - Nova Geração - v. 2.1
CentroIn Internet Provider  http://www.centroin.com.br
Tel: (21) 2542-4849, (21) 2295-3331Fax: (21) 2295-2978
Empresa 100% Brasileira - Desde 1992 prestando servicos online


-- Original Message ---
From: Fabio Niski <[EMAIL PROTECTED]>
To: [EMAIL PROTECTED]
Sent: Sun, 31 Oct 2004 05:21:24 -0200
Subject: [obm-l] um problema do livro 'A matematica do ensino medio'

> Na pag. 154, o problema 11 é
> "No máximo quantos pontos de interseção existem quando sao 
> desenhadas n circunferencias" É mais provavel que eu seja um mal 
> leitor do que o autor do livro um mal escritor. Entao por favor me 
> expliquem o que o problema quer. Uma interpretacao boba porem 
> correta é tomar n circunferencias iguais sobrepostas nesse caso 
> teriamos infinitos pontos de interseçao.
> =
> Instruções para entrar na lista, sair da lista e usar a lista em
> http://www.mat.puc-rio.br/~nicolau/olimp/obm-l.html
> =
--- End of Original Message ---

=
Instruções para entrar na lista, sair da lista e usar a lista em
http://www.mat.puc-rio.br/~nicolau/olimp/obm-l.html
=


Re: [obm-l] IME

2004-10-30 Por tôpico Augusto Cesar de Oliveira Morgado
Desde que a IUPAC decidiu que está.

==
Mensagem  enviada  pelo  CIP  WebMAIL  - Nova Geração - v. 2.1
CentroIn Internet Provider  http://www.centroin.com.br
Tel: (21) 2542-4849, (21) 2295-3331Fax: (21) 2295-2978
Empresa 100% Brasileira - Desde 1992 prestando servicos online


-- Original Message ---
From: "Osvaldo Mello Sponquiado" <[EMAIL PROTECTED]>
To: "obm-l" <[EMAIL PROTECTED]>
Sent: Sat, 30 Oct 2004 10:52:52 -0300
Subject: Re: [obm-l] IME

> Olá ?
> 
> Desde quando equivalente-grama está obsoleto ?
> Apesar da unidade de concentração molaridade ser a mais utilizada na 
> prática do laboratório, esta torna-se incoveniente pois muitas 
> reações não ocorrem equimolarmente, mas sim equi-equivalente-grama:  
>O motivo é que, no ponto final de qualquer titulação, o número de 
> equivalentes da substância titulada tem que ser igual ao número de 
> equivalentes da substância titulante (usualmente um padrão primário)
> . Como resultado, os cálculos podem ser executados sem que se 
> tenha sempre que levar em conta a razão entre as molaridades dos reagentes.
> Foi tambem utilizada por Faraday em seus testes empíricos.
> 
> Até.
> 
> > Sei q jah eh fugir d+ da matematica, mas ainda no IME desse ano, acho
> > simplesmente ridiculo se pedir um conceito tão antigo e já obsoleto como o
> > Equivalente Grama numa prova de Quimica. E ainda pior é pedir pra 
demonstrar
> > N Avogadro por uma maneira tão decorada de livros...decepcionante...
> > Tomara q a banca seja mais criativa e moderna no proximo ano
> > 
> > - Original Message -
> > From: "Felipe Torres" <[EMAIL PROTECTED]>
> > To: <[EMAIL PROTECTED]>
> > Sent: Wednesday, October 27, 2004 10:20 AM
> > Subject: Re: [obm-l] IME questão do logaritmo
> > 
> > 
> > > oi
> > > Só p botar mais um desenvolvimento q mostra q o
> > > enunciado tava claramente errado:
> > >
> > > c^2 = (ac)^loga(d)
> > >
> > > c^2/c^loga(d)= d
> > >
> > > c^(2-loga(d))=c^(logc(d))
> > > igualando os expoentes
> > >
> > > 2 - loga(d) = logc(d)
> > >
> > > log a(d)+ log c(d) = 2
> > >
> > > aí  deveria ter um logb(d) do lado do dois ali
> > >
> > > logo b deveria ser igual a d para questão estar
> > > correta
> > >
> > > {}s
> > > Felipe
> > >
> > >
> > > __
> > > Do You Yahoo!?
> > > Tired of spam?  Yahoo! Mail has the best spam protection around
> > > http://mail.yahoo.com
> > > 
=
> > > Instruções para entrar na lista, sair da lista e usar a lista em
> > > http://www.mat.puc-rio.br/~nicolau/olimp/obm-l.html
> > > 
=
> > >
> > >
> > 
> > 
> > =
> > Instruções para entrar na lista, sair da lista e usar a lista em
> > http://www.mat.puc-rio.br/~nicolau/olimp/obm-l.html
> > =
> >
> 
> Atenciosamente,
> 
> Osvaldo Mello Sponquiado 
> Engenharia Elétrica, 2ºano 
> UNESP - Ilha Solteira
> 
> __
> Acabe com aquelas janelinhas que pulam na sua tela.
> AntiPop-up UOL - É grátis!
> http://antipopup.uol.com.br/
> 
> =
> Instruções para entrar na lista, sair da lista e usar a lista em
> http://www.mat.puc-rio.br/~nicolau/olimp/obm-l.html
> =
--- End of Original Message ---

=
Instruções para entrar na lista, sair da lista e usar a lista em
http://www.mat.puc-rio.br/~nicolau/olimp/obm-l.html
=


Re: [obm-l] espaços vetoriais

2004-10-09 Por tôpico Augusto Cesar de Oliveira Morgado
(1, 1, -2) e (2, 1, -5) pertencem a W mas a soma (3, 2, -7) não.

==
Mensagem  enviada  pelo  CIP  WebMAIL  - Nova Geração - v. 2.1
CentroIn Internet Provider  http://www.centroin.com.br
Tel: (21) 2542-4849, (21) 2295-3331Fax: (21) 2295-2978
Empresa 100% Brasileira - Desde 1992 prestando servicos online


-- Original Message ---
From: "andrey.bg" <[EMAIL PROTECTED]>
To: "obm-l" <[EMAIL PROTECTED]>
Sent: Sat,  9 Oct 2004 15:54:14 -0300
Subject: [obm-l] espaços vetoriais

> Mostre que W={(x,y,z)/x^2+y+z=0}"não" é um subespaço 
> vetorial do R^3.
> 
> se alguém poder me ajudar eu agradeço muito.
> 
> __
> Acabe com aquelas janelinhas que pulam na sua tela.
> AntiPop-up UOL - É grátis!
> http://antipopup.uol.com.br/
> 
> =
> Instruções para entrar na lista, sair da lista e usar a lista em
> http://www.mat.puc-rio.br/~nicolau/olimp/obm-l.html
> =
--- End of Original Message ---

=
Instruções para entrar na lista, sair da lista e usar a lista em
http://www.mat.puc-rio.br/~nicolau/olimp/obm-l.html
=


Re: [obm-l] sutileza, o retorno

2004-10-02 Por tôpico Augusto Cesar de Oliveira Morgado
O google fornece 116 000 referências para nested intervals.

==
Mensagem  enviada  pelo  CIP  WebMAIL  - Nova Geração - v. 2.1
CentroIn Internet Provider  http://www.centroin.com.br
Tel: (21) 2542-4849, (21) 2295-3331Fax: (21) 2295-2978
Empresa 100% Brasileira - Desde 1992 prestando servicos online


-- Original Message ---
From: "Osvaldo Mello Sponquiado" <[EMAIL PROTECTED]>
To: "obm-l" <[EMAIL PROTECTED]>
Sent: Sat,  2 Oct 2004 01:56:16 -0300
Subject: Re: [obm-l] sutileza, o retorno

> Assim como a afirmacao "todo dragao (daqueles
> > que poem fogo pelas ventas) eh um profundo 
> conhecedor da integral de
> > Lebesgue" tambem eh. Nao ha pedras que Deus nao 
> possa caregar. Logo, por
> > vacuidade, tais pedras satisfazem ateh mesmo aa 
> propreidade de nao poderem
> > ser carregada por Deus. Nao ha contadicao.
> > Artur
> 
> Citando o critério de Lesbegue para integração, andei 
> procurando sobre o princípio dos intervalos 
> encaixantes, o que eu adoro. Alguem ai tem um link que 
> fale mais sobre esse princípio ? Não consegui achar 
> nada de específico no google.com, nem no astalavista...
> 
> Atenciosamente,
> 
> Osvaldo Mello Sponquiado 
> 2º ano em Engenharia Elétrica 
> UNESP - Ilha Solteira
> 
> __
> Acabe com aquelas janelinhas que pulam na sua tela.
> AntiPop-up UOL - É grátis!
> http://antipopup.uol.com.br/
> 
> =
> Instruções para entrar na lista, sair da lista e usar a lista em
> http://www.mat.puc-rio.br/~nicolau/olimp/obm-l.html
> =
--- End of Original Message ---

=
Instruções para entrar na lista, sair da lista e usar a lista em
http://www.mat.puc-rio.br/~nicolau/olimp/obm-l.html
=


Re: [obm-l] Desinventando Ruffini

2004-09-27 Por tôpico Augusto Cesar de Oliveira Morgado



Cliquei em enviar antes do que devia.
Quero declarar que esse algoritmo é completamente inútil, ocupa no cérebro espaço que podia estar sendo ocupado com coisas mais úteis. Só conheço essa bobagem porque no meu tempo de aluno caía no vestibular. 
O perigo de divulgar essas coisas é que daqui a pouco um desses educadores aprende e olha o desastre aí, gente!, e começa a haver palestras, nesses encontros de educação, do tipo "resgatando conhecimentos esquecidos: o algoritmo de Jacques Peletier". 
Tudo que Peletarius faz Ruffini faz melhor!


== 
Mensagem  enviada  pelo  CIP  WebMAIL  - Nova Geração - v. 2.1 
CentroIn Internet Provider          http://www.centroin.com.br 
Tel: (21) 2542-4849, (21) 2295-3331        Fax: (21) 2295-2978 
Empresa 100% Brasileira - Desde 1992 prestando servicos online 


-- Original Message ---
From: "Augusto Cesar de Oliveira Morgado" <[EMAIL PROTECTED]> 
To: [EMAIL PROTECTED] 
Sent: Mon, 27 Sep 2004 20:19:03 -0200 
Subject: [obm-l] Desinventando Ruffini 

> O algoritmo de Peletarius para verificar se um numero a eh raiz de um polinomio P(x) eh o seguinte: 
> Exemplo 1: Verificar se 2 eh raiz de 2x^3 +x^2-5x+4 = 0 (aliás, não é!) 
> 2    //    1 //  -5    //   4 
> 
> 7/8 //   -1/4  //  -3/2  //  2 
> 
> Na primeira linha, os coeficientes do polinômio. 
> Os elementos da segunda linha foram determinados do seguinte modo: na última coluna, dividimos o coeficiente pelo número que estamos testando se é raiz; os demais são obtidos somando em diagonal e dividindo pelo número que está sendo testado. Sse o primeiro elemento da segunda linha é 0, é raiz. 
> 
> Exemplo 2: Verificar se 2 eh raiz de x^3 +x^2-5x-2 = 0 (aliás, é!) 
> 1    //    1 //  -5    //   -2 
> 0   //   -1 //  -3   //    -1 
> 
> == 
> Mensagem  enviada  pelo  CIP  WebMAIL  - Nova Geração - v. 2.1 
> CentroIn Internet Provider          http://www.centroin.com.br 
> Tel: (21) 2542-4849, (21) 2295-3331        Fax: (21) 2295-2978 
> Empresa 100% Brasileira - Desde 1992 prestando servicos online 
> 
> -- Original Message --- 
> From: samanta <[EMAIL PROTECTED]> 
> To: [EMAIL PROTECTED] 
> Sent: Sun, 26 Sep 2004 20:16:07 -0300 (ART) 
> Subject: Re: [obm-l] equaçõesalgébricas 
> 
> > Olá prof. Morgado (e demais amigos do fórum), 
> > o senhor poderia transcrever o algorítimo de peletarius? antes de postar a mensagem anterior, procurei em sebos os livros que você citou (inclusive a RPM indicava alguns), mas não encontrei nenhum. Mesmo ele sendo superfluo, estou com muita curiosidade de conhecê-lo. 
> >   
> > Grata, 
> > Samanta 
> > 
> > Augusto Cesar de Oliveira Morgado <[EMAIL PROTECTED]> wrote: Livros da década de 60, para o terceiro ano científico, como 4 autores (Peixoto, Lisboa, Dacorso e Roxo), Ari Quintela, Tales de Melo Carvalho. 
> > Mas o Peletarius é uma bobagem, é um Ruffini piorado (serve para testar se um numero a é raiz de um polinomio P(x), mas nao fornece o quociente da divisao por 
> > x-a).  
> > 
> > == 
> > Mensagem  enviada  pelo  CIP  WebMAIL  - Nova Geração - v. 2.1 
> > CentroIn Internet Provider          http://www.centroin.com.br 
> > Tel: (21) 2542-4849, (21) 2295-3331        Fax: (21) 2295-2978 
> > Empresa 100% Brasileira - Desde 1992 prestando servicos online 
> > 
> > -- Original Message --- 
> > From: samanta <[EMAIL PROTECTED]> 
> > To: [EMAIL PROTECTED] 
> > Sent: Sun, 26 Sep 2004 04:55:25 -0300 (ART) 
> > Subject: [obm-l] equações algébricas 
> > 
> > > Gostaria de saber o teorema "Regras de exclusão de Newton", e o "algoritmo de Peletarius". E se existe algum livro 2º grau com esses assuntos. 
> > > P.S: encontrei esses assuntos na RPM 14 (pg. 39,40,41), mas são tratados de forma um pouco superficial. 
> > > []´s Samanta 
> > >   
> > > 
Yahoo! Messenger 6.0 - jogos, emoticons sonoros e muita diversão. Instale agora! 
> > --- End of Original Message --- 
> > 
> > 
Yahoo! Messenger 6.0 - jogos, emoticons sonoros e muita diversão. Instale agora! 
> --- End of Original Message --- 
> 
--- End of Original Message ---






[obm-l] Desinventando Ruffini

2004-09-27 Por tôpico Augusto Cesar de Oliveira Morgado



O algoritmo de Peletarius para verificar se um numero a eh raiz de um polinomio P(x) eh o seguinte:
Exemplo 1: Verificar se 2 eh raiz de 2x^3 +x^2-5x+4 = 0 (aliás, não é!)
2    //    1 //  -5    //   4

7/8 //   -1/4  //  -3/2  //  2

Na primeira linha, os coeficientes do polinômio. 
Os elementos da segunda linha foram determinados do seguinte modo: na última coluna, dividimos o coeficiente pelo número que estamos testando se é raiz; os demais são obtidos somando em diagonal e dividindo pelo número que está sendo testado. Sse o primeiro elemento da segunda linha é 0, é raiz.

Exemplo 2: Verificar se 2 eh raiz de x^3 +x^2-5x-2 = 0 (aliás, é!)
1    //    1 //  -5    //   -2
0   //   -1 //  -3   //    -1


== 
Mensagem  enviada  pelo  CIP  WebMAIL  - Nova Geração - v. 2.1 
CentroIn Internet Provider          http://www.centroin.com.br 
Tel: (21) 2542-4849, (21) 2295-3331        Fax: (21) 2295-2978 
Empresa 100% Brasileira - Desde 1992 prestando servicos online 


-- Original Message ---
From: samanta <[EMAIL PROTECTED]> 
To: [EMAIL PROTECTED] 
Sent: Sun, 26 Sep 2004 20:16:07 -0300 (ART) 
Subject: Re: [obm-l] equaçõesalgébricas 

> Olá prof. Morgado (e demais amigos do fórum), 
> o senhor poderia transcrever o algorítimo de peletarius? antes de postar a mensagem anterior, procurei em sebos os livros que você citou (inclusive a RPM indicava alguns), mas não encontrei nenhum. Mesmo ele sendo superfluo, estou com muita curiosidade de conhecê-lo. 
>   
> Grata, 
> Samanta
> 
> Augusto Cesar de Oliveira Morgado <[EMAIL PROTECTED]> wrote: Livros da década de 60, para o terceiro ano científico, como 4 autores (Peixoto, Lisboa, Dacorso e Roxo), Ari Quintela, Tales de Melo Carvalho. 
> Mas o Peletarius é uma bobagem, é um Ruffini piorado (serve para testar se um numero a é raiz de um polinomio P(x), mas nao fornece o quociente da divisao por 
> x-a).  
> 
> == 
> Mensagem  enviada  pelo  CIP  WebMAIL  - Nova Geração - v. 2.1 
> CentroIn Internet Provider          http://www.centroin.com.br 
> Tel: (21) 2542-4849, (21) 2295-3331        Fax: (21) 2295-2978 
> Empresa 100% Brasileira - Desde 1992 prestando servicos online 
> 
> -- Original Message --- 
> From: samanta <[EMAIL PROTECTED]> 
> To: [EMAIL PROTECTED] 
> Sent: Sun, 26 Sep 2004 04:55:25 -0300 (ART) 
> Subject: [obm-l] equações algébricas 
> 
> > Gostaria de saber o teorema "Regras de exclusão de Newton", e o "algoritmo de Peletarius". E se existe algum livro 2º grau com esses assuntos. 
> > P.S: encontrei esses assuntos na RPM 14 (pg. 39,40,41), mas são tratados de forma um pouco superficial. 
> > []´s Samanta 
> >   
> > 
Yahoo! Messenger 6.0 - jogos, emoticons sonoros e muita diversão. Instale agora! 
> --- End of Original Message --- 
> 
> 
Yahoo! Messenger 6.0 - jogos, emoticons sonoros e muita diversão. Instale agora! 
--- End of Original Message ---






Re: [obm-l] equaçõesalgébricas

2004-09-26 Por tôpico Augusto Cesar de Oliveira Morgado



Livros da década de 60, para o terceiro ano científico, como 4 autores (Peixoto, Lisboa, Dacorso e Roxo), Ari Quintela, Tales de Melo Carvalho.
Mas o Peletarius é uma bobagem, é um Ruffini piorado (serve para testar se um numero a é raiz de um polinomio P(x), mas nao fornece o quociente da divisao por 
x-a). 

== 
Mensagem  enviada  pelo  CIP  WebMAIL  - Nova Geração - v. 2.1 
CentroIn Internet Provider          http://www.centroin.com.br 
Tel: (21) 2542-4849, (21) 2295-3331        Fax: (21) 2295-2978 
Empresa 100% Brasileira - Desde 1992 prestando servicos online 


-- Original Message ---
From: samanta <[EMAIL PROTECTED]> 
To: [EMAIL PROTECTED] 
Sent: Sun, 26 Sep 2004 04:55:25 -0300 (ART) 
Subject: [obm-l] equações algébricas 

> Gostaria de saber o teorema "Regras de exclusão de Newton", e o "algoritmo de Peletarius". E se existe algum livro 2º grau com esses assuntos. 
> P.S: encontrei esses assuntos na RPM 14 (pg. 39,40,41), mas são tratados de forma um pouco superficial. 
> []´s Samanta 
>  
> 
Yahoo! Messenger 6.0 - jogos, emoticons sonoros e muita diversão. Instale agora! 
--- End of Original Message ---






Re: [obm-l] Sistema linear homogêneo

2004-09-25 Por tôpico Augusto Cesar de Oliveira Morgado
x+y=0
2x+y=0
2x+2y=0
Escalonando,
x+y=0
0x-y=0
0x+0y=0
Apesar do 0x+0y=0 , o sistema é possível e DETERMINADO.

==
Mensagem  enviada  pelo  CIP  WebMAIL  - Nova Geração - v. 2.1
CentroIn Internet Provider  http://www.centroin.com.br
Tel: (21) 2542-4849, (21) 2295-3331Fax: (21) 2295-2978
Empresa 100% Brasileira - Desde 1992 prestando servicos online


-- Original Message ---
From: Guilherme Carlos Moreira e Silva <[EMAIL PROTECTED]>
To: [EMAIL PROTECTED]
Sent: Sat, 25 Sep 2004 12:15:38 -0300 (ART)
Subject: Re: [obm-l] Sistema linear homogênio

> encontrando 0*x1 + 0*x2 + ... + 0*xn = k
>  se k =0, sist indet
>  se k!=0, sist impos
> caso contrario, sist poss
> 
> > Resolvendo um sistema linear homogênio por
> > escalonamento, como eu sei se ele 
> > é determinado ou indeterminado?
> >
> > Uílton
> >
> =
> > Instruções para entrar na lista, sair da lista e
> > usar a lista em
> > http://www.mat.puc-rio.br/~nicolau/olimp/obm-l.html
> >
> =
> >
> 
>   
>   
>   
> ___
> Yahoo! Messenger 6.0 - jogos, emoticons sonoros e muita diversão. 
> Instale agora! http://br.download.yahoo.com/messenger/
> =
> Instruções para entrar na lista, sair da lista e usar a lista em
> http://www.mat.puc-rio.br/~nicolau/olimp/obm-l.html
> =
--- End of Original Message ---

=
Instruções para entrar na lista, sair da lista e usar a lista em
http://www.mat.puc-rio.br/~nicolau/olimp/obm-l.html
=


Re: [obm-l] Ajuda

2004-09-24 Por tôpico Augusto Cesar de Oliveira Morgado
1/x + 1/y = 1/1998
1998y+1998x = xy
(x-1998)(y-1998) = 1998^2
O numero de soluções inteiras da equação acima (que infelizmente não é 
equivalente a original, pois na original x e y devem ser diferentes de zero) 
é igual ao numero de deomposiçoes de 1998^2 em um produto de inteiros, que 
por sua vez é igual ao numero de divisores de 1998^2.
Bem, conte os divisores, exclua as decomposiçoes em que x ou y sao negativos 
ou nulos.

==
Mensagem  enviada  pelo  CIP  WebMAIL  - Nova Geração - v. 2.1
CentroIn Internet Provider  http://www.centroin.com.br
Tel: (21) 2542-4849, (21) 2295-3331Fax: (21) 2295-2978
Empresa 100% Brasileira - Desde 1992 prestando servicos online


-- Original Message ---
From: "Daniel S. Braz" <[EMAIL PROTECTED]>
To: [EMAIL PROTECTED]
Sent: Fri, 24 Sep 2004 10:57:56 -0300
Subject: Re: [obm-l] Ajuda

> Fagner,
> 
> Tente desenhar o grafico de 1/x + 1/y e de 1/1998..as soluções serão
> os pontos onde
> os graficos se cruzam...
> 
> []s
> daniel
> 
> On Thu, 23 Sep 2004 23:29:41 -0300 (ART), fagner almeida
> <[EMAIL PROTECTED]> wrote:
> > olá  galera,  um  colega  me deu uma questão , e  eu
> > 
> > não  consegui resolve , será  que  alguem me dá  uma
> > mão.
> > 
> > Determine o número de soluções de 1/x + 1/y = 1/1998
> > 
> > com x e y inteiros positivos
> > 
> > 
> > ___
> > Yahoo! Messenger 6.0 - jogos, emoticons sonoros e muita diversão. Instale 
agora!
> > http://br.download.yahoo.com/messenger/
> > =
> > Instruções para entrar na lista, sair da lista e usar a lista em
> > http://www.mat.puc-rio.br/~nicolau/olimp/obm-l.html
> > =
> >
> 
> -- 
> "Uma das coisas notáveis acerca do comportamento do Universo é que 
> ele parece fundamentar-se na Matemática num grau totalmente 
> extraordinário. Quanto mais profundamente entramos nas leis da 
> Natureza, mais parece que o mundo físico quase se evapora e ficamos 
> com a Matemática. Quanto mais profundamente entendemos a Natureza, 
> mais somos conduzidos para dentro desse mundo da Matemática e de 
> conceitos matemáticos." (Roger Penrose)
> 
> =
> Instruções para entrar na lista, sair da lista e usar a lista em
> http://www.mat.puc-rio.br/~nicolau/olimp/obm-l.html
> =
--- End of Original Message ---

=
Instruções para entrar na lista, sair da lista e usar a lista em
http://www.mat.puc-rio.br/~nicolau/olimp/obm-l.html
=


Re: [obm-l] AnáliseCombinatória, ops

2004-09-23 Por tôpico Augusto Cesar de Oliveira Morgado



Ops, 3x24x(2x2x2x2)=1152

== 
Mensagem  enviada  pelo  CIP  WebMAIL  - Nova Geração - v. 2.1 
CentroIn Internet Provider          http://www.centroin.com.br 
Tel: (21) 2542-4849, (21) 2295-3331        Fax: (21) 2295-2978 
Empresa 100% Brasileira - Desde 1992 prestando servicos online 


-- Original Message ---
From: "Augusto Cesar de Oliveira Morgado" <[EMAIL PROTECTED]> 
To: [EMAIL PROTECTED] 
Sent: Thu, 23 Sep 2004 20:51:13 -0200 
Subject: Re: [obm-l] Análise Combinatória 

> 
> 
> == 
> Mensagem  enviada  pelo  CIP  WebMAIL  - Nova Geração - v. 2.1 
> CentroIn Internet Provider          http://www.centroin.com.br 
> Tel: (21) 2542-4849, (21) 2295-3331        Fax: (21) 2295-2978 
> Empresa 100% Brasileira - Desde 1992 prestando servicos online 
> > A montanha -russa de um parque de diversões é composta de 3 carros,cada um com 4 bancos de 2 lugares.De quantos modos podem ser acomodados 4 casais em um mesmo carro, de modo que cada casal ocupe o mesmo banco? 
> >   Pensei assim, escolha do carro.3 
> >     dos bancos.4! =24 
> > por casal em cada banco2 
> >  Logo: 3x24x(2x2x2x2)=576,  bateu com o gabarito ! 
> 
> -- Original Message --- 
> From: "Gustavo" <[EMAIL PROTECTED]> 
> To: Olímpiada <[EMAIL PROTECTED]> 
> Sent: Thu, 23 Sep 2004 18:53:05 -0300 
> Subject: [obm-l] Análise Combinatória 
> 
> > A montanha -russa de um parque de diversões é composta de 3 carros,cada um com 4 bancos de 2 lugares.De quantos modos podem ser acomodados 4 casais em um mesmo carro, de modo que cada casal ocupe o mesmo banco? 
> >   Pensei assim, escolha do carro.3 
> >     dos bancos.4! =24 
> > por casal em cada banco2 
> >  Logo: 3x24x2=144, não bateu com o gabarito ! 
> >   
> >   Pois  o gabarito marca 24x24x2=1152. 
> >  Alguém pode me ajudar ??? Desde já agradeço 
> >   
> --- End of Original Message --- 
> 
--- End of Original Message ---






Re: [obm-l] AnáliseCombinatória

2004-09-23 Por tôpico Augusto Cesar de Oliveira Morgado





== 
Mensagem  enviada  pelo  CIP  WebMAIL  - Nova Geração - v. 2.1 
CentroIn Internet Provider          http://www.centroin.com.br 
Tel: (21) 2542-4849, (21) 2295-3331        Fax: (21) 2295-2978 
Empresa 100% Brasileira - Desde 1992 prestando servicos online 
> A montanha -russa de um parque de diversões é composta de 3 carros,cada um com 4 bancos de 2 lugares.De quantos modos podem ser acomodados 4 casais em um mesmo carro, de modo que cada casal ocupe o mesmo banco? 
>   Pensei assim, escolha do carro.3 
>     dos bancos.4! =24 
> por casal em cada banco2 
>  Logo: 3x24x(2x2x2x2)=576,  bateu com o gabarito ! 


-- Original Message ---
From: "Gustavo" <[EMAIL PROTECTED]> 
To: Olímpiada <[EMAIL PROTECTED]> 
Sent: Thu, 23 Sep 2004 18:53:05 -0300 
Subject: [obm-l] Análise Combinatória 

> A montanha -russa de um parque de diversões é composta de 3 carros,cada um com 4 bancos de 2 lugares.De quantos modos podem ser acomodados 4 casais em um mesmo carro, de modo que cada casal ocupe o mesmo banco? 
>   Pensei assim, escolha do carro.3 
>     dos bancos.4! =24 
> por casal em cada banco2 
>  Logo: 3x24x2=144, não bateu com o gabarito ! 
>   
>   Pois  o gabarito marca 24x24x2=1152. 
>  Alguém pode me ajudar ??? Desde já agradeço 
>   
--- End of Original Message ---






Re: [obm-l] Pequeno teorema de Fermat

2004-09-23 Por tôpico Augusto Cesar de Oliveira Morgado
http://www.utm.edu/research/primes/notes/proofs/FermatsLittleTheorem.html
Mas a prova por induçao eh muito simples e foi ate esboçada aqui na lista por 
muitos no caso particular de n^5 - n.
==
Mensagem  enviada  pelo  CIP  WebMAIL  - Nova Geração - v. 2.1
CentroIn Internet Provider  http://www.centroin.com.br
Tel: (21) 2542-4849, (21) 2295-3331Fax: (21) 2295-2978
Empresa 100% Brasileira - Desde 1992 prestando servicos online


-- Original Message ---
From: Artur Costa Steiner <[EMAIL PROTECTED]>
To: [EMAIL PROTECTED]
Sent: Thu, 23 Sep 2004 06:26:32 -0700 (PDT)
Subject: [obm-l] Pequeno teorema de Fermat

> Alguem poderia apresentar ou indicar aonde posso
> encontrar a demonstracao deste teorema?
> Obrigado
> Artur
> 
>   
>   
> __
> Do you Yahoo!?
> New and Improved Yahoo! Mail - 100MB free storage!
> http://promotions.yahoo.com/new_mail 
> =
> Instruções para entrar na lista, sair da lista e usar a lista em
> http://www.mat.puc-rio.br/~nicolau/olimp/obm-l.html
> =
--- End of Original Message ---

=
Instruções para entrar na lista, sair da lista e usar a lista em
http://www.mat.puc-rio.br/~nicolau/olimp/obm-l.html
=


Re: [obm-l] Matriz por triangularização

2004-09-05 Por tôpico Augusto Cesar de Oliveira Morgado
Some à primeira coluna a soma das outras duas, obtendo 
t+3//-1//1
t+3//t-3//1
t+4//-6//t+4
Faça agora cada linha menos a primeira, obtendo
t+3//-1//1
0//t-2//0
1//-5//t+3
O determinante é igual a (t-2)[(t+3)^2-1]= (t-2)(t-2)(t+4)

==
Mensagem  enviada  pelo  CIP  WebMAIL  - Nova Geração - v. 2.1
CentroIn Internet Provider  http://www.centroin.com.br
Tel: (21) 2542-4849, (21) 2295-3331Fax: (21) 2295-2978
Empresa 100% Brasileira - Desde 1992 prestando servicos online


-- Original Message ---
From: "Domingos Jr." <[EMAIL PROTECTED]>
To: [EMAIL PROTECTED]
Sent: Sun, 05 Sep 2004 13:23:30 -0300
Subject: Re: [obm-l] Matriz por triangularização

> Trocar uma linha/coluna da matriz por uma combinação linear das 
> linhas/colunas da matriz não afeta o determinante, então por exemplo,
>  você pode trocar a primeira coluna pela soma desta com a segunda 
> coluna e assim introduzir um zero em (3, 1). Repita o processo de 
> forma a introduzir quantos zeros forem possíveis, isso vai te 
> facilitar a vida.
> 
> >Olá pessoal boa noite.
> >
> >Recebi uma questão  e depois de muito tentar, sem conseguir resolvê-la, 
decidi pedir ajuda na lista. Tenho que resolver a matriz 3x3, que se seugue 
por triangularização, calculando o seu determinante. Eis a matriz:
> >
> >t+3   -1 1
> >
> >5  t-31
> >
> >6  -6t+4 
> >
> >Pede-se ainda determinar t para que a matriz dada seja inversível.
> >
> >Bem se alguém puder me dar uma mãozinha, agradeço bastante,
> >
> >Um abraço, Marcelo.
> >
> >---
> >iBestMail, agora com POP3/SMTP e 120MB de espaço!
> >Experimente: http://www.ibestmail.com.br
> >=
> >Instruções para entrar na lista, sair da lista e usar a lista em
> >http://www.mat.puc-rio.br/~nicolau/olimp/obm-l.html
> >=
> >
> >  
> >
> 
> =
> Instruções para entrar na lista, sair da lista e usar a lista em
> http://www.mat.puc-rio.br/~nicolau/olimp/obm-l.html
> =
--- End of Original Message ---

=
Instruções para entrar na lista, sair da lista e usar a lista em
http://www.mat.puc-rio.br/~nicolau/olimp/obm-l.html
=


Re: [obm-l] Problema das Oito Rainhas...

2004-08-30 Por tôpico Augusto Cesar de Oliveira Morgado



http://en.wikipedia.org/wiki/Eight_queens_puzzle

== 
Mensagem  enviada  pelo  CIP  WebMAIL  - Nova Geração - v. 2.1 
CentroIn Internet Provider          http://www.centroin.com.br 
Tel: (21) 2542-4849, (21) 2295-3331        Fax: (21) 2295-2978 
Empresa 100% Brasileira - Desde 1992 prestando servicos online 


-- Original Message ---
From: Rodrigo Alberto <[EMAIL PROTECTED]> 
To: [EMAIL PROTECTED] 
Sent: Mon, 30 Aug 2004 14:31:58 -0300 (ART) 
Subject: [obm-l] Problema das Oito Rainhas... 

> Alguem poderia me ajudar com a solução desse problema Estou realmente precisando...  Obrigrado... se for possivel me mandar a solução para esse e-mail ou [EMAIL PROTECTED] ficaria muito grato! 
>   
> Problema - colocar oito rainhas no tabuleiro de xadrez modo que nenhuma delas ataque as outras. As rainhas podem se mover em linhas, em colunas ou em diagonais, qualquer número de casas e sentido.  identificando as colunas de A a H e as linhas de 1 a 8. . Marque no tabuleiro a posição de cada rainha (R1,...,R8) e as casas atacadas por pelo menos uma das rainhas com 'X'. Identifique o fim do processo e número de rainhas posicionadas em cada tentativa. Pense numa heurística para chegar às 8 rainhas a partir de qualquer posição inicial
> 
Yahoo! Acesso Grátis - navegue de graça com conexão de qualidade! 
--- End of Original Message ---






Re: [obm-l] mais uma de combinatóriaentão

2004-08-28 Por tôpico Augusto Cesar de Oliveira Morgado



Confesso-me maravilhado com a competência de alguns Champollions que frequentam a lista (no caso, o Claudio Buffara). Que enunciado tosco! Que significa 2 homens não sentem juntos? São dois homens determinados, tipo Paulo e José? São dois homens quaisquer?
Seria tão mais simples dizer: de modo que não haja homens em posições adjacentes!
Depois não sabem porque os alunos odeiam e acham difíceis problemas de Combinatória.

== 
Mensagem  enviada  pelo  CIP  WebMAIL  - Nova Geração - v. 2.1 
CentroIn Internet Provider          http://www.centroin.com.br 
Tel: (21) 2542-4849, (21) 2295-3331        Fax: (21) 2295-2978 
Empresa 100% Brasileira - Desde 1992 prestando servicos online 


-- Original Message ---
From: Andre Silveira Ramos <[EMAIL PROTECTED]> 
To: [EMAIL PROTECTED] 
Sent: Sat, 28 Aug 2004 15:02:50 -0300 (ART) 
Subject: [obm-l] mais uma de combinatória então 

> Aqui está mais um probleminha de combinatória. 
> Tem a ver com permutação circular também. 
> Não deve ser tão difícil, acho que não estou pensando do jeito certo. 
> Bom, aí está: 
>   
> De quantas maneiras 7 homens e 12 mulheres podem sentar-se ao redor de uma mesa redonda de forma que 2 homens não sentem juntos? 
>   
>   
>   
> Resp: 
>   
> (11!) * (11!) / 10 
>   
> Abraços, 
> André Silveira Ramos
> 
Yahoo! Acesso Grátis - navegue de graça com conexão de qualidade! 
--- End of Original Message ---






Re: [obm-l] Morgado, Guilherme e a todos amigos

2004-08-13 Por tôpico Augusto Cesar de Oliveira Morgado
Os representantes da SBM em Pernambuco são:
1) Cícero Monteiro de Souza – [EMAIL PROTECTED]

Universidade Federal Rural de Pernambuco - Departamento de Física e Matemática

Av. D. Manoel de Medeiros, s/nº

Dois Irmãos

CEP: 52071-900 - Recife – PE

Fone: (81) 441-4577

2) Paulo Roberto Santiago – [EMAIL PROTECTED]

Universidade Federal de Pernambuco - Departamento de Matemática 

Av. Prof. Luiz Freire, s/n - CCEN - Cidade Universitária 

CEP: 50740-540 - Recife - PE. 

Fone: (81) 3271-8410 - Ramal: 238

Pode também ser pedida a SBM, que enviará pelo correio. http://www.sbm.org.br/

A nova edição saiu ha menos de um mes e contém as soluçoes de todos os 
problemas. Foi vendida no recente ENEM de Recife.
Abraços.
Morgado


==
Mensagem  enviada  pelo  CIP  WebMAIL  - Nova Geração - v. 2.1
CentroIn Internet Provider  http://www.centroin.com.br
Tel: (21) 2542-4849, (21) 2295-3331Fax: (21) 2295-2978
Empresa 100% Brasileira - Desde 1992 prestando servicos online


-- Original Message ---
From: "Gustavo" <[EMAIL PROTECTED]>
To: <[EMAIL PROTECTED]>
Sent: Fri, 13 Aug 2004 17:53:18 -0300
Subject: Re: [obm-l] Morgado, Guilherme e a todos amigos

> Oi Morgado sou de Recife, e por aqui todas as vezes que procurei seu
> livro(no dep. de matemática da UFPE) escutava que ele estava 
> esgotado !!Falo sobre :Analise Combinatória e Probabilidade.Será que 
> chegará esta nova edição ??

=
Instruções para entrar na lista, sair da lista e usar a lista em
http://www.mat.puc-rio.br/~nicolau/olimp/obm-l.html
=


Re: [obm-l] combinatória

2004-08-10 Por tôpico Augusto Cesar de Oliveira Morgado



Bonita solução, Dirichlet. Entretanto, na pressa, o fim ficou mal redigido. Tomo a liberdade de mudar o final da sua solução.

== 
Mensagem  enviada  pelo  CIP  WebMAIL  - Nova Geração - v. 2.1 
CentroIn Internet Provider          http://www.centroin.com.br 
Tel: (21) 2542-4849, (21) 2295-3331        Fax: (21) 2295-2978 
Empresa 100% Brasileira - Desde 1992 prestando servicos online 


-- Original Message ---
From: Johann Peter Gustav Lejeune Dirichlet <[EMAIL PROTECTED]> 
To: [EMAIL PROTECTED] 
Sent: Tue, 10 Aug 2004 17:58:27 -0300 (ART) 
Subject: Re: [obm-l] combinatória 

> Em minha opiniao, esta questao nao e de Combinatoria dependendo do que voce quer. 
> Por exemplo, se voce tivesse escrito "De uma resposta usando argumentos unicamente combinatorios", eu escreveria algo assim: 
>   
> nCi e o numero de modos de escolher i elementos do conjunto [n]. 
> i*(nCi) e o total de modos de escolher i elementos do conjunto [n], e rotular um deles (por exemplo, peça para que ele vista uma roupa laranja fluorescente). 
> Logo o somatorio conta de quantas formas podemos escolher um subconjunto de [n] e pedir a um dos elementos do subconjunto escolhido que vista uma roupa laranja fluorescente. Mas esse total e 
> n*(2^(n-1)) (voce pede para que algum numero se habilite a vestir a roupa laranja fluorescente; depois voce seleciona alguns dos demais (n-1) caras para acompanharem-no, ou seja, você seleciona um subconjunto do conjunto formado pelos demais n-1 elementos.). 
>   
> Mas como voce nao disse se queria esse ou outro tipo de resposta, tu podes ate mesmo desconsiderar essa mensagem... 
>   
> 
> nilton rr <[EMAIL PROTECTED]> wrote: 
> 
> Agradeço pela ajuda 
> 
> Obtenha, de forma simplificada, o valor da soma C(n,1) + 2C(n,2) + 3C(n,3) +...+nC(n,n) 
> __
> Do You Yahoo!?
> Tired of spam? Yahoo! Mail has the best spam protection around 
> http://mail.yahoo.com 
> __
> Do You Yahoo!?
> Tired of spam? Yahoo! Mail has the best spam protection around 
> http://mail.yahoo.com 
--- End of Original Message ---






Re: [obm-l] Morgado, Guilherme e a todos amigos

2004-08-09 Por tôpico Augusto Cesar de Oliveira Morgado
Acrescentei as respostas.

==
Mensagem  enviada  pelo  CIP  WebMAIL  - Nova Geração - v. 2.1
CentroIn Internet Provider  http://www.centroin.com.br
Tel: (21) 2542-4849, (21) 2295-3331Fax: (21) 2295-2978
Empresa 100% Brasileira - Desde 1992 prestando servicos online


-- Original Message ---
From: "Marcos Paulo" <[EMAIL PROTECTED]>
To: <[EMAIL PROTECTED]>
Sent: 9 Aug 2004 18:58:57 -0300
Subject: Re: [obm-l] Morgado, Guilherme e a todos amigos


> >Por favor companheiros preciso dessas respostas
> >até quarta feira dia da minha aula, agradeço
> >antecipadamente.
> >1) Uma pastelaria vende pastéis de palmito,
> >carne, queijo e espinafre. de quantas maneiras
> >uma pessoa pode comprar 8 pastéis sendo pelo
> >menos 2 de queijo?
> Seja p o número de pastéis de palmito, y o número de pastéis de 
> carne, z o número de pastéis de queijo e w o número de pastéis de espinafre.
> p + y + z + w = 8. Ainda devemos ter p maior que 2, portanto faremos 
> p = x + 2 e dessa forma mesmo q x seja zero p será pelo menos 2. A 
> pergunta agora será: Quantas soluções naturais tem a equação x + y + 
> z + w = 6?
> 
> Para repartir 6 unidades (u) em 4 grupos são necessários 3 
> separadores (s) Por exemplo: s u u s u u u s u seria o equivalenbte 
> a x = 0, y = 2, z = 3 e w = 1
> 
> Cada permutação dessas 9 letras será uma resposta nova, portanto a 
> minha resposta será igual ao número de permutações da palavra acima 
> (formada por s e u) Resposta: 9!/(3!*6!)
> 
> >2)Uma livraria vai doar 25 livros iguais a 5
> >bibliotecas municipais. Cada biblioteca deve
> >receber pelo menos 3 livros. qual o nr de
> >maneiras distintas que esta livraria poderá
> >repartir os livros dessa doação?
> 
> Repetindo o processo acima vc tem:
> X1+X2+X3+X4+X5 = 25 e faça X1 = 3+A1, ... X5 = 3+A5
> resolva igual ao primeiro exercício.
> A resposta é 1001

> >3) quantas são as soluções inteiras não
> >negativas de x1+x2+x3+x4+x5+x6+x7 =15 nas quais
> >exatamente duas incógnitas são nulas?
> 
> Aqui vc tem q escolher 2 entre 7 para serem nulas C7, 2
> Vão sobrar 5 incógnitas: A1, A2, ...,A5 substitua cada uma delas por 
> B1+1, B2+1, ..B5 + 1 e resolva igual ao primeiro exercicio 
> encontrando um resultado R = 1001 a resposta será R* C7,2 = 21 021
> 
> no livro Do Morgado (Que pra alegria de muita gente está numa edição 
> nova com os exercícios resolvidos) Há uma parte da teoria bastante 
> util em exercicios como esse das combinações completas (ou com 
> repetição). Resumidamente, se vc quer escolher p objetos dentre n 
> disponíveis sendo que nesses p vc pode repetir a sua escolha o 
> número de maneiras de faze-lo é CRn,p. A relação entre o núemro de 
> combinações completas e o número de combinações simples (onde vc 
> deve necesariamente fazer escolhas de objetos distintos) é: CRn,p = 
> C(n+p-1),p
> 
> Espero não ter confundido tudo..
> 
> []'s MP
> 
> 
> 
> =
> Instruções para entrar na lista, sair da lista e usar a lista em
> http://www.mat.puc-rio.br/~nicolau/olimp/obm-l.html
> =
--- End of Original Message ---

=
Instruções para entrar na lista, sair da lista e usar a lista em
http://www.mat.puc-rio.br/~nicolau/olimp/obm-l.html
=


[UTF-8?]Re: [obm-l] Re: [obm-l] Questões estranhas

2004-08-04 Por tôpico Augusto Cesar de Oliveira Morgado
Ponto de vista doido esse. Por ele, não se poderia perguntar quantos são os 
numeros inteiros compreendidos entre 3 e 5. Ou quantas sao as raízes 
distintas de x^2-4x+4=0. Falando claro: pisaram na bola!

==
Mensagem  enviada  pelo  CIP  WebMAIL  - Nova Geração - v. 2.1
CentroIn Internet Provider  http://www.centroin.com.br
Tel: (21) 2542-4849, (21) 2295-3331Fax: (21) 2295-2978
Empresa 100% Brasileira - Desde 1992 prestando servicos online


-- Original Message ---
From: [EMAIL PROTECTED]
To: [EMAIL PROTECTED]
Cc: [EMAIL PROTECTED], [EMAIL PROTECTED]
Sent: Wed, 4 Aug 2004 09:28:26 -0300
[UTF-8?]Subject: Re: [obm-l] Re: [obm-l]  Questões estranhas

> Estava estranhando que ninguem falava dessa questao. Na hora que fizemos
> ficou claro que a redacao fora um pouco infeliz. Como jah vi a banca 
> do colegio manter opcoes piores que essa como corretas acho que 
> ainda vai dar pano para manga. A questao seria de portugues e 
> estaria centrada na palavra DISTINTOS, ou seja, plural. Dessa 
> depreende-se que a quesao continha uma armadilha interpretativa 
> sutil (acreditam os elaboradores da questao). Sendo entao unica 
> possibilidade correta (sob esse ponto de vista) a resposta 2.
> 
> No mais acho que sempre existirao questoes faceis (de pouco valor 
> agregado) medias e inteligentes.
> 
> Coisas do Naval ...
> 
> Paulo André B. Melo
> 
>   "Augusto Cesar
> 
>     de Oliveira  To:  [EMAIL PROTECTED]
> 
>   Morgado" cc:
> 
> <[EMAIL PROTECTED] Subject: Re: [obm-l] Re: [obm-l] Questões 
> estranhas  
>   n.com.br>
> 
> Sent by:
> 
>   [EMAIL PROTECTED]   
>  puc-
> rio.br
> 
>   03/08/2004 20:13
> 
> Please respond
> 
>   to obm-l
> 
> Na questao 18, o numero procurado pode ser 1 ou 2.
> 
> ==
> Mensagem  enviada  pelo  CIP  WebMAIL  - Nova Geração - v. 2.1
> CentroIn Internet Provider  http://www.centroin.com.br
> Tel: (21) 2542-4849, (21) 2295-3331Fax: (21) 2295-2978
> Empresa 100% Brasileira - Desde 1992 prestando servicos online
> 
> -- Original Message ---
> From: Marcos Paulo <[EMAIL PROTECTED]>
> To: [EMAIL PROTECTED]
> Sent: Tue, 03 Aug 2004 16:16:31 -0300
> Subject: Re: [obm-l] Re: [obm-l] Questões estranhas
> 
> > Oi Paulo,
> > eu discordo da estranheza da prova. Achei que a prova foi bastante
> > interessante ressucitando temas interessantes que estavam meio que
> > às traças como o retângulo áureo (questão 7); o eixo radical
> > (questão 17), a fórmula de transformação de radicais duplos em soma
> > de radicais simples
> > (questão 2). O produto notável pedido na questão 1 aparece em quase
> > todo livro de oitava série (mesmo os piores) e eu não conheço outra
> > justificativa (ou uma melhor) para que uma divisão entre inteiros
> > resulte numa dízima periódica a não ser o fato de que exista uma
> > quantidfade finita de restos possíveis na divisão, enquanto o
> > processo (o algoritmo da divisão) pode ser repetido infinitamente.
> > Talvez a opinião dos outros membros da lista fosse interessante
> > nessa questão.
> >
> > []'s MP
> >
> > P.S. Os números das questões que eu citei são referentes à prova azul.
> >
> > At 14:12 3/8/2004, you wrote:
> >
> > >Essa prova do CN está esquisita mesmo. Vocês viram as questões 1 e 16?
> > >
> > >No caso da 16, a resposta certa é a única que faz algum sentido, mas dá
> a
> > >entender que toda seq. com uma quantidade limitada de valores é
> periódica!
> > >
> > >Essas provas do CN já não foram melhores?
> > >
> > >Paulo
> >
> > --
> > Mensagens enviadas estão livres de vírus.
> > Verificado por AVG Anti-Vírus (http://www.avgbrasil.com.br).
[UTF-8?]> > Versão: 7.0.262 / Banco de dados de Vírus: 264.2.0 – Data de
> > Lançamento: 2/8/2004
> >
> > =
> > Instruções para entrar na lista, sair da lista e usar a lista em
> > http://www.mat.puc-rio.br/~nicolau/olimp/obm-l.html
> > =
> --- End of Original Message ---
> 
> =
> Instruções para en

Re: [obm-l] Re: [obm-l] Questões estranhas

2004-08-03 Por tôpico Augusto Cesar de Oliveira Morgado
Na questao 18, o numero procurado pode ser 1 ou 2.

==
Mensagem  enviada  pelo  CIP  WebMAIL  - Nova Geração - v. 2.1
CentroIn Internet Provider  http://www.centroin.com.br
Tel: (21) 2542-4849, (21) 2295-3331Fax: (21) 2295-2978
Empresa 100% Brasileira - Desde 1992 prestando servicos online


-- Original Message ---
From: Marcos Paulo <[EMAIL PROTECTED]>
To: [EMAIL PROTECTED]
Sent: Tue, 03 Aug 2004 16:16:31 -0300
Subject: Re: [obm-l] Re: [obm-l] Questões estranhas

> Oi Paulo,
> eu discordo da estranheza da prova. Achei que a prova foi bastante 
> interessante ressucitando temas interessantes que estavam meio que 
> às traças como o retângulo áureo (questão 7); o eixo radical 
> (questão 17), a fórmula de transformação de radicais duplos em soma 
> de radicais simples 
> (questão 2). O produto notável pedido na questão 1 aparece em quase 
> todo livro de oitava série (mesmo os piores) e eu não conheço outra 
> justificativa (ou uma melhor) para que uma divisão entre inteiros 
> resulte numa dízima periódica a não ser o fato de que exista uma 
> quantidfade finita de restos possíveis na divisão, enquanto o 
> processo (o algoritmo da divisão) pode ser repetido infinitamente. 
> Talvez a opinião dos outros membros da lista fosse interessante 
> nessa questão.
> 
> []'s MP
> 
> P.S. Os números das questões que eu citei são referentes à prova azul.
> 
> At 14:12 3/8/2004, you wrote:
> 
> >Essa prova do CN está esquisita mesmo. Vocês viram as questões 1 e 16?
> >
> >No caso da 16, a resposta certa é a única que faz algum sentido, mas dá a
> >entender que toda seq. com uma quantidade limitada de valores é periódica!
> >
> >Essas provas do CN já não foram melhores?
> >
> >Paulo
> 
> -- 
> Mensagens enviadas estão livres de vírus.
> Verificado por AVG Anti-Vírus (http://www.avgbrasil.com.br).
> Versão: 7.0.262 / Banco de dados de Vírus: 264.2.0 – Data de 
> Lançamento: 2/8/2004
> 
> =
> Instruções para entrar na lista, sair da lista e usar a lista em
> http://www.mat.puc-rio.br/~nicolau/olimp/obm-l.html
> =
--- End of Original Message ---

=
Instruções para entrar na lista, sair da lista e usar a lista em
http://www.mat.puc-rio.br/~nicolau/olimp/obm-l.html
=


Re: [obm-l] Equação exponencial e áreapentágono

2004-07-31 Por tôpico Augusto Cesar de Oliveira Morgado



2) A área de um triângulo de lados a, b e ângulo compreendido C vale 
(1/2)absenC. No caso de um pentágono regular convexo, ele se decompõe em 5 triângulos isósceles de lados R, R, L e ângulo compreendido entre os lados iguais 72 graus (R é o raio da circunferência circunscrita). A área é 
A = 5*(1/2)* (R^2) * sen 72 . 
A relação entre L e R é 2Rcos 54 = L.

== 
Mensagem  enviada  pelo  CIP  WebMAIL  - Nova Geração - v. 2.1 
CentroIn Internet Provider          http://www.centroin.com.br 
Tel: (21) 2542-4849, (21) 2295-3331        Fax: (21) 2295-2978 
Empresa 100% Brasileira - Desde 1992 prestando servicos online 


-- Original Message ---
From: Alan Pellejero <[EMAIL PROTECTED]> 
To: [EMAIL PROTECTED] 
Sent: Sat, 31 Jul 2004 13:45:29 -0300 (ART) 
Subject: [obm-l] Equação exponencial e área pentágono 

> Olá pessoal, 
> tenho duas dúvidas: 
> 1 -) Calcule o valor de x tal que: 
>   
> 2^(X^ - 2) - 5*(2^x) + 2 = 0 
>   
> 2 -) Encontre uma fórmula para calcular a área de um pentágono sendo dado a medida do lado. 
>   
> Grato! 
> Alan 
>   
> Obs: No "(2)", encontrei a expressão A = [l^2*sen(108º)]*[1 - cos(108º~)], mas não tenho certeza se está correta. Por falar em "expressão", é esse mesmo o termo que se deve usar nesse caso?
> 
Yahoo! Mail agora ainda melhor: 100MB, anti-spam e antivírus grátis! 
--- End of Original Message ---






Re: [obm-l] Geom. Espacial

2004-07-31 Por tôpico Augusto Cesar de Oliveira Morgado



Duplamente irônico um grupo que discute Português chamar-se ezatas.

== 
Mensagem  enviada  pelo  CIP  WebMAIL  - Nova Geração - v. 2.1 
CentroIn Internet Provider          http://www.centroin.com.br 
Tel: (21) 2542-4849, (21) 2295-3331        Fax: (21) 2295-2978 
Empresa 100% Brasileira - Desde 1992 prestando servicos online 


-- Original Message ---
From: [EMAIL PROTECTED] 
To: [EMAIL PROTECTED] 
Sent: Sat, 31 Jul 2004 15:39:32 EDT 
Subject: Re: [obm-l] Geom. Espacial 

> Grupo para questoes de Fisica, Quimica, Matematica e Lingua Portuguesa: 
> 
> http://groups.yahoo.com/group/ezatas/ 
> 
> Em uma mensagem de 31/7/2004 11:04:11 Hora padrão leste da Am. Sul, [EMAIL PROTECTED] escreveu: 
> 
> 
> Nao vou nem questionar se a questao nao seria mas apropriada para outros 
> grupos de discussao voltados para assuntos nao olimpicos.  Qualquer que seja 
> o nivel da questao que vc envie seria melhor que vc enviasseapenas 1 email.   
> O seu email original chegou 7 vezes na minha caixa postal 
> e depois que o Felipe Sardinha responde, vc nem se da ao trabalho de ver se 
> responderam e manda denovo? 
> 
> Poluicao eletronica afeta a todos. 
> 
> Para que vc nao manda MAIS UMA VEZ o mesmo email aqui vai a resposta do 
> Felipe: 
> 
> "Caro aryqueirozq, 
> 
> A diferenca entre os volumes inicial e final do cilindro corresponde ao 
> volume de liquido que o paralelepipedo recebeu atraves do escoamento pela 
> tubulacao. Certo? 
> 
> Vi = 2000pi litros = 2pi m³.   (Volume inicial do cilindro) 
> Vi = BxH = 2pi m³. (B - area da base, H - altura) 
> Vi = Bx2 = 2pi m³. (Assim, B = pi m²). 
> 
> Vp = Vi - Vf ( onde Vp - volume do paralelepipedo, Vf - volume final do 
> cilindro). 
> Vp = 2pi - pi.h ( h - altura final dos 2 recipientes). 
> 3/2.pi.h = 2pi - pi.h 
> 5/2.pi.h = 2pi 
> h=0,8 m  ( Acha-se, entao, a altura final ) 
> 
> Vp = 3/2.pi.0,8 
> Vp = 1,2.pi m³ de líquido no paralelepipedo. 
> 
> Convertendo para litros: 
> 
> Vp = 1,2.pi.1000 litros 
> Vp = 1200 pi litros. 
> 
> Bem, de coração, espero ter ajudado. 
> Qualquer coisa, é só mandar. 
> 
> Aquele abraço, 
> Felipe Marinho de O. Sardinha" 
> 
> 
> 
> 
--- End of Original Message ---






[obm-l] Outra nota de falecimento

2004-07-30 Por tôpico Augusto Cesar de Oliveira Morgado
Faleceu ontem, no Rio de Janeiro, o professor Octávio Gitirana. Foi, nas 
décadas de 60 e 70 um dos mais importantes professores de ensino médio de 
Matemática no Rio de Janeiro, tendo lecionado em alguns dos melhores colégios 
e cursos de pre-vestibular do Rio. Certamente foi professor de muitos membros 
cariocas desta lista.

==
Mensagem  enviada  pelo  CIP  WebMAIL  - Nova Geração - v. 2.1
CentroIn Internet Provider  http://www.centroin.com.br
Tel: (21) 2542-4849, (21) 2295-3331Fax: (21) 2295-2978
Empresa 100% Brasileira - Desde 1992 prestando servicos online



=
Instruções para entrar na lista, sair da lista e usar a lista em
http://www.mat.puc-rio.br/~nicolau/olimp/obm-l.html
=


Re: [obm-l]

2004-07-30 Por tôpico Augusto Cesar de Oliveira Morgado
a+b = 5-c
ab + c(a+b)= 3
ab = 3-c(5-c)
a e b sao as raizes de X^2 - (5-c)X + (3-5c+c^2) = 0
delta >= 0
3c^2-10c-13<=0
-1 <= c <= 13/3
==
Mensagem  enviada  pelo  CIP  WebMAIL  - Nova Geração - v. 2.1
CentroIn Internet Provider  http://www.centroin.com.br
Tel: (21) 2542-4849, (21) 2295-3331Fax: (21) 2295-2978
Empresa 100% Brasileira - Desde 1992 prestando servicos online


-- Original Message ---


 Thiago wrote:
 
 a + b + c = 5  e ab + ac + bc = 3  (a, b e c são números reais) qual é 
 o máximo valor para c?

=
Instruções para entrar na lista, sair da lista e usar a lista em
http://www.mat.puc-rio.br/~nicolau/olimp/obm-l.html
=


[obm-l] "Nota de Falecimento"

2004-07-30 Por tôpico Augusto Cesar de Oliveira Morgado


Faleceu esta manhã, no Hospital Oswaldo Cruz, o Prof. Dr. Flávio Wagner
Rodrigues, aposentado do IME-USP.
O Prof. Dr. Flávio Wagner Rodrigues era um dos editores da Revista do 
Professor de Matemática, da SBM, e, por muitos anos, encarregado da seção de 
problemas da referida revista. Uma enorme perda para a Estatística e a 
Matemática brasileiras. 



==
Mensagem  enviada  pelo  CIP  WebMAIL  - Nova Geração - v. 2.1
CentroIn Internet Provider  http://www.centroin.com.br
Tel: (21) 2542-4849, (21) 2295-3331Fax: (21) 2295-2978
Empresa 100% Brasileira - Desde 1992 prestando servicos online

=
Instruções para entrar na lista, sair da lista e usar a lista em
http://www.mat.puc-rio.br/~nicolau/olimp/obm-l.html
=


Re: [obm-l] Problema Subconjuntos. Correção

2004-07-21 Por tôpico Augusto Cesar de Oliveira Morgado
Favor esquecer a bobagem abaixo. 
Morgado



-- Original Message ---
From: "Augusto Cesar de Oliveira Morgado" <[EMAIL PROTECTED]>
To: [EMAIL PROTECTED]
Sent: Wed, 21 Jul 2004 02:51:09 -0200
Subject: Re: RES: [obm-l] Problema Subconjuntos

> C(n-2;3). Basta usar o primeiro lema de Kaplansky.
> 
> 
> -- Original Message ---
> From: "David M. Cardoso" <[EMAIL PROTECTED]>
> To: <[EMAIL PROTECTED]>
> Sent: Tue, 20 Jul 2004 20:57:24 -0300
> Subject: RES: [obm-l] Problema Subconjuntos
> 
> > Cara, muito obrigado..
> > Sendo que ta dando trabalho pra eu entender algumas coisas,
> > como "teremos T[n-3] - T[n-4] subconjuntos com os elementos n-1 e n-2"..
> > hora eu penso que entendi, hora eu não entendo mais e fico tentando 
lembrar
> > pq eu fico entendido antes, talvez seja o nervosismo, talvez seja apenas
> > porque o raciocinio eh complicado demais pra mim..
> > 
> > Outra duvida que tenho é se é possível transformar a recorrência num
> > "polinomiozinho" em função de n ou se uma resposta desse tipo já esta
> > completa o suficiente..
> > 
> > []'s
> > David
> > 
> > > -Mensagem original-
> > > De: [EMAIL PROTECTED] 
> > > [mailto:[EMAIL PROTECTED] Em nome de Helder Suzuki
> > > Enviada em: terça-feira, 20 de julho de 2004 19:30
> > > Para: [EMAIL PROTECTED]
> > > Assunto: Re: [obm-l] Problema Subconjuntos
> > > 
> > > vamos ver, seguindo a dica de usar recorrencia
> > > 
> > > se T[n] for igual ao numero de subconjuntos do conjunto {1, 
> > > 2, ..., n} que nao contem 3 inteiros consecutivos.
> > > temos que:
> > > T[0] = 1
> > > {}
> > > 
> > > T[1] = 2
> > > {} e {1}
> > > 
> > > T[2] = 4
> > > {}, {1},
> > > {2} e {1, 2}
> > > 
> > > T[3] = 7
> > > {}, {1}, {2}, {1, 2},
> > > {3}, {1, 3}, {2, 3}
> > > 
> > > T[4] = 13
> > > {}, {1}, {2}, {3}, {1, 2}, {1, 3}, {2, 3}, {4}, {1, 4}, {2, 
> > > 4}, {3, 4}, {1, 2, 4}, {1, 3, 4}
> > > 
> > > bom, suponha que sabemos o valor de T[n-1], T[n-2], ..., 
> > > T[1]; como podemos achar T[n] em funcao de T[n-1]? humm...
> > > considere todos subconjuntos de {1, 2, 3, 4, ..., n-1} que 
> > > satisfazem a condicao do enunciado.
> > > se adicionarmos um elemento n, em quais desses subconjuntos o 
> > > n pode entrar e quais ele nao pode(para manter a condicao do 
> > > enunciado)?
> > > se n nao pode entrar em X subconjuntos, temos que T[n] = 
> > > T[n-1] + T[n-1] - X T[n] = 2*T[n-1] - X mas X eh o numero de 
> > > subconjuntos que tem os elementos
> > > n-1 e n-2.
> > > 
> > > imagine que temos os subconjnutos de {1, 2, ..., n-3} e 
> > > queremos adicionar os elementos n-1 e n-2 a esses 
> > > subconjuntos ao mesmo tempo, nesse caso só nao poderemos 
> > > adicionar n-1 e n-2 aos subconjuntos que tem o elemento n-3, 
> > > entao teremos T[n-3] - T[n-4] subconjuntos com os elementos n-1 e n-2:
> > > X = T[n-3] - T[n-4]
> > > 
> > > entao nossa recorrencia fica:
> > > T[n] = 2*T[n-1] - T[n-3] + T[n-4]
> > > 
> > > []'s,
> > > Helder
> > > 
> > > --- "David M. Cardoso" <[EMAIL PROTECTED]>
> > > escreveu: > 
> > > > 
> > > > Olá,
> > > > 
> > > > Alguem pode me ajudar? Não consegui resolver o seguinte problema:
> > > > 
> > > > "Quantos subconjuntos o conjunto {1,2,3,...,n} tais que não contêm 
> > > > três inteiros consecutivos?"
> > > > 
> > > > A dica dada na questão é: "Encontre uma recorrência." 
> > > Porém, qualquer 
> > > > solução (sem/com recorrência) vai ajudar.
> > > > 
> > > > []'s
> > > > David
> > > 
> > > 
> > >   
> > >   
> > >   
> > > ___
> > > Yahoo! Mail agora com 100MB, anti-spam e antivírus grátis!
> > > http://br.info.mail.yahoo.com/
> > > ==
> > > ===
> > > Instruções para entrar na lista, sair da lista e usar a lista 
> > > em http://www.mat.puc-rio.br/~nicolau/olimp/obm-l.html
> > > ==
> > > ===
> > >
> > 
> > =
> > Instruções para entrar na lista, sair da lista e usar a lista em
> > http://www.mat.puc-rio.br/~nicolau/olimp/obm-l.html
> > =
> --- End of Original Message ---
> 
> =
> Instruções para entrar na lista, sair da lista e usar a lista em
> http://www.mat.puc-rio.br/~nicolau/olimp/obm-l.html
> =
--- End of Original Message ---

=
Instruções para entrar na lista, sair da lista e usar a lista em
http://www.mat.puc-rio.br/~nicolau/olimp/obm-l.html
=


Re: RES: [obm-l] Problema Subconjuntos

2004-07-20 Por tôpico Augusto Cesar de Oliveira Morgado
C(n-2;3). Basta usar o primeiro lema de Kaplansky.

==
Mensagem  enviada  pelo  CIP  WebMAIL  - Nova Geração - v. 2.1
CentroIn Internet Provider  http://www.centroin.com.br
Tel: (21) 2542-4849, (21) 2295-3331Fax: (21) 2295-2978
Empresa 100% Brasileira - Desde 1992 prestando servicos online


-- Original Message ---
From: "David M. Cardoso" <[EMAIL PROTECTED]>
To: <[EMAIL PROTECTED]>
Sent: Tue, 20 Jul 2004 20:57:24 -0300
Subject: RES: [obm-l] Problema Subconjuntos

> Cara, muito obrigado..
> Sendo que ta dando trabalho pra eu entender algumas coisas,
> como "teremos T[n-3] - T[n-4] subconjuntos com os elementos n-1 e n-2"..
> hora eu penso que entendi, hora eu não entendo mais e fico tentando lembrar
> pq eu fico entendido antes, talvez seja o nervosismo, talvez seja apenas
> porque o raciocinio eh complicado demais pra mim..
> 
> Outra duvida que tenho é se é possível transformar a recorrência num
> "polinomiozinho" em função de n ou se uma resposta desse tipo já esta
> completa o suficiente..
> 
> []'s
> David
> 
> > -Mensagem original-
> > De: [EMAIL PROTECTED] 
> > [mailto:[EMAIL PROTECTED] Em nome de Helder Suzuki
> > Enviada em: terça-feira, 20 de julho de 2004 19:30
> > Para: [EMAIL PROTECTED]
> > Assunto: Re: [obm-l] Problema Subconjuntos
> > 
> > vamos ver, seguindo a dica de usar recorrencia
> > 
> > se T[n] for igual ao numero de subconjuntos do conjunto {1, 
> > 2, ..., n} que nao contem 3 inteiros consecutivos.
> > temos que:
> > T[0] = 1
> > {}
> > 
> > T[1] = 2
> > {} e {1}
> > 
> > T[2] = 4
> > {}, {1},
> > {2} e {1, 2}
> > 
> > T[3] = 7
> > {}, {1}, {2}, {1, 2},
> > {3}, {1, 3}, {2, 3}
> > 
> > T[4] = 13
> > {}, {1}, {2}, {3}, {1, 2}, {1, 3}, {2, 3}, {4}, {1, 4}, {2, 
> > 4}, {3, 4}, {1, 2, 4}, {1, 3, 4}
> > 
> > bom, suponha que sabemos o valor de T[n-1], T[n-2], ..., 
> > T[1]; como podemos achar T[n] em funcao de T[n-1]? humm...
> > considere todos subconjuntos de {1, 2, 3, 4, ..., n-1} que 
> > satisfazem a condicao do enunciado.
> > se adicionarmos um elemento n, em quais desses subconjuntos o 
> > n pode entrar e quais ele nao pode(para manter a condicao do 
> > enunciado)?
> > se n nao pode entrar em X subconjuntos, temos que T[n] = 
> > T[n-1] + T[n-1] - X T[n] = 2*T[n-1] - X mas X eh o numero de 
> > subconjuntos que tem os elementos
> > n-1 e n-2.
> > 
> > imagine que temos os subconjnutos de {1, 2, ..., n-3} e 
> > queremos adicionar os elementos n-1 e n-2 a esses 
> > subconjuntos ao mesmo tempo, nesse caso só nao poderemos 
> > adicionar n-1 e n-2 aos subconjuntos que tem o elemento n-3, 
> > entao teremos T[n-3] - T[n-4] subconjuntos com os elementos n-1 e n-2:
> > X = T[n-3] - T[n-4]
> > 
> > entao nossa recorrencia fica:
> > T[n] = 2*T[n-1] - T[n-3] + T[n-4]
> > 
> > []'s,
> > Helder
> > 
> > --- "David M. Cardoso" <[EMAIL PROTECTED]>
> > escreveu: > 
> > > 
> > > Olá,
> > > 
> > > Alguem pode me ajudar? Não consegui resolver o seguinte problema:
> > > 
> > > "Quantos subconjuntos o conjunto {1,2,3,...,n} tais que não contêm 
> > > três inteiros consecutivos?"
> > > 
> > > A dica dada na questão é: "Encontre uma recorrência." 
> > Porém, qualquer 
> > > solução (sem/com recorrência) vai ajudar.
> > > 
> > > []'s
> > > David
> > 
> > 
> > 
> > 
> > 
> > ___
> > Yahoo! Mail agora com 100MB, anti-spam e antivírus grátis!
> > http://br.info.mail.yahoo.com/
> > ==
> > ===
> > Instruções para entrar na lista, sair da lista e usar a lista 
> > em http://www.mat.puc-rio.br/~nicolau/olimp/obm-l.html
> > ==
> > ===
> >
> 
> =
> Instruções para entrar na lista, sair da lista e usar a lista em
> http://www.mat.puc-rio.br/~nicolau/olimp/obm-l.html
> =
--- End of Original Message ---

=
Instruções para entrar na lista, sair da lista e usar a lista em
http://www.mat.puc-rio.br/~nicolau/olimp/obm-l.html
=


Re: [obm-l] Espacial

2004-07-18 Por tôpico Augusto Cesar de Oliveira Morgado
Pelamordideus, Fael é Fael. Se a gente usar os nomes, cria-se uma enorme 
dificuldade de localizar mensagens. Se o nome é Rafael, é o caos porque há 
pelo menos 5 na lista.
[]s
Morgado


-- Original Message ---
From: [EMAIL PROTECTED]
To: [EMAIL PROTECTED]
Sent: Sun, 18 Jul 2004 19:21:48 +
Subject: Re: [obm-l] Espacial


> 
> A solução do [EMAIL PROTECTED] (desculpe, não sei seu nome!) ...
=
Instruções para entrar na lista, sair da lista e usar a lista em
http://www.mat.puc-rio.br/~nicolau/olimp/obm-l.html
=


Re: [obm-l] RE: [obm-l] Re: [obm-l] questõesinhas....

2004-07-18 Por tôpico Augusto Cesar de Oliveira Morgado
1/3; 2+sqrt3; 2-sqrt3

==
Mensagem  enviada  pelo  CIP  WebMAIL  - Nova Geração - v. 2.1
CentroIn Internet Provider  http://www.centroin.com.br
Tel: (21) 2542-4849, (21) 2295-3331Fax: (21) 2295-2978
Empresa 100% Brasileira - Desde 1992 prestando servicos online


-- Original Message ---
From: "Daniel Regufe" <[EMAIL PROTECTED]>
To: [EMAIL PROTECTED]
Sent: Sun, 18 Jul 2004 17:46:23 +
Subject: [obm-l] RE: [obm-l] Re: [obm-l] questõesinhas

> OPA  João ...  vc ta confundindo !
> 
> Produto (2à2) = 7/3
> Atotal = 2*7/3 = 14/3
> Produto (3à3) = 1/3 = Volume
> Atotal/Volume = 14
> 
> As raizes eu nao consegui achar nao.
> 
> []`s
> Regufe
> 
> >From: João Vitor <[EMAIL PROTECTED]>
> >Reply-To: [EMAIL PROTECTED]
> >To: <[EMAIL PROTECTED]>
> >Subject: [obm-l] Re: [obm-l] questõesinhas
> >Date: Sun, 18 Jul 2004 14:19:16 -0300
> >
> >Sua questão aí!
> >
> >13) 3x³ -13x² +7x -1=0
> >  a)
> >Girrard --> Produto = C/A =7/3
> >   --> Produto(2à2) = -D/A =1/3
> >
> >Volume = Produto = 7/3 u.v
> >Atotal = 2 .Produto(2à2) = 2 . 1/3 = 2/3 u.a.
> >
> >Seria isso?
> >   - Original Message -
> >   From: [EMAIL PROTECTED]
> >   To: [EMAIL PROTECTED]
> >   Sent: Saturday, July 17, 2004 11:35 PM
> >   Subject: [obm-l] questõesinhas
> >
> >
> >   13) As dimensões de um paralelepípido retângulo são dadas pelas raízes 
> >do polinômio a seguir.
> >
> >   3x³-13x²+7x-1
> >
> >   Em relação a esse paralelepípedo, determine:
> >   a) a razão entre a sua área total e seu volume
> >   b)suas dimensões
> >
> >
> >   CN)
> >   X^4-4(m+2)x²+m²=0 admite quatro raízes reais então:
> >   a) o maior valor inteiro de m é -3
> >   b)a soma dos três menores valores inteiros de m é zero
> >   c) a soma dos três menores valores de m é -12
> >   d) só existem valores inteiros e positivos para m
> >   e) só existem valores negativos para m
> >
> >   abços
> >
> >   Junior
> 
> _
> MSN Messenger: converse com os seus amigos online.  
> http://messenger.msn.com.br
> 
> =
> Instruções para entrar na lista, sair da lista e usar a lista em
> http://www.mat.puc-rio.br/~nicolau/olimp/obm-l.html
> =
--- End of Original Message ---

=
Instruções para entrar na lista, sair da lista e usar a lista em
http://www.mat.puc-rio.br/~nicolau/olimp/obm-l.html
=


Re: [obm-l] Problema em semi-aberto

2004-07-16 Por tôpico Augusto Cesar de Oliveira Morgado



Numere os setores. A estrategia sera contar todos os modos permitindo os 
setores 1 e 10 com a mesma cor e depois descontar os modos em que o primeiro e o ultimo setores tem a mesma cor. Vou chamar de A(10) o numero de modos de colorir o circulo com 10 setores. 
A(10) = 3*[2^9]- A(9) 
A(9) = 3*[2^8] - A( 8) 
 
A(4) = 3*[2^3] - A(3) 
A(3) = 6 
Substitua e faça a conta. 


== 
Mensagem  enviada  pelo  CIP  WebMAIL  - Nova Geração - v. 2.1 
CentroIn Internet Provider          http://www.centroin.com.br 
Tel: (21) 2542-4849, (21) 2295-3331        Fax: (21) 2295-2978 
Empresa 100% Brasileira - Desde 1992 prestando servicos online 


-- Original Message ---
From: [EMAIL PROTECTED] 
To: [EMAIL PROTECTED] 
Sent: Fri, 16 Jul 2004 02:44:17 EDT 
Subject: [obm-l] Problema em semi-aberto 

> Ola pessoal, 
> 
> Aqui na lista foi dada uma solucao por equacoes de recorrencia --por isso disse semi-aberto no thread -- mas acredito que haja uma solucao atraves de matematica de Ensino medio -- por combinatoria, talvez. Pois este problema caiu na OBM de 1997 (fase senior) . 
> Alguem se propoe a resolve-lo com Matematica de Ensino medio ? 
> 
> 1) 
> 
> Os vertices de um decagono regular convexo ABC...J devem ser coloridos usando-se apenas as cores verde, amarela e azul. De quantos modos isso pode ser feito se vertices adjacentes não podem receber a mesma cor? 
> 
> a)1022 b)1024 c)1026 d)1524 e)1536 
> 
> 
--- End of Original Message ---






Re: [obm-l] Dúvida

2004-07-10 Por tôpico Augusto Cesar de Oliveira Morgado
Não, x^2+13x+61 eh divisivel por 3 para x=1 e seu minimo eh 18,75.

==
Mensagem  enviada  pelo  CIP  WebMAIL  - Nova Geração - v. 2.1
CentroIn Internet Provider  http://www.centroin.com.br
Tel: (21) 2542-4849, (21) 2295-3331Fax: (21) 2295-2978
Empresa 100% Brasileira - Desde 1992 prestando servicos online


-- Original Message ---
From: [EMAIL PROTECTED]
To: [EMAIL PROTECTED]
Sent: Sun, 11 Jul 2004 02:12:53 +
Subject: Re: [obm-l] Dúvida

> Algumas partes da minha mensagem foram apagadas; logo na primeira 
> linha, faltou " congruente a 0 módulo 2,3,5,7,11,13 ".
> 
> Sobre a pergunta no final, é falsa em, por exemplo, x^2 + 5x + 22, 
> onde o mínimo é 15.75 mas 2 divide y quando x = 1, ou 11 divide y 
> quando x = 11... A pergunta, portanto, deveria ser:
> 
> Em y = x^2 + q*x + p, com p e q primos positivos, q primo que divide y para x inteiro é o primo maior ou igual ao mínimo 
> de y?
> 
> []s,
> Daniel
> 
> [EMAIL PROTECTED] escreveu:
> >
> >Bem, y = x^2 + 5x + 23 não pode ser congruente a 0 módulo ,
> >e para ver isso, só consegui provando caso a caso. Para ilustrar:
> >
> >A incongruência a 0 módulo 2 é verificada facilmente pois, se x é par, y é
> >ímpar, e se x é ímpar, x^2 + 5x é par donde y é ímpar.
> >
> >Prosseguindo, se fosse x^2 + 5x + 23 == 0 (mod 3), teríamos
> >x^2 + 5x == 1 (mod 3)
> >x*(x+5) == 1 (mod 3)
> >x*(x + 2) == 1 (mod 3), como x não congruente a 0 ou 1 módulo 3.
> >Logo, só pode ser x == 2(mod 3), mas isto leva a x*(x+2) == 2 (mod 3),
> >contradição.
> >
> >Se eu não errei nada, encontrei contradições até p = 17, em que basta tomar
> >x = -3 (ou x=-2) --> y = 17.
> >
> >Vale observar que 17 é, como se era de esperar, o menor inteiro positivo
> >assumido por y, visto que o mínimo da função é 16,75 quando x= -2.5.
> >
> >A pergunta é: será que o fato do mínimo de y ser 16,75 implica,
> >necessariamente, que nenhum primo menor que 17 divida y?
> >
> >[]s,
> >Daniel
> >
> >[EMAIL PROTECTED] escreveu:
> >>
> >>Determine o menor número primo positivo que divide x² + 5x + 23 para algum
> >>inteiro x.
> >>
> >>Peço ajuda para todos os colegas da lista e agradeço previamente,
> >>Matheus
> >>
> >
> >=
> >Instruções para entrar na lista, sair da lista e usar a lista em
> >http://www.mat.puc-rio.br/~nicolau/olimp/obm-l.html
> >=
> >
> 
> =
> Instruções para entrar na lista, sair da lista e usar a lista em
> http://www.mat.puc-rio.br/~nicolau/olimp/obm-l.html
> =
--- End of Original Message ---

=
Instruções para entrar na lista, sair da lista e usar a lista em
http://www.mat.puc-rio.br/~nicolau/olimp/obm-l.html
=


Re: [obm-l] Dúvida

2004-07-10 Por tôpico Augusto Cesar de Oliveira Morgado
Nao, x^2+17 tem minimo 17 e eh divisivel por 13 quando x=3.

==
Mensagem  enviada  pelo  CIP  WebMAIL  - Nova Geração - v. 2.1
CentroIn Internet Provider  http://www.centroin.com.br
Tel: (21) 2542-4849, (21) 2295-3331Fax: (21) 2295-2978
Empresa 100% Brasileira - Desde 1992 prestando servicos online


-- Original Message ---
From: [EMAIL PROTECTED]
To: [EMAIL PROTECTED]
Sent: Sun, 11 Jul 2004 01:27:28 +
Subject: Re: [obm-l] Dúvida

> Bem, y = x^2 + 5x + 23 não pode ser congruente a 0 módulo {2,3,5,7,
> ...,13}, e para ver isso, só consegui provando caso a caso. Para ilustrar:
> 
> A incongruência a 0 módulo 2 é verificada facilmente pois, se x é 
> par, y é ímpar, e se x é ímpar, x^2 + 5x é par donde y é ímpar.
> 
> Prosseguindo, se fosse x^2 + 5x + 23 == 0 (mod 3), teríamos
> x^2 + 5x == 1 (mod 3)
> x*(x+5) == 1 (mod 3)
> x*(x + 2) == 1 (mod 3), como x não congruente a 0 ou 1 módulo 3.
> Logo, só pode ser x == 2(mod 3), mas isto leva a x*(x+2) == 2 (mod 3)
> , contradição.
> 
> Se eu não errei nada, encontrei contradições até p = 17, em que 
> basta tomar x = -3 (ou x=-2) --> y = 17.
> 
> Vale observar que 17 é, como se era de esperar, o menor inteiro positivo
> assumido por y, visto que o mínimo da função é 16,75 quando x= -2.5.
> 
> A pergunta é: será que o fato do mínimo de y ser 16,75 implica,
> necessariamente, que nenhum primo menor que 17 divida y?
> 
> []s,
> Daniel
> 
> [EMAIL PROTECTED] escreveu:
> >
> >Determine o menor número primo positivo que divide x² + 5x + 23 para algum
> >inteiro x.
> >
> >Peço ajuda para todos os colegas da lista e agradeço previamente,
> >Matheus
> >
> 
> =
> Instruções para entrar na lista, sair da lista e usar a lista em
> http://www.mat.puc-rio.br/~nicolau/olimp/obm-l.html
> =
--- End of Original Message ---

=
Instruções para entrar na lista, sair da lista e usar a lista em
http://www.mat.puc-rio.br/~nicolau/olimp/obm-l.html
=


Re: [obm-l] Dúvida

2004-07-10 Por tôpico Augusto Cesar de Oliveira Morgado



Para x=14, a expressão vale 289, que é divisível por 17.
Para mostrar que 17 é o menor primo, devemos mostrar que a expressão nunca é divisível por 2, nem por 3, nem por 5, nem por 7, nem por 11, nem por 13.
Vou mostrar que ela nao eh divisivel por 7 (os demais casos sao analogos)
x² + 5x + 23 eh congruo, modulo 7, a  x² - 2x + 2. Se x for congruo a 0, 1, 2, 3, 4, 5 ou 6,  x² - 2x + 2 sera congruo a  2, 1, 2, 5, 3, 3, 5; logo, nao eh congruo a 0.
== 
Mensagem  enviada  pelo  CIP  WebMAIL  - Nova Geração - v. 2.1 
CentroIn Internet Provider          http://www.centroin.com.br 
Tel: (21) 2542-4849, (21) 2295-3331        Fax: (21) 2295-2978 
Empresa 100% Brasileira - Desde 1992 prestando servicos online 


-- Original Message ---
From: [EMAIL PROTECTED] 
To: [EMAIL PROTECTED] 
Sent: Sat, 10 Jul 2004 14:09:00 EDT 
Subject: [obm-l] Dúvida 

> Determine o menor número primo positivo que divide x² + 5x + 23 para algum inteiro x.
> 
> Peço ajuda para todos os colegas da lista e agradeço previamente,
> Matheus 
--- End of Original Message ---






Re: [obm-l] Duvidas

2004-07-07 Por tôpico Augusto Cesar de Oliveira Morgado
A, em 4 horas, fez 4/6 = 2/3 da tarefa. Resta 1/3. B fara esse 1/3 em 1/3 de 
10 hotas, ou seja, em 3h20min.
A

==
Mensagem  enviada  pelo  CIP  WebMAIL  - Nova Geração - v. 2.1
CentroIn Internet Provider  http://www.centroin.com.br
Tel: (21) 2542-4849, (21) 2295-3331Fax: (21) 2295-2978
Empresa 100% Brasileira - Desde 1992 prestando servicos online


-- Original Message ---
From: "aryqueirozq" <[EMAIL PROTECTED]>
To: "obm-l" <[EMAIL PROTECTED]>
Sent: Wed,  7 Jul 2004 00:00:08 -0300
Subject: [obm-l] Duvidas

> O pedreiro A executa determinada tarefa em 6 horas de 
> trabalho. A mesma tarefa é executada pelo pedreiro B em 
> 10 horas de trabalho. Se A , após de trabalhar 4 
> horas , deixasse o restante para B concluir , este 
> terminaria a tarefa em:
> 
> a) 3 h 20min
> b) 3h 300min
> c) 2h 40min
> d) 3 h 30min
> e) 3h
> 
>  Agradeço desde de já.
> 
> 
> __
> Acabe com aquelas janelinhas que pulam na sua tela.
> AntiPop-up UOL - É grátis!
> http://antipopup.uol.com.br/
> 
> __
> Acabe com aquelas janelinhas que pulam na sua tela.
> AntiPop-up UOL - É grátis!
> http://antipopup.uol.com.br/
> 
> =
> Instruções para entrar na lista, sair da lista e usar a lista em
> http://www.mat.puc-rio.br/~nicolau/olimp/obm-l.html
> =
--- End of Original Message ---

=
Instruções para entrar na lista, sair da lista e usar a lista em
http://www.mat.puc-rio.br/~nicolau/olimp/obm-l.html
=


Re: [obm-l] Trigonometria

2004-07-01 Por tôpico Augusto Cesar de Oliveira Morgado
cos 3x = sen 2x
cos 3x = cos (pi/2 - 2x)
3x + pi/2 - 2x = 2kpi ou 3x -pi/2 +2x = 2kpi 
x = 2kpi - pi/2  ou  x = 2kpi/5 + pi/10
==
Mensagem  enviada  pelo  CIP  WebMAIL  - Nova Geração - v. 2.1
CentroIn Internet Provider  http://www.centroin.com.br
Tel: (21) 2542-4849, (21) 2295-3331Fax: (21) 2295-2978
Empresa 100% Brasileira - Desde 1992 prestando servicos online
=
Instruções para entrar na lista, sair da lista e usar a lista em
http://www.mat.puc-rio.br/~nicolau/olimp/obm-l.html
=


Re: [obm-l] Polinômio

2004-06-24 Por tôpico Augusto Cesar de Oliveira Morgado
Se A fosse divisivel por B, AC+BD=1 tambem o seria, o que eh absurdo pois 
grau de B >1.

==
Mensagem  enviada  pelo  CIP  WebMAIL  - Nova Geração - v. 2.1
CentroIn Internet Provider  http://www.centroin.com.br
Tel: (21) 2542-4849, (21) 2295-3331Fax: (21) 2295-2978
Empresa 100% Brasileira - Desde 1992 prestando servicos online


-- Original Message ---
From: "Daniel Regufe" <[EMAIL PROTECTED]>
To: [EMAIL PROTECTED]
Sent: Thu, 24 Jun 2004 11:13:43 +
Subject: [obm-l] Polinômio

> Gostaria de uma ajuda nessa questão ...
> 
>   Sejam A(x) e B(x) polinômios de grau maior que um e admita que 
> existam polinômios C(x) e D(x) tais que a igualdade abaixo se 
> verifica A(x)*C(x) + B(x)*D(x) = 1( para todo x pertencente aos 
> reais ) Prove que A(x) não é divisível por B(x).
> 
> Abraços ,
> 
> Daniel Regufe
> 
> _
> MSN Messenger: converse com os seus amigos online.  
> http://messenger.msn.com.br
> 
> =
> Instruções para entrar na lista, sair da lista e usar a lista em
> http://www.mat.puc-rio.br/~nicolau/olimp/obm-l.html
> =
--- End of Original Message ---

=
Instruções para entrar na lista, sair da lista e usar a lista em
http://www.mat.puc-rio.br/~nicolau/olimp/obm-l.html
=


Re: [obm-l] Probabilidade!!!

2004-06-23 Por tôpico Augusto Cesar de Oliveira Morgado



B ganha se e somente se ganha as proximas 3 partidas, o que ocorre com probabilidade 1/8.
Com muito boa vontade em relaçao a esse enunciado misterioso (repartir do ponto de vista probabilistico é um primor de obscuridade -- eu sei, o enunciado não é seu, é de um vestibular paulista), B deve receber 1/8 do total apostado e A, 7/8.

Deveriam ter dito "repartindo proporcionalmente à probabilidade que cada um tem de ganhar".
== 
Mensagem  enviada  pelo  CIP  WebMAIL  - Nova Geração - v. 2.1 
CentroIn Internet Provider          http://www.centroin.com.br 
Tel: (21) 2542-4849, (21) 2295-3331        Fax: (21) 2295-2978 
Empresa 100% Brasileira - Desde 1992 prestando servicos online 


-- Original Message ---
From: Carlos Alberto <[EMAIL PROTECTED]> 
To: [EMAIL PROTECTED] 
Sent: Wed, 23 Jun 2004 07:04:53 -0300 (ART) 
Subject: [obm-l] Probabilidade!!! 

> 
> Alguém poderia me ajudar com esse exercicio? 
> Dois individuos A e B vão jogar Cara ou Coroa com uma moeda honesta. Eles combinam lançar a moeda 5 vezes, e ganha o jogo aquele que ganhar em 3 ou mais lançamentos. Cada um aposta R$ 2.800,00. Feito os dois primeiros lançamentos, em ambos os quais A vence, eles resolvem encerrar o jogo. Do ponto de vista probabilistico, de que forma devem ser repartidos os R$ 5.600,00? 
>   
> Desde já agradeço!!! 
> Carlos
> 
> 
Yahoo! Messenger - Fale com seus amigos online. Instale agora! 
--- End of Original Message ---






Re: [obm-l] Integrais de funçõesímpares

2004-06-23 Por tôpico Augusto Cesar de Oliveira Morgado
f par e derivavel  implica f(-x) = f(x) implica -f'(-x) = f'(x)implica f' 
impar.
f impar e derivavel  implica f(-x) = -f(x) implica -f'(-x) = -f'(x)implica f' 
par.

==
Mensagem  enviada  pelo  CIP  WebMAIL  - Nova Geração - v. 2.1
CentroIn Internet Provider  http://www.centroin.com.br
Tel: (21) 2542-4849, (21) 2295-3331Fax: (21) 2295-2978
Empresa 100% Brasileira - Desde 1992 prestando servicos online


-- Original Message ---
From: "Henrique Patrício Sant'Anna Branco" <[EMAIL PROTECTED]>
To: <[EMAIL PROTECTED]>
Sent: Wed, 23 Jun 2004 01:18:12 -0300
Subject: [obm-l] Integrais de funções ímpares

> Pessoal,
> 
> Ensinando primitivação (integrais simples) para uma colega, notei 
> que a integral de funções polinomiais ímpares e da função seno, por 
> exemplo, gera uma primitiva representada por uma função par.
> 
> Isso é um resultado geral? Quero dizer:
> Seja f(x) uma função ímpar com primitiva elementar. Int f(x) dx 
> sempre vai ser uma função par? Como demonstro?
> 
> Obrigado.
> Henrique.
> 
> =
> Instruções para entrar na lista, sair da lista e usar a lista em
> http://www.mat.puc-rio.br/~nicolau/olimp/obm-l.html
> =
--- End of Original Message ---

=
Instruções para entrar na lista, sair da lista e usar a lista em
http://www.mat.puc-rio.br/~nicolau/olimp/obm-l.html
=


Re: [obm-l] poligono

2004-06-20 Por tôpico Augusto Cesar de Oliveira Morgado
Parece que pisei na bola. Não tenho mais o enunciado, mas pensei que X e Y 
eram vértices e O era o centro.

==
Mensagem  enviada  pelo  CIP  WebMAIL  - Nova Geração - v. 2.1
CentroIn Internet Provider  http://www.centroin.com.br
Tel: (21) 2542-4849, (21) 2295-3331Fax: (21) 2295-2978
Empresa 100% Brasileira - Desde 1992 prestando servicos online


-- Original Message ---
From: Fábio Dias Moreira <[EMAIL PROTECTED]>
To: [EMAIL PROTECTED]
Sent: Sun, 20 Jun 2004 19:59:11 -0300
Subject: Re: [obm-l] poligono

> -BEGIN PGP SIGNED MESSAGE-
> Hash: SHA1
> 
> "Augusto Cesar de Oliveira Morgado" <[EMAIL PROTECTED]> said:
> > (F) Todas sao falsas.
> >
> > 22,5 graus eh o angulo central de um poligono de 16 lados. Se X e Y sao
> > vertices consecutivos, n=16. Se nao forem consecutivos, n serah multiplo 
de
> > 16. [...]
> 
> Mas o ângulo inscrito na circunferência vale metade do ângulo 
> central, logo basta que 8|n, não? O ponto O é um dos vértices do 
> polígono, não o seu centro.
> 
> []s,
> 
> - -- 
> Fábio Dias Moreira
> -BEGIN PGP SIGNATURE-
> Version: GnuPG v1.2.3 (GNU/Linux)
> 
> iD8DBQFA1hbEalOQFrvzGQoRAv+CAKDKLXKCnP5VutDVaxfh7Ilo9roFkQCfb6s2
> uD636mHDGdFisml1THNOMUQ=
> =TsNf
> -END PGP SIGNATURE-
> 
> =
> Instruções para entrar na lista, sair da lista e usar a lista em
> http://www.mat.puc-rio.br/~nicolau/olimp/obm-l.html
> =
--- End of Original Message ---

=
Instruções para entrar na lista, sair da lista e usar a lista em
http://www.mat.puc-rio.br/~nicolau/olimp/obm-l.html
=


Re: [obm-l] poligono

2004-06-20 Por tôpico Augusto Cesar de Oliveira Morgado



(F) Todas sao falsas.

22,5 graus eh o angulo central de um poligono de 16 lados. Se X e Y sao vertices consecutivos, n=16. Se nao forem consecutivos, n serah multiplo de 16.
== 
Mensagem  enviada  pelo  CIP  WebMAIL  - Nova Geração - v. 2.1 
CentroIn Internet Provider          http://www.centroin.com.br 
Tel: (21) 2542-4849, (21) 2295-3331        Fax: (21) 2295-2978 
Empresa 100% Brasileira - Desde 1992 prestando servicos online 


-- Original Message ---
From: "Eduardo Soares" <[EMAIL PROTECTED]> 
To: "obm" <[EMAIL PROTECTED]> 
Sent: Sun, 20 Jun 2004 16:02:24 -0300 
Subject: [obm-l] poligono 

> Os pontos X, O e Y são vértices de um polígono regular
> de n lados. Se o ângulo XOY mede 22 30', considere as
> afirmativas:
> 
> ( I ) n pode ser igual a 8.
> ( II ) n pode ser igual a 12.
> ( III ) n pode ser igual a 24.
> 
> Podemos afirmar que:
> A) apenas I e II são verdadeiras.
> B) apenas I e III são verdadeiras.
> C) apenas II e III são verdadeiras.
> D) apenas uma delas é verdadeira.
> E) I, II e III são verdadeiras 
--- End of Original Message ---






Re: [obm-l] Re:_[obm-l]_Polígonos_Construtíveis

2004-06-16 Por tôpico Augusto Cesar de Oliveira Morgado
Quem te disse que a bola de peso diferente do das demais é mais pesada?

==
Mensagem  enviada  pelo  CIP  WebMAIL  - Nova Geração - v. 2.1
CentroIn Internet Provider  http://www.centroin.com.br
Tel: (21) 2542-4849, (21) 2295-3331Fax: (21) 2295-2978
Empresa 100% Brasileira - Desde 1992 prestando servicos online


-- Original Message ---
From: [EMAIL PROTECTED]
To: [EMAIL PROTECTED]
Sent: Wed, 16 Jun 2004 19:06:00 -0300
Subject: Re: [obm-l] Re:_[obm-l]_Polígonos_Constr utíveis

> 1)DIVIDA AS BOLAS EM 2 GRUPOS DE 6 , E  PONHA NA BALANÇA 
> A QUE PESAR MAIS CONTEM A BOLA MAIS PESADA
> 
> 2)AGORA DIVIDA O GRUPO DE 6 EM 2 DE TRES  , O QUE PESAR MAIS CONTEM 
> A BOLA MAIS PESADA
> 
> 3) ENTAO AGORA TENHO 3 BOLAS , PESO 2 DE UM LADO E UMA DO OUTRO
> 
> SE A BALANÇA PESAR MAIS DO LADO QUE SO TEM UMA BOLA ENTAO ESSA É A 
> MAIS PESADA , SENAO SERA NECESSARIO ,MAIS UMA PESAGEM
> 
>  UM ABRAÇO ESPERO TER AJUDADO ( REINALDO BELLINI GONÇALVES )
> 
> 
__
___
> Quer mais velocidade?
> Só com o acesso Aditivado iG, a velocidade que você quer na hora que 
> você precisa. Clique aqui: http://www.acessoaditivado.ig.com.br
--- End of Original Message ---

=
Instruções para entrar na lista, sair da lista e usar a lista em
http://www.mat.puc-rio.br/~nicolau/olimp/obm-l.html
=


Re: [obm-l] polinomio interpolador na forma de newton

2004-06-16 Por tôpico Augusto Cesar de Oliveira Morgado
An introduction to the calculus of finite differences
Richardson

==
Mensagem  enviada  pelo  CIP  WebMAIL  - Nova Geração - v. 2.1
CentroIn Internet Provider  http://www.centroin.com.br
Tel: (21) 2542-4849, (21) 2295-3331Fax: (21) 2295-2978
Empresa 100% Brasileira - Desde 1992 prestando servicos online


-- Original Message ---
From: niski <[EMAIL PROTECTED]>
To: [EMAIL PROTECTED]
Sent: Wed, 16 Jun 2004 15:55:30 -0300
Subject: [obm-l] polinomio interpolador na forma de newton

> Estou estudando interpolacao polinomial pelo livro da Ana Flora 
> Humes, Ines Homem de Melo, Luzia Yoshida e Wagner Tunis Martins. O 
> livro é muito bom, mas particularmente nessa parte do polinomio 
> interpolador na forma de newton as provas sao na maior parte feitas 
> por indução sonolentas e gigantes. Lamentavel o livro nao traga 
> referencias historicas, mas acredito que o metodo foi criado por 
> Newton e certamente ele nao usou inducao para chegar aos mesmos 
> resultados. Assim eu pergunto: Alguem conhece algum lugar (site, 
> livro) onde eu possa ver as ideias originais do Newton? Obrigado.
> 
> -- 
> Niski - http://www.linux.ime.usp.br/~niski
> 
> [upon losing the use of his right eye]
> "Now I will have less distraction"
> Leonhard Euler
> 
> =
> Instruções para entrar na lista, sair da lista e usar a lista em
> http://www.mat.puc-rio.br/~nicolau/olimp/obm-l.html
> =
--- End of Original Message ---

=
Instruções para entrar na lista, sair da lista e usar a lista em
http://www.mat.puc-rio.br/~nicolau/olimp/obm-l.html
=


Re: [obm-l] Quadrado perfeito

2004-06-10 Por tôpico Augusto Cesar de Oliveira Morgado



x = 100 004
N = (x-4)(x-2)(x+2)(x+4) +n = (x^2-16)(x^2-4)+n = x^4 -20x^2+64+n = (x^2-10)^2+(n-36)

Se n=36, N eh quadrado perfeito.
== 
Mensagem  enviada  pelo  CIP  WebMAIL  - Nova Geração - v. 2.1 
CentroIn Internet Provider          http://www.centroin.com.br 
Tel: (21) 2542-4849, (21) 2295-3331        Fax: (21) 2295-2978 
Empresa 100% Brasileira - Desde 1992 prestando servicos online 


-- Original Message ---
From: Fábio Bernardo <[EMAIL PROTECTED]> 
To: "OBM" <[EMAIL PROTECTED]> 
Cc: <[EMAIL PROTECTED]> 
Sent: Thu, 10 Jun 2004 12:09:05 -0300 
Subject: [obm-l] Quadrado perfeito 

>   
> O meno inteiro positivo n para o qual o número 
>   
> N = 10.12.16.18+n 
>   
> é um quadrado perfeito é: 
> a) 30 
> b) 32 
> c) 34 
> d) 36 
> e) 38 
--- End of Original Message ---






Re: [obm-l] aritmética

2004-06-10 Por tôpico Augusto Cesar de Oliveira Morgado
Este problema ja foi respondido pouco depois de haver sido enviado pela 
primeira vez (esta ja eh a terceira). Consulte os arquivos.

==
Mensagem  enviada  pelo  CIP  WebMAIL  - Nova Geração - v. 2.1
CentroIn Internet Provider  http://www.centroin.com.br
Tel: (21) 2542-4849, (21) 2295-3331Fax: (21) 2295-2978
Empresa 100% Brasileira - Desde 1992 prestando servicos online


-- Original Message ---
From: elton francisco ferreira <[EMAIL PROTECTED]>
To: [EMAIL PROTECTED]
Sent: Thu, 10 Jun 2004 12:37:42 -0300 (ART)
Subject: [obm-l] aritmética

> Um reservatório é alimentado por duas torneiras: a
> primeira dá 38 litros por minuto e a segunda, 47. A
> saída de água é por um orifício que deixa passar 21
> litros por minuto, deixando abertas as torneiras e o
> orifício, o reservatório se enche em 680 minutos. Qual
> é a sua capacidade?
> 
> __
> 
> Participe da pesquisa global sobre o Yahoo! Mail: 
> http://br.surveys.yahoo.com/global_mail_survey_br
> =
> Instruções para entrar na lista, sair da lista e usar a lista em
> http://www.mat.puc-rio.br/~nicolau/olimp/obm-l.html
> =
--- End of Original Message ---

=
Instruções para entrar na lista, sair da lista e usar a lista em
http://www.mat.puc-rio.br/~nicolau/olimp/obm-l.html
=


Re: [obm-l] Probleminha

2004-06-10 Por tôpico Augusto Cesar de Oliveira Morgado



Como um carro anda com 4 pneus, a soma dos quilometros rodados pelos pneus todos é 
4*20 000 = 80 000 km. Se os 5 pneus foram usados igualmente, cada um rodou 80 000/5 =
16 000 km.

== 
Mensagem  enviada  pelo  CIP  WebMAIL  - Nova Geração - v. 2.1 
CentroIn Internet Provider          http://www.centroin.com.br 
Tel: (21) 2542-4849, (21) 2295-3331        Fax: (21) 2295-2978 
Empresa 100% Brasileira - Desde 1992 prestando servicos online 


-- Original Message ---
From: Fábio Bernardo <[EMAIL PROTECTED]> 
To: "OBM" <[EMAIL PROTECTED]> 
Sent: Thu, 10 Jun 2004 12:50:36 -0300 
Subject: [obm-l] Probleminha 

> Pessoal, tô enrolado nesse. Ajudem-me por favor. 
>   
> Em um carro foram usados os 4 pneus mais o estepe, rodando igualmente a mesma quilometragem. Após o carro ter percorrido 2km, cada pneu foi usado por: 
> a) 2km 
> b) 16000km 
> c) 12000km 
> d) 8000km 
> e) 4000km 
>   
>   
--- End of Original Message ---






Re: [obm-l] Re:[obm-l] Re: [obm-l] Re: [obm-l] funçãomonótona

2004-06-06 Por tôpico Augusto Cesar de Oliveira Morgado
Nem se existir. f(x)=x^3 eh estritamente crescente em [-1;1] e f'(0)=0.

==
Mensagem  enviada  pelo  CIP  WebMAIL  - Nova Geração - v. 2.1
CentroIn Internet Provider  http://www.centroin.com.br
Tel: (21) 2542-4849, (21) 2295-3331Fax: (21) 2295-2978
Empresa 100% Brasileira - Desde 1992 prestando servicos online


-- Original Message ---
From: "Osvaldo" <[EMAIL PROTECTED]>
To: "obm-l" <[EMAIL PROTECTED]>
Sent: Sun,  6 Jun 2004 03:41:53 -0300
Subject: [obm-l]  Re:[obm-l] Re: [obm-l] Re: [obm-l] função  monótona

> Seja a funçao f definida em um intervalo [a,b] por ex.
> Para que f seja estrit. crescente teremos que
> para quaisquer x_1, x_2 pertencentes a [a,b], o fato 
> de x_1 
> Bom, SE EXISTIR derivada teremos que ela não se 
> anulará em (a,b), seria um lema facil de ser mostrado.
> 
> Desculpe meu equivoco anterior. Fui.
> 
> > o que é uma função estritamente crescente?
> > 
> > fabiano
> >   - Original Message - 
> >   From: Lista OBM 
> >   To: [EMAIL PROTECTED] 
> >   Sent: Saturday, June 05, 2004 9:00 PM
> >   Subject: Re: [obm-l] Re: [obm-l] função monótona
> > 
> > 
> >   Osvaldo, ainda não vi diferenciabilidade.
> > 
> >   Osvaldo <[EMAIL PROTECTED]> wrote: 
> > Acredito que seja um dos tipos de funçoes abaixo:
> > 
> > Estritamente crescente;
> > Estritamente decrescente;
> > Crescente;
> > Decrescene;
> > 
> > Os dois primeiros tipos de funçoes monotonas 
> acima tem 
> > a prop. de que a derivada de primeira ordem 
> nunca se 
> > anula e os dois restantes que ela nao é nula em 
> todo 
> > intervalo, porem podendo anular se em um 
> subconjunto 
> > do domínio.
> > 
> > Nao sei se isso te ajuda mais to mandando mesmo 
> assim.
> > ]
> > > 
> > > O que é uma função monótona?
> > > Lista OBM wrote:Gostaria 
> > que alguém me ajudasse com o problema abaixo:
> > > 
> > > Seja f: J --> R uma função monótona, definida 
> no 
> > intervalo J. Se a 
> > > 
> > > imagem f(J) é um intervalo, prove que f é 
> contínua.
> > > 
> > > Obs.: Tentei supondo o contrário, mas não 
> consegui!!!
> > > 
> > > Grato, Éder.
> > > 
> > > > 
> > > -
> > > Yahoo! Messenger - Fale com seus amigos 
> online. 
> > Instale agora!
> > > 
> > > 
> > > 
> > > -
> > > Yahoo! Mail - Participe da pesquisa global 
> sobre o 
> > Yahoo! Mail. Clique aqui!
> > 
> > Atenciosamente,
> > 
> > Engenharia Elétrica - UNESP Ilha Solteira
> > Osvaldo Mello Sponquiado 
> > Usuário de GNU/Linux
> > 
> > 
> > 
> > 
> ___
> ___
> > Acabe com aquelas janelinhas que pulam na sua 
> tela.
> > AntiPop-up UOL - É grátis!
> > http://antipopup.uol.com.br/
> > 
> > 
> > 
> > 
> ===
> ==
> > Instruções para entrar na lista, sair da lista e 
> usar a lista em
> > http://www.mat.puc-rio.br/~nicolau/olimp/obm-
> l.html
> > 
> ===
> ==
> > 
> > 
> > 
> > 
> > -
> -
> >   Yahoo! Messenger - Fale com seus amigos online. 
> Instale agora!
> >
> 
> Atenciosamente,
> 
> Engenharia Elétrica - UNESP Ilha Solteira
> Osvaldo Mello Sponquiado 
> Usuário de GNU/Linux
> 
> __
> Acabe com aquelas janelinhas que pulam na sua tela.
> AntiPop-up UOL - É grátis!
> http://antipopup.uol.com.br/
> 
> =
> Instruções para entrar na lista, sair da lista e usar a lista em
> http://www.mat.puc-rio.br/~nicolau/olimp/obm-l.html
> =
--- End of Original Message ---

=
Instruções para entrar na lista, sair da lista e usar a lista em
http://www.mat.puc-rio.br/~nicolau/olimp/obm-l.html
=


Re: [obm-l] Três problemas

2004-06-02 Por tôpico Augusto Cesar de Oliveira Morgado



2) (p^2+3/(p+1) = p -1 + 4/(p+1)
p so pode ser 0, 1 ou 3.
Os naturais correspondentes sao 3, 2 e 3.

A resposta eh 3 e 2.

3) 1 - 2 (sen^2) x + 4 senx = p
2 (sen^2) x - 4 senx + (p-1) = 0
delta = 16 - 8(p-1) = 24 - 8p
p<3
Se o intervalo eh aberto, devemos excluir p=1 (nesse caso as soluçoes seriam 0 e pi).
== 
Mensagem  enviada  pelo  CIP  WebMAIL  - Nova Geração - v. 2.1 
CentroIn Internet Provider          http://www.centroin.com.br 
Tel: (21) 2542-4849, (21) 2295-3331        Fax: (21) 2295-2978 
Empresa 100% Brasileira - Desde 1992 prestando servicos online 


-- Original Message ---
From: João Luís <[EMAIL PROTECTED]> 
To: Lista Matemática <[EMAIL PROTECTED]> 
Sent: Mon, 31 May 2004 11:14:56 -0300 
Subject: [obm-l] Três problemas 

> Olá lista, 
> Estou enviando três problemas que eu gostaria de ver comentados por vocês da lista: 
>   
> 1) Calcule os valores de k, 0<=k<=2PI, que satisfazem a desigualdade -x^2 + 1/2 < sen (k) 
>   
> 2) Divida o polinômio p^2 + 3 por p + 1; utilizando essa divisão, ache todos os naturais da forma (p^2 + 3) / (p + 1), com "p" pertencente ao conjunto N. 
>   
> 3) A equação cos2x + 4senx = p admite duas soluções no intervalo (0, PI). Qual é a condição satisfeita por "p"? 
>   
> Essas questões são de provas de vestibulares. 
> Desde já eu agradeço a atenção de todos. 
--- End of Original Message ---






Re:[obm-l] elipse

2004-05-31 Por tôpico Augusto Cesar de Oliveira Morgado



É claro que não está certo, até porque as equações encontradas não representam retas.

== 
Mensagem  enviada  pelo  CIP  WebMAIL  - Nova Geração - v. 2.1 
CentroIn Internet Provider          http://www.centroin.com.br 
Tel: (21) 2542-4849, (21) 2295-3331        Fax: (21) 2295-2978 
Empresa 100% Brasileira - Desde 1992 prestando servicos online 


-- Original Message ---
From: Jefferson Franca <[EMAIL PROTECTED]> 
To: [EMAIL PROTECTED] 
Sent: Mon, 31 May 2004 22:21:51 -0300 (ART) 
Subject: Re:[obm-l] elipse 

> Valeu! Uma curiosidade: E sem derivada? Como ficaria?
> 
> Osvaldo <[EMAIL PROTECTED]> wrote: Posso decompor esta eq. ai em duas funçoes
> f(x)_1 = +sqrt(1-(x/2)^2)
> f(x)_2 = -sqrt(1-(x/2)^2)
> (x_0,y_o)=(3,2)
> Uma saída é utilizar y-y_0=y'.(x-x_0) (y'=d(f(x))/dx) 
> como reta tangente em (x_0,y_0)
> 
> Da primeira funçao vem que y-2=-x(x-3)/sqrt(1-(x/2)^2)
> Da segunda funçao vem que y-2=x(x-3)/sqrt(1-(x/2)^2)
> 
> Bom, não sei se ta certo, se estiver a eq. vai 
> corresponder a 1-(x/2)^2=x(x-3)/(y-2) 
> 
> falow ai
> 
> > Será q alguém poderia me ajudar com a questão: 
> Determine a equação das tangentes à elipse (x^2)/4 + 
> (y^2) = 1, que passam pelo ponto P(3,2). 
> > 
> > 
> > 4-x^2 /4 -2x
> 
> -
> > 
> > -
> > Yahoo! Mail - Participe da pesquisa global sobre o 
> Yahoo! Mail. Clique aqui!
> 
> Atenciosamente,
> 
> Engenharia Elétrica - UNESP Ilha Solteira
> Osvaldo Mello S! ponquiado 
> Usuário de GNU/Linux
> 
> __
> Acabe com aquelas janelinhas que pulam na sua tela.
> AntiPop-up UOL - É grátis!
> http://antipopup.uol.com.br/
> 
> =
> Instruções para entrar na lista, sair da lista e usar a lista em
> http://www.mat.puc-rio.br/~nicolau/olimp/obm-l.html
> =
> 
> 
Yahoo! Mail - Participe da pesquisa global sobre o Yahoo! Mail. Clique aqui! 
--- End of Original Message ---






Re: [obm-l] Problema_de_combinatória

2004-05-31 Por tôpico Augusto Cesar de Oliveira Morgado





Corrigindo uns errinhos do Dirichlet e, espero, não introduzindo outros:

As classes de congruencia mod 3 sao: 
   
 C0={0,3,6,9}
C1={1,4,7}
C2={2,5,8} 
   
 Existem , de 102 ate 996, 150 multiplos de 6.  
 Quantos deles tem algarismos repetidos?
1) aaa: 
 Essa nem precisa pensar muito... 
 222 444 666 888 
(qualquer numero de tres algarismos iguais e multiplo de 3. Como todo par multiplo de 3 e multiplo de 6, acabou!) 
   
2) aab, com b diferente de a: 
 b deve ser 0,2,4,6,8 ; a nao pode ser 0
 2a+b=0 (mod 3) 
 a=b (mod 3) 
   
 Assim b determina a (mod 3). 
    b=0 da 3 possibilidades para a 
b=2 da 2 possibilidades para a 
    b=4 da 2 possibilidades para a  
    b=6 da 2 possibilidades para a 
    b=8 da 2 possibilidades para a 
   
   O total e 11. 
   
3) aba, com b diferente de a: 
 a deve ser  2,4,6,8. 
 2a+b=0 mod 3 
 a=b mod 3 
  
a=2 da 2 possibilidades para b 
    a=4 da 2 possibilidades para b  
    a=6 da 3 possibilidades para b 
    a=8 da 2 possibilidades para b 
   
   O total e 9. 

   
4) baa, com b diferente de a: 
 a deve ser  0, 2, 4, 6, 8 ; b nao pode ser 0.
2a+b=0 mod 3 
a=b mod 3 
  
    a=0 da 3 possibilidades para b 
a=2 da 2 possibilidades para b 
    a=4 da 2 possibilidades para b  
    a=6 da 2 possibilidades para b 
    a=8 da 2 possibilidades para a 


O total e 11.

 
 Logo, fazendo as contas, temos 11+9+11+6=37 
 150-37=113. 
   
 

Fernando Villar <[EMAIL PROTECTED]> wrote: 

> Olá pessoal, é um prazer participar desta lista. 
>   
> Resolvi o problema abaixo dividindo-o em muitos casos.  
>   
> "Quantos números de 3 algarismos distintos são divisíveis por 6?" 
>   
> Peço sugestões para uma resolução mais sucinta. 
>   
> Agradeço
> 
> TRANSIRE SVVM PECTVS MVNDOQVE POTIRI 
> 
> CONGREGATI EX TOTO ORBE MATHEMATICI OB SCRIPTA INSIGNIA TRIBVERE 
> 
> Fields Medal(John Charles Fields) 
>   
> N.F.C. (Ne Fronti Crede)
> 
> 
Yahoo! Mail - Participe da pesquisa global sobre o Yahoo! Mail. Clique aqui! 
--- End of Original Message ---






Re: [obm-l] ITA 73

2004-05-30 Por tôpico Augusto Cesar de Oliveira Morgado
m/lnp + n/lmp + p/lmn = (m^2+l^2+p^2+n^2)/(lmnp)=
= [(m+n+l+p)^2-2*(mn+ml+mp+nl+np+lp)]/(lmnp) = (q^2-2r)/ t

==
Mensagem  enviada  pelo  CIP  WebMAIL  - Nova Geração - v. 2.1
CentroIn Internet Provider  http://www.centroin.com.br
Tel: (21) 2542-4849, (21) 2295-3331Fax: (21) 2295-2978
Empresa 100% Brasileira - Desde 1992 prestando servicos online


-- Original Message ---
From: <[EMAIL PROTECTED]>
To: [EMAIL PROTECTED]
Sent: 30 May 2004 20:13:13 -0300
Subject: [obm-l] ITA 73

> oi pessoal! Sou o Rafael Lima aluno do Sistema Elite de Ensino- RJ e 
> queria q vcs me ajuda-se a Fazer essa questão: Seja a equação do 4° 
> x^4+qx^3+rx^2+sx+t=0 onde q,r,s,t são números racionais não nulos 
> tais que l,m,n,p são raízes reais dessa equação. o valor de l/mnp + 
> m/lnp + n/lmp + p/lmn = ???
> 
> 
__
_
> Acesse nosso portal www.click21.com.br
> 
> Porque internet grátis, nem a Embratel pode fazer mais barato. Mas 
> pode fazer melhor.
> 
> =
> Instruções para entrar na lista, sair da lista e usar a lista em
> http://www.mat.puc-rio.br/~nicolau/olimp/obm-l.html
> =
--- End of Original Message ---

=
Instruções para entrar na lista, sair da lista e usar a lista em
http://www.mat.puc-rio.br/~nicolau/olimp/obm-l.html
=


Re: [obm-l] Geometria Espacial

2004-05-30 Por tôpico Augusto Cesar de Oliveira Morgado



Contreiras, que prova foi essa?

== 
Mensagem  enviada  pelo  CIP  WebMAIL  - Nova Geração - v. 2.1 
CentroIn Internet Provider          http://www.centroin.com.br 
Tel: (21) 2542-4849, (21) 2295-3331        Fax: (21) 2295-2978 
Empresa 100% Brasileira - Desde 1992 prestando servicos online 


 
> - Original Message - 
> From: Fabio Contreiras 
> To: [EMAIL PROTECTED] 
> Sent: Sunday, May 30, 2004 2:23 PM 
> Subject: [obm-l] Geometria Espacial 
> 
> Acabei de sair de uma prova no qual me deparei com a questão : 
>   
> A área da superfície lateral de um cone equilátero inscrito numa esfera de raio R é ? 
>   
>   
> Gostaria de uma solução plausível para que o resultado dê [ ( pi R.R sqrt(3) ) / 2 ] ! 
>   
> ps. achei [ (  pi . 3.R.R ) / 2 ] 
>   
>   
> obrigado!

--- End of Original Message ---






Re: [obm-l] Re: [obm-l] Combinatória

2004-05-30 Por tôpico Augusto Cesar de Oliveira Morgado



Posso lhe garantir (estava na banca nessa prova) que o gabarito oficial era 2.
Na prova, garantia-se que essa situação era possível.
No seu enunciado não se garante isso. Eu não provei que a resposta era 2. Eu provei que, se o problema tem solução (ou seja, se a situação proposta é possível), a resposta é 2.
Para resolver completamente o seu problema, há que provar que a situação é possível. Este é um bom problema.
Só por curiosidade, a versão inicialmente proposta para esse problema era impossível.

== 
Mensagem  enviada  pelo  CIP  WebMAIL  - Nova Geração - v. 2.1 
CentroIn Internet Provider          http://www.centroin.com.br 
Tel: (21) 2542-4849, (21) 2295-3331        Fax: (21) 2295-2978 
Empresa 100% Brasileira - Desde 1992 prestando servicos online 


-- Original Message ---
From: "Thor" <[EMAIL PROTECTED]> 
To: <[EMAIL PROTECTED]> 
Sent: Sun, 30 May 2004 11:18:39 -0300 
Subject: [obm-l] Re: [obm-l] Combinatória 

>   
> - Original Message - 
> From: Augusto Cesar de Oliveira Morgado 
> To: [EMAIL PROTECTED] 
> Sent: Sunday, May 30, 2004 10:12 AM 
> Subject: Re: [obm-l] Combinatória

> Caro , Morgado, tambem cheguei a essa resposta, essa questão foi da escola naval( se não estou enganado) 
> e ele não dá 2 como resposta, aí que está minha duvida. 
> Onde está a casca de banana desse questão? 
>   
>   Fico agradecido. 
>   
>     Cláudio thor 
>   
>    
>   
>   
> 
> Em cada time ha C(4;2) = 6 pares de atletas. Contando os pares de atletas em todos os times, inclusive as repetições, encontramos 57*6 = 342 pares. 
> Ha C(19;2) = 171 pares de atletas. Logo, cada par aparece 2 vezes. A resposta é x=2. 
> Parece que está certo e completo, não? 
> Mas não está não. Por quê? 
> 
> == 
> Mensagem  enviada  pelo  CIP  WebMAIL  - Nova Geração - v. 2.1 
> CentroIn Internet Provider          http://www.centroin.com.br 
> Tel: (21) 2542-4849, (21) 2295-3331        Fax: (21) 2295-2978 
> Empresa 100% Brasileira - Desde 1992 prestando servicos online 
> 
> -- Original Message --- 
> From: "Thor" <[EMAIL PROTECTED]> 
> To: <[EMAIL PROTECTED]> 
> Sent: Sat, 29 May 2004 19:38:48 -0300 
> Subject: [obm-l] Combinatória  
> 
> A partir de um conjunto de 19 atletas, formam 57 times  de 4 atletas cada.  
> Todos os atletas participam de um mesmo numero de times e cada par de  
> atletas fica junto no mesmo  time um mesmo numero x de vezes . O valor de x eh?  
> --- End of Original Message --- 
> 
--- End of Original Message ---






Re: [obm-l] Análise I

2004-05-30 Por tôpico Augusto Cesar de Oliveira Morgado



Os limites sao todos com x tendendo para a.
g'(a) = lim [g(x)-g(a)]/[x-a] = lim [f(x)-f(a)-(x-a)f'(a)]/(x-a)^2 = lim [f'(x)-f'(a)]/2(x-a) =lim f''(x)/2 = f''(a)/2.
Isso e o resultado a que voce chegou provam a continuidade de g' em a. A continuidade de g' nos demais pontos decorre imediatamente do fato de f ser de classe C2.
== 
Mensagem  enviada  pelo  CIP  WebMAIL  - Nova Geração - v. 2.1 
CentroIn Internet Provider          http://www.centroin.com.br 
Tel: (21) 2542-4849, (21) 2295-3331        Fax: (21) 2295-2978 
Empresa 100% Brasileira - Desde 1992 prestando servicos online 


-- Original Message ---
From: Lista OBM <[EMAIL PROTECTED]> 
To: [EMAIL PROTECTED] 
Sent: Sun, 30 May 2004 09:26:02 -0300 (ART) 
Subject: Re: [obm-l] Análise I  
 
 
 ii) Seja  f: I à R de classe C2. Dado a em I, defina g: I ! à R por g(x) = [f(x) – f(a)]/(x – a) se x ¹ a e g(a) = f´(a). Prove que g é de classe C1. Usando o pol. de Taylor com resto de Lagrange para f, cheguei que: limx®a g´(x) = [f´´(a)]/2 . Mas não estou conseguindo concluir que g é de classe C1. 





Re: [obm-l] Combinatória

2004-05-30 Por tôpico Augusto Cesar de Oliveira Morgado



Em cada time ha C(4;2) = 6 pares de atletas. Contando os pares de atletas em todos os times, inclusive as repetições, encontramos 57*6 = 342 pares.
Ha C(19;2) = 171 pares de atletas. Logo, cada par aparece 2 vezes. A resposta é x=2.
Parece que está certo e completo, não?
Mas não está não. Por quê?

== 
Mensagem  enviada  pelo  CIP  WebMAIL  - Nova Geração - v. 2.1 
CentroIn Internet Provider          http://www.centroin.com.br 
Tel: (21) 2542-4849, (21) 2295-3331        Fax: (21) 2295-2978 
Empresa 100% Brasileira - Desde 1992 prestando servicos online 


-- Original Message ---
From: "Thor" <[EMAIL PROTECTED]> 
To: <[EMAIL PROTECTED]> 
Sent: Sat, 29 May 2004 19:38:48 -0300 
Subject: [obm-l] Combinatória 

 A partir de um conjunto de 19 atletas, formam 57 times  de 4 atletas cada. 
 Todos os atletas participam de um mesmo numero de times e cada par de 
 atletas fica junto no mesmo  time um mesmo numero x de vezes . O valor de x eh?  
--- End of Original Message ---






Re: [obm-l] Análise I

2004-05-29 Por tôpico Augusto Cesar de Oliveira Morgado




Parece (os simbolos estao incompreensiveis) que se quer ptovar que o modulo de (x-a)^n / n!
tende a 0 quando n tende a infinito. Pense nisso como o termo geral de uma serie, prove pelo criterio da razao de D'Alembert que ela eh convergente (a razao a(n+1)/a(n) tende a 0) e conclua que o termo geral tende a 0.
== 

Mensagem  enviada  pelo  CIP  WebMAIL  - Nova Geração - v. 2.1 

CentroIn Internet Provider          http://www.centroin.com.br 

Tel: (21) 2542-4849, (21) 2295-3331        Fax: (21) 2295-2978 

Empresa 100% Brasileira - Desde 1992 prestando servicos online 



-- Original Message ---

From: Lista OBM <[EMAIL PROTECTED]> 

To: [EMAIL PROTECTED] 

Sent: Sat, 29 May 2004 17:05:44 -0300 (ART) 

Subject: [obm-l] Análise I 


> Gostaria de saber se alguém pode me ajudar com os "dois problemas" abaixo:

>  

> 
> i) Sendo | r(x) | £ [K|x-xo|n+1]/(n + 1)!, onde K > 0, prove que limn®¥ r(x) = 0; 

>  

> ii) Seja  f: I à R de classe C2. Dado a em I, defina g: I !
à R por g(x) = [f(x) – f(a)]/(x – a) se x ¹ a e g(a) = f´(a). Prove que g é de classe C1. Usando o pol. de Taylor com resto de Lagrange para f, cheguei que: limx®a g´(x) = [f´´(a)]/2 . Mas não estou conseguindo 

>  concluir que g é de classe C1.

> $1 

> Grato desde já com a possível ajuda de vocês.
> 
> 
Yahoo! Messenger - Fale com seus amigos online. Instale agora!

--- End of Original Message ---








Re: [obm-l] Análise I

2004-05-29 Por tôpico Augusto Cesar de Oliveira Morgado




Simbolos incompreensiveis!

== 

Mensagem  enviada  pelo  CIP  WebMAIL  - Nova Geração - v. 2.1 

CentroIn Internet Provider          http://www.centroin.com.br 

Tel: (21) 2542-4849, (21) 2295-3331        Fax: (21) 2295-2978 

Empresa 100% Brasileira - Desde 1992 prestando servicos online 



-- Original Message ---

From: Lista OBM <[EMAIL PROTECTED]> 

To: [EMAIL PROTECTED] 

Sent: Sat, 29 May 2004 17:05:44 -0300 (ART) 

Subject: [obm-l] Análise I 


> Gostaria de saber se alguém pode me ajudar com os "dois problemas" abaixo:

>  

> 
> i) Sendo | r(x) | £ [K|x-xo|n+1]/(n + 1)!, onde K > 0, prove que limn®¥ r(x) = 0; 

>  

> ii) Seja  f: I à R de classe C2. Dado a em I, defina g: I !
à R por g(x) = [f(x) – f(a)]/(x – a) se x ¹ a e g(a) = f´(a). Prove que g é de classe C1. Usando o pol. de Taylor com resto de Lagrange para f, cheguei que: limx®a g´(x) = [f´´(a)]/2 . Mas não estou conseguindo 

>  concluir que g é de classe C1.

> $1 

> Grato desde já com a possível ajuda de vocês.
> 
> 
Yahoo! Messenger - Fale com seus amigos online. Instale agora!

--- End of Original Message ---








Re: [obm-l] Re:[obm-l] Números complexos e outro

2004-05-29 Por tôpico Augusto Cesar de Oliveira Morgado
Um errinho bobo na primeira raiz: onde está pi/2 é pi/8.

==
Mensagem  enviada  pelo  CIP  WebMAIL  - Nova Geração - v. 2.1
CentroIn Internet Provider  http://www.centroin.com.br
Tel: (21) 2542-4849, (21) 2295-3331Fax: (21) 2295-2978
Empresa 100% Brasileira - Desde 1992 prestando servicos online


-- Original Message ---
From: "Osvaldo" <[EMAIL PROTECTED]>
To: "obm-l" <[EMAIL PROTECTED]>
Sent: Sat, 29 May 2004 19:35:33 -0300
Subject: [obm-l] Re:[obm-l] Números complexos e outro

> 2° ex.
> 
> Usando a def. de exponencial complexa, mesmo assim 
> temos:
> 
> z=e^Pi*i/2 =e^(0+i.pi/2)=e^0.(cos(pi/2)+i.sen(pi/2))
> 
> Assim as raízes quartas de z são da forma:
> 
> z_k=1^4.{cos[(pi/2+2kpi)/4]+i.sen[(pi/2+2kpi)/4] para 
> k=0,1,2,3.
> 
> Assim as raizes são:
> 
> z_1=1.(cos(pi/2)+i.sen(pi/2))
> z_2=1.(cos(5pi/8)+i.sen(5pi/8))
> z_3=1.(cos(9pi/8)+i.sen(9pi/8))
> z_4=1.(cos(13pi/8)+i.sen(13pi/8))
> 
> Axo que deva ser isto. A definição de exp.complexa é 
> fundamental para o estudo de números complexos (no 
> ensino médio não creio que seja dada, eu vi semestre 
> passado.) A definição é a seguinte: e^(x+i.y)=
> e^x.(cosy+i.seny). Para encontrar e^i faça a expansão 
> da exponencial supondo que esta satisfaça as prop. do 
> corpo dos reais, desta maneira separe os termos de 
> ordem par dos de ordem impar.
> 
> falow ai
> 
> > Olá
> > 
> > Eis alguns exercícios :
> > 
> > 1 ] Sejam a e b dois números naturais com b # 0 . Se 
> r é oresto da divisão 
> > de a por b então o resto da divisão de a^n por b é 
> igual ao resto da divisão 
> > de r ^n por b . Utilizando este teorema , calcular o 
> resto da divisão de 
> > [5342177]^8 por 9.
> > 
> > 2 ]  ITA - As raízes de ordem 4 do número 
> z=e^Pi*i/2  , onde i é a unidade 
> > imaginária , são [na forma trigonométrica] ?
> > 
> > 3 ]  ITA - Seja S o conjunto dos números complexos 
> que satisfazem, 
> > simultaneamente às equções
> > | z - 3 i | = 3 e | z + i | = | z - 2 - i |
> > O produto de todos os elememtos de S é iguaL a ?
> > 
> > 
> ___
> __
> > MSN Messenger: converse com os seus amigos online.  
> > http://messenger.msn.com.br
> > 
> > 
> ===
> ==
> > Instruções para entrar na lista, sair da lista e 
> usar a lista em
> > http://www.mat.puc-rio.br/~nicolau/olimp/obm-l.html
> > 
> ===
> ==
> >
> 
> Atenciosamente,
> 
> Engenharia Elétrica - UNESP Ilha Solteira
> Osvaldo Mello Sponquiado 
> Usuário de GNU/Linux
> 
> __
> Acabe com aquelas janelinhas que pulam na sua tela.
> AntiPop-up UOL - É grátis!
> http://antipopup.uol.com.br/
> 
> =
> Instruções para entrar na lista, sair da lista e usar a lista em
> http://www.mat.puc-rio.br/~nicolau/olimp/obm-l.html
> =
--- End of Original Message ---

=
Instruções para entrar na lista, sair da lista e usar a lista em
http://www.mat.puc-rio.br/~nicolau/olimp/obm-l.html
=


Re: [obm-l] Problema

2004-05-29 Por tôpico Augusto Cesar de Oliveira Morgado



Claro, que burrice a minha! A soma é preservada, mas a multiplicação não!
Desculpe.
Morgado

== 
Mensagem  enviada  pelo  CIP  WebMAIL  - Nova Geração - v. 2.1 
CentroIn Internet Provider          http://www.centroin.com.br 
Tel: (21) 2542-4849, (21) 2295-3331        Fax: (21) 2295-2978 
Empresa 100% Brasileira - Desde 1992 prestando servicos online 


-- Original Message ---
From: Lista OBM <[EMAIL PROTECTED]> 
To: [EMAIL PROTECTED] 
Sent: Sat, 29 May 2004 08:54:55 -0300 (ART) 
Subject: Re: [obm-l] Problema 

> Meu caro Morgado, 
> não sei se estou equivocado, mas a aplicação que você definiu não é um homomorfismo, pois: f(x.y) = -(x.y) = -x.y e f(x).f(y) =(-x).(-y) = x.y, ou seja, f(x.y) é diferente de f(x).f(y). Além do mais, num homomorfismo f entre domínios de integridade sempre temos que: ou f leva o elemento identidade (em relção a multiplicação) do domínio no elemento identidade do contadomínio ou f é a função constante zero. De fato, f(0) = f(0+0) = f(0)+f(0) =>f(0) = 0 e f(1) = f(1.1) = f(1).f(1) => f(1)[1 - f(1)] = 0 => 1 = f(1) ou f(1) = 0. Se f(1) = 0 então segue que f(x) = f(x.1) = f(x).f(1) = f(x).0 = 0, para todo x do domínio, ou seja, f é a função constate zero. Assim, nunca pode ocorrer f(1) = -1 num homomorfismo entre corpos.   
> 
> Augusto Cesar de Oliveira Morgado <[EMAIL PROTECTED]> wrote: Isso é falso! Tome K=Q e defina f por f(x)=-x. 
> 1 é positivo e f(1) não é. 
> 
> == 
> Mensagem  enviada  pelo  CIP  WebMAIL  - Nova Geração - v. 2.1 
> CentroIn Internet Provider          http://www.centroin.com.br 
> Tel: (21) 2542-4849, (21) 2295-3331        Fax: (21) 2295-2978 
> Empresa 100% Brasileira - Desde 1992 prestando servicos online 
> 
> -- Original Message --- 
> From: Lista OBM <[EMAIL PROTECTED]> 
> To: [EMAIL PROTECTED] 
> Sent: Fri, 28 May 2004 11:41:34 -0300 (ART) 
> Subject: [obm-l] Problema 
> 
> > Gostaria de saber se alguém poderia me ajudar com o seguinte problema: 
> > Sejam A e B anéis ordenados. Diz-se que um homomorfismo injetivo f: A --> B preserva ordem se, para todo a > 0 em A, tivermos f(a) > 0. Sejam K um corpo ordenado e f: Q --> K um homomorfismo injetivo dos números racionais em K. Mostre que, necessariamente, f preserva a ordem. 
> >   
> > Grato desde já com a possível ajuda de vocês. 
> > 
> > 
Yahoo! Messenger - Fale com seus amigos online. Instale agora! 
> --- End of Original Message --- 
> 
> 
> 
Yahoo! Messenger - Fale com seus amigos online. Instale agora! 
--- End of Original Message ---






Re: [obm-l] Problema

2004-05-29 Por tôpico Augusto Cesar de Oliveira Morgado



Isso é falso! Tome K=Q e defina f por f(x)=-x. 
1 é positivo e f(1) não é.

== 
Mensagem  enviada  pelo  CIP  WebMAIL  - Nova Geração - v. 2.1 
CentroIn Internet Provider          http://www.centroin.com.br 
Tel: (21) 2542-4849, (21) 2295-3331        Fax: (21) 2295-2978 
Empresa 100% Brasileira - Desde 1992 prestando servicos online 


-- Original Message ---
From: Lista OBM <[EMAIL PROTECTED]> 
To: [EMAIL PROTECTED] 
Sent: Fri, 28 May 2004 11:41:34 -0300 (ART) 
Subject: [obm-l] Problema 

> Gostaria de saber se alguém poderia me ajudar com o seguinte problema: 
> Sejam A e B anéis ordenados. Diz-se que um homomorfismo injetivo f: A --> B preserva ordem se, para todo a > 0 em A, tivermos f(a) > 0. Sejam K um corpo ordenado e f: Q --> K um homomorfismo injetivo dos números racionais em K. Mostre que, necessariamente, f preserva a ordem. 
>   
> Grato desde já com a possível ajuda de vocês.
> 
> 
Yahoo! Messenger - Fale com seus amigos online. Instale agora! 
--- End of Original Message ---






Re: [obm-l] Resultado da OIMU

2004-05-26 Por tôpico Augusto Cesar de Oliveira Morgado
Parabéns a todos.
S. José dos Campos não é a cidade de origem deles, talvez com uma exceção 
(não sei qual é a cidade de origem do Estillac). Estão cursando o ITA.

==
Mensagem  enviada  pelo  CIP  WebMAIL  - Nova Geração - v. 2.1
CentroIn Internet Provider  http://www.centroin.com.br
Tel: (21) 2542-4849, (21) 2295-3331Fax: (21) 2295-2978
Empresa 100% Brasileira - Desde 1992 prestando servicos online


-- Original Message ---
From: Sergio Lima Netto <[EMAIL PROTECTED]>
To: [EMAIL PROTECTED]
Sent: Wed, 26 May 2004 11:09:14 -0300 (BRT)
Subject: Re: [obm-l] Resultado da OIMU

> De onde vem toda esta forca
> de Sao Jose dos Campos?
> Que bacana. Parabens a todos
> (organizadores, participantes e premiados)
> Abraco,
> sergio
> 
> On Wed, 26 May 2004, Olimpiada Brasileira de Matematica wrote:
> 
> > Caros(as) amigos(as) da lista:
> > 
> > Finalmente publicamos o resultado da VI OIMU
> > no site da OBM.
> > Confiram também as provas, soluções e resultados
> > internacionais.
> > 
> > Abraços, Nelly.
> > 
> > 
> > Resultado Brasileiro:
> > 
> > Humberto Silva Naves Ouro (S.J. dos Campos - SP)
> > Marcio Afonso Assad Cohen Prata (Rio de Janeiro - RJ)
> > Carlos Stein Naves de Brito Prata (S.J. dos Campos - SP)
> > Bernardo Freitas Paulo da Costa Bronze (Rio de Janeiro - RJ)
> > Yuri Gomes Lima Bronze (Fortaleza - CE)
> > Marcos Francisco Ferreira Martinelli Bronze (Rio de Janeiro - RJ)
> > Daniele Veras de Andrade Bronze (Rio de Janeiro - RJ)
> > Einstein dos Nascimento Júnior Menção (Fortaleza - CE)
> > Estillac Lins Maciel Borges Filho Menção (S.J. dos Campos - SP)
> > Thiago da Silva Sobral Menção (S.J. dos Campos - SP)
> > 
> >
> 
> =
> Instruções para entrar na lista, sair da lista e usar a lista em
> http://www.mat.puc-rio.br/~nicolau/olimp/obm-l.html
> =
--- End of Original Message ---

=
Instruções para entrar na lista, sair da lista e usar a lista em
http://www.mat.puc-rio.br/~nicolau/olimp/obm-l.html
=


Re: [obm-l] Duvidas

2004-05-26 Por tôpico Augusto Cesar de Oliveira Morgado
Talvez já tenham respondido. 
O ponto médio das raízes é -b/2. Logo, -2<-b/2<3. Daí, -6http://www.centroin.com.br
Tel: (21) 2542-4849, (21) 2295-3331Fax: (21) 2295-2978
Empresa 100% Brasileira - Desde 1992 prestando servicos online


-- Original Message ---
From: "aryqueirozq" <[EMAIL PROTECTED]>
To: "[EMAIL PROTECTED]" <[EMAIL PROTECTED]>
Sent: Fri, 14 May 2004 14:44:54 -0300
Subject: [obm-l] Duvidas

> Aguém poderia me ajudar nesta questao:
> 
>A funcao f( x) = x^2 + bx + c , com  b e c reais, 
> tem duas raizes distintas pertencente ao intervalo
> [- 2 , 3].Entao , sobre os valores de b e c , a única 
> afirmativa correta eh
> a)c>9
> b)-6 c)b<-6
> d)4 e)c<-6
> 
> __
> Acabe com aquelas janelinhas que pulam na sua tela.
> AntiPop-up UOL - É grátis!
> http://antipopup.uol.com.br/
> 
> =
> Instruções para entrar na lista, sair da lista e usar a lista em
> http://www.mat.puc-rio.br/~nicolau/olimp/obm-l.html
> =
--- End of Original Message ---

=
Instruções para entrar na lista, sair da lista e usar a lista em
http://www.mat.puc-rio.br/~nicolau/olimp/obm-l.html
=


RE: [obm-l] determinantes

2004-05-25 Por tôpico Augusto Cesar de Oliveira Morgado




Quando eu estava no Científico (hoje ensino médio) os livros didáticos definiam determinante como o Hugo. Parece-me a maneira melhor de faze-lo para um aluno de ensino medio. Na verdade, os livros davam uma simplificadazinha que tornava incompreensivel o porque de o determinante de uma matriz ser igual ao da sua transposta, mas, fora isso, enunciavam-se e provavam-se todos os teoremas relevantes, inclusive o de Laplace, de modo compreensível para um aluno do segundo científico. Sofria-se no inicio, com uma semana inteira dedicada a inversões antes de chegar a definição de determinante, porem, posteriormente, o desenvolvimento da teoria era rapido e justificado (provava-se tudo).
Hoje, com o ensino "pratico e objetivo", livre dos professores que so querem complicar (provar para que? todo mundo ja sabe que é verdade mesmo!), define-se determinante pelo teorema de Laplace; isso torna complicado provar os teoremas, mas, nesses tempos praticos e objetivos, ninguem vai provar nada mesmo, a Matematica que se ensina nao tem mais teoremas, tem apenas observações...
Na época, início da década de 60, tentando resolver o que era, para mim, um mistério (o do determinante da transposta) descobri um livro que expunha de modo claro e compreensivel a teoria dos determinantes: Lições de Algebra e Analise, de Bento de Jesus Caraça. Nele se encontra a mais didatica exposição da teoria dos determinantes, provando-se tudo.

== 
Mensagem  enviada  pelo  CIP  WebMAIL  - Nova Geração - v. 2.1 
CentroIn Internet Provider          http://www.centroin.com.br 
Tel: (21) 2542-4849, (21) 2295-3331        Fax: (21) 2295-2978 
Empresa 100% Brasileira - Desde 1992 prestando servicos online 


 Em uma mensagem de 25/5/2004 00:29:48 Hora padrão leste da Am. Sul, [EMAIL PROTECTED] escreveu: 
 olá, gostaria de saber se existe uma definição exata de determinante de uma matriz... 
 
 é que eu já vi 3 definições distintas e gostaria de saber se todas sao aceitas como definições mesmo, ou apenas uma delas é a certa e as outras sao teoremas a partir dessa, ou é ainda uma outra além dessa 3... 
 
 uma das definições, dada pelo Manoel Paiva, vol 2 é: 
 "O determinante de uma matriz quadrada A = (a_ij)_(nXn), com n >= 2, é igual ao produto dos elementos da diagonal principal de qualquer matriz triangular B, equiparável a A." 
 
 bom, nesse caso eu gostaria de saber se existe algum lugar em que eu posso encontra a demonstração desses dois teoremas: 
 
 "Dada uma matriz quadrada A = (a_ij)_(nXn), existe uma matriz triangular B = (b_ij)_(nXn) equiparável a A." 
 esse eu acho meio intuitivo, mas tentei provar matematicamente e não consegui... 
 
 "Se duas matrizes triangulares A e B são equiparáveis, então ambas possuem o mesmo produto dos elementos da diagonal principal." 
 esse nao é nem um pouco intuitivo e tb nao consegui demonstrar. 
 
 bom, a outra definição que encontrei para determinante foi no Gelson Iezzi vol. 4.: 
 "O determinante de uma matriz de ordem n >= 2 é a soma dos produtos dos elementos da primeira coluna pelos respectivos cofatores." 
 
 a outra definição que encontrei foi em um e-mail enviado para esta lista, por Hugo Iver Vasconcelos Gonçalves: 
 "o determinante de uma matriz é a soma algébrica de todos os possíveis fatores em que estão presentes um (e apenas um) elemento de cada linha e cada coluna,  sendo que aqueles em que os índices dos elementos da matriz formam uma permutação de primeira classe são tomados positivamente e os demais, negativamente." 
 nesse caso a explicação que ele deu para permutação de primeira classe foi: 
 "permutação de primeira classe é aquela em que o número de inversões é par" 
e a explicação para inversões foi: 
 "inversão é o fato de um par de elementos de uma permutação não aparecer na mesma ordem que apareceram na permutação inicial.  No caso de a permutação inicial de n números ser a disposição deste em ordem crescente, uma inversão seria basicamente o fato de aparecer um número maior antes de um menor. E se a ordem inicial deles for outra, pode-se sempre chamar o 1o elemento de a1 e o n-ésimo de an, de modo que uma inversão será simplesmente quando aparecer um número ap antes de um aq, tais que p > q." 
 
 nesse caso eu nao entendi como calcular quantas inversoes foram necessarias para chegar a dada permutação... 
 
 bom, é isso, sanadas minha dúvidas e se não for abuso, gostaria de saber onde poderia encontrar a demonstração do teorema fundamental de Laplace. 
 
 desde já agradeço 
   
--- End of Original Message ---







Re: [obm-l] determinantes

2004-05-25 Por tôpico Augusto Cesar de Oliveira Morgado
So uma curiosidade: parece que determinantes sao assunto de ensino medio 
apenas no Brasil e em Portugal.
Morgado

> Fael escreveu:
> > Pegando um gancho:
> > 
> > De todos os conceitos matematicos de Ensino Medio, 
> os unicos que ate agora eu 
> > nao vejo contextualizacao sao *determinantes* e 
> *numeros complexos*. Sei que 
> > ambos estao presentes na historia da criacao dos 
> computadores, por exemplo, 
> > mas nao consigo imaginar uma situacao-problema em 
> que seja necessario utilizar 
> > estes 2 conceitos. Todos os outros conceitos de 
> matematica de Ensino Medio sao 
> > facilmente contextualizados, mas estes 2 sao um 
> *estranho no ninho* da 
> > matematica de Ensino Medio. E para piorar, muitos 
> livros definem *determinante* como 
> > um numero associado a uma matriz (Grande definicao ! 
> Ironicamente falando :-)
> > 
> > 
> > 
> > Em uma mensagem de 25/5/2004 00:29:48 Hora padrão 
> leste da Am. Sul, 
> > [EMAIL PROTECTED] escreveu:
> > 
> > 
> > > 
> > > 
> > > olá, gostaria de saber se existe uma definição 
> exata de determinante de uma 
> > > matriz... 
> > > 
> > > é que eu já vi 3 definições distintas e gostaria 
> de saber se todas sao 
> > > aceitas como definições mesmo, ou apenas uma delas 
> é a certa e as outras sao 
> > > teoremas a partir dessa, ou é ainda uma outra além 
> dessa 3...
> > > 
> > > uma das definições, dada pelo Manoel Paiva, vol 2 
> é:
> > > "O determinante de uma matriz quadrada A = (a_ij)_
> (nXn), com n >= 2, é igual 
> > > ao produto dos elementos da diagonal principal de 
> qualquer matriz triangular 
> > > B, equiparável a A."
> > > 
> > > bom, nesse caso eu gostaria de saber se existe 
> algum lugar em que eu posso 
> > > encontra a demonstração desses dois teoremas:
> > > 
> > > "Dada uma matriz quadrada A = (a_ij)_(nXn), existe 
> uma matriz triangular B = 
> > > (b_ij)_(nXn) equiparável a A."
> > > esse eu acho meio intuitivo, mas tentei provar 
> matematicamente e não 
> > > consegui...
> > > 
> > > 
> > > "Se duas matrizes triangulares A e B são 
> equiparáveis, então ambas possuem o 
> > > mesmo produto dos elementos da diagonal principal."
> > > esse nao é nem um pouco intuitivo e tb nao 
> consegui demonstrar.
> > > 
> > > bom, a outra definição que encontrei para 
> determinante foi no Gelson Iezzi 
> > > vol. 4.:
> > > "O determinante de uma matriz de ordem n >= 2 é a 
> soma dos produtos dos 
> > > elementos da primeira coluna pelos respectivos 
> cofatores."
> > > 
> > > a outra definição que encontrei foi em um e-mail 
> enviado para esta lista, 
> > > por Hugo Iver Vasconcelos Gonçalves:
> > > "o determinante de uma matriz é a soma algébrica 
> de todos os possíveis 
> > > fatores em que estão presentes um (e apenas um) 
> elemento de cada linha e cada 
> > > coluna,  sendo que aqueles em que os índices dos 
> elementos da matriz formam uma 
> > > permutação de primeira classe são tomados 
> positivamente e os demais, 
> > > negativamente."
> > > nesse caso a explicação que ele deu para 
> permutação de primeira classe foi:
> > > "permutação de primeira classe é aquela em que o 
> número de inversões é par"
> > > e a explicação para inversões foi:
> > > "inversão é o fato de um par de elementos de uma 
> permutação não aparecer na 
> > > mesma ordem que apareceram na permutação inicial.  
> No caso de a permutação 
> > > inicial de n números ser a disposição deste em 
> ordem crescente, uma inversão 
> > > seria basicamente o fato de aparecer um número 
> maior antes de um menor. E se a 
> > > ordem inicial deles for outra, pode-se sempre 
> chamar o 1o elemento de a1 e o 
> > > n-ésimo de an, de modo que uma inversão será 
> simplesmente quando aparecer um 
> > > número ap antes de um aq, tais que p > q."
> > > 
> > > nesse caso eu nao entendi como calcular quantas 
> inversoes foram necessarias 
> > > para chegar a dada permutação...
> > > 
> > > 
> > > bom, é isso, sanadas minha dúvidas e se não for 
> abuso, gostaria de saber 
> > > onde poderia encontrar a demonstração do teorema 
> fundamental de Laplace.
> > > 
> > > desde já agradeço
> > > 
> > 
> > 
> >
> 
> Atenciosamente,
> 
> Engenharia Elétrica - UNESP Ilha Solteira
> Osvaldo Mello Sponquiado 
> Usuário de GNU/Linux
> 
> __
> Acabe com aquelas janelinhas que pulam na sua tela.
> AntiPop-up UOL - É grátis!
> http://antipopup.uol.com.br/
> 
> =
> Instruções para entrar na lista, sair da lista e usar a lista em
> http://www.mat.puc-rio.br/~nicolau/olimp/obm-l.html
> =
--- End of Original Message ---

=
Instruções para entrar na lista, sair da lista e usar a lista em
http://www.mat.puc-rio.br/~nicolau/olimp/obm-l.html
=


[obm-l] Re: N/A

2004-05-23 Por tôpico Augusto Cesar de Oliveira Morgado
Os restos das divisões dessas potências por 7620 repetem-se em ciclos de seis:
19, 361, 6859, 781, 7219, 1.
Como a soma desses seis números é o dobro de 7620, cada bloco de seis 
parcelas consecutivas é múltiplo de 7620.
Como 2001= 6*333+3, sua soma é formada por 333 blocos de multiplos de 7620 e 
mais tres parcelas.
Ou seja, a resposta do seu problema é a mesma que seria se a soma fosse apenas
19+19^2+19^3.
7239 é a resposta.
==
Mensagem  enviada  pelo  CIP  WebMAIL  - Nova Geração - v. 2.1
CentroIn Internet Provider  http://www.centroin.com.br
Tel: (21) 2542-4849, (21) 2295-3331Fax: (21) 2295-2978
Empresa 100% Brasileira - Desde 1992 prestando servicos online


-- Original Message ---
From: "biper" <[EMAIL PROTECTED]>
To: "obm-l" <[EMAIL PROTECTED]>
Sent: Sat, 22 May 2004 14:53:32 -0300
Subject: N/A

> E aí pessoal bleza?
> Alguem pode me ajudar nessa aqui:
> 
> Se n é a soma de todas as potencias de 19 até 19^2001
> 
>n = 19 + 19^2 + 19^3 + 19^4 ... + 19^2001
> 
> qual o valor do resto divisão de n por 7620?
> 
> Um agrande abrço a todos 
> Felipe Santana
> 
> __
> Acabe com aquelas janelinhas que pulam na sua tela.
> AntiPop-up UOL - É grátis!
> http://antipopup.uol.com.br/
> 
> =
> Instruções para entrar na lista, sair da lista e usar a lista em
> http://www.mat.puc-rio.br/~nicolau/olimp/obm-l.html
> =
--- End of Original Message ---

=
Instruções para entrar na lista, sair da lista e usar a lista em
http://www.mat.puc-rio.br/~nicolau/olimp/obm-l.html
=


Re: [obm-l] Mais duvidas?

2004-05-22 Por tôpico Augusto Cesar de Oliveira Morgado
Se as raizes sao -3 e 1, f(x)= a(x+3)(x-1). A abscissa do vértice está no 
ponto medio das raízes, x do vertice = -1.
f(-1)= a.2.(-2) = -4a = 8, logo a = -2.
f(x)= -2(x+3)(x-1). 
A

==
Mensagem  enviada  pelo  CIP  WebMAIL  - Nova Geração - v. 2.1
CentroIn Internet Provider  http://www.centroin.com.br
Tel: (21) 2542-4849, (21) 2295-3331Fax: (21) 2295-2978
Empresa 100% Brasileira - Desde 1992 prestando servicos online


-- Original Message ---
From: "aryqueirozq" <[EMAIL PROTECTED]>
To: "[EMAIL PROTECTED]" <[EMAIL PROTECTED]>
Sent: Fri, 21 May 2004 19:20:31 -0300
Subject: [obm-l] Mais duvidas?

> A função f(x) do segundo grau tem raízes -3 e 1. A 
> ordenada do vértice da parábola, gráfico de f(x), é 
> igual a 8.
> A única afirmativa VERDADEIRA sobre f(x) é
> a) f(x) = -2(x-1)(x+3)
> b) f(x) = -(x-1)(x+3)
> c) f(x) = -2(x+1)(x-3)
> d) f(x) = (x-1)(x+3)
> e) f(x) = 2(x+1)(x-3)
> 
>  Agradeço desde de já.
> 
> __
> Acabe com aquelas janelinhas que pulam na sua tela.
> AntiPop-up UOL - É grátis!
> http://antipopup.uol.com.br/
> 
> =
> Instruções para entrar na lista, sair da lista e usar a lista em
> http://www.mat.puc-rio.br/~nicolau/olimp/obm-l.html
> =
--- End of Original Message ---

=
Instruções para entrar na lista, sair da lista e usar a lista em
http://www.mat.puc-rio.br/~nicolau/olimp/obm-l.html
=


Re: [obm-l] Re:[obm-l] + funcoes( correçao)

2004-05-22 Por tôpico Augusto Cesar de Oliveira Morgado
f(x) = [- x^2 + 2x]- [4x-8] = - x^2 - 2x + 8
Trata-se de uma função quadrática cujo gráfico tem a concavidade voltada para 
baixo (a<0). O máximo ocorre no ponto médio das raízes, x = -b/2a = -1
==
Mensagem  enviada  pelo  CIP  WebMAIL  - Nova Geração - v. 2.1
CentroIn Internet Provider  http://www.centroin.com.br
Tel: (21) 2542-4849, (21) 2295-3331Fax: (21) 2295-2978
Empresa 100% Brasileira - Desde 1992 prestando servicos online

-- Original Message ---
From: "aryqueirozq" <[EMAIL PROTECTED]>
To: "obm-l" <[EMAIL PROTECTED]>
Sent: Fri, 21 May 2004 19:44:03 -0300
Subject: [obm-l] Re:[obm-l] + funcoes( correçao)

 Sejam uma reta de equação y - 4x + 8 =0 e uma função 
 quadrática  g(x) = - x^2 + 2x. 
 A reta  intercepta a parábola nos pontos (-4, -24) e 
 (2, 0).
 Seja f(x) a diferença entre as ordenadas de pontos de 
 mesma abscissas x, nesta ordem: um sobre a parábola e 
 o outro sobre a reta r.
 Determine x para que f(x) seja a maior possível.


=
Instruções para entrar na lista, sair da lista e usar a lista em
http://www.mat.puc-rio.br/~nicolau/olimp/obm-l.html
=


Re: [obm-l] Funcão Quadratica(Máximos)

2004-05-22 Por tôpico Augusto Cesar de Oliveira Morgado
O lucro total mensal eh igual ao lucro na venda de uma unidade (x-10) 
multiplicado pelo numero de unidades vendidas (70-x). Portanto,
L(x) = (x-10)(70-x).
Trata-se de uma função quadrática com raízes 10 e 70 e com a concavidade 
voltada para baixo (a<0). O maximo ocorre no ponto medio das raizes, 40.
A resposta eh L(40) = 30*30 = 900
C
==
Mensagem  enviada  pelo  CIP  WebMAIL  - Nova Geração - v. 2.1
CentroIn Internet Provider  http://www.centroin.com.br
Tel: (21) 2542-4849, (21) 2295-3331Fax: (21) 2295-2978
Empresa 100% Brasileira - Desde 1992 prestando servicos online


-- Original Message ---
From: "aryqueirozq" <[EMAIL PROTECTED]>
To: "[EMAIL PROTECTED]" <[EMAIL PROTECTED]>
Sent: Fri, 21 May 2004 18:55:07 -0300
Subject: [obm-l] Funcão Quadratica(Máximos)

  Usando uma unidade monetária conveniente, o lucro obtido 
  com a venda de uma unidade de certo produto é x-10, 
  sendo x o preço de venda e 10 o preço de custo. A 
  quantidade vendida, a cada mês, depende do preço de 
  venda e é, aproximadamente, igual a 70-x.
  Nas condições dadas, o lucro mensal obtido com a venda 
  do produto é, aproximadamente, uma função quadrática de 
  x, cujo valor máximo, na unidade monetária usada, é
 a) 1200 b) 1000   c) 900d) 800 e) 600

--- End of Original Message ---

=
Instruções para entrar na lista, sair da lista e usar a lista em
http://www.mat.puc-rio.br/~nicolau/olimp/obm-l.html
=


Re: [obm-l] Somatório

2004-05-19 Por tôpico Augusto Cesar de Oliveira Morgado
Somatorio y^x, com x variando de 0 a infinito = 1/(1-y). Imagine isso como 
funçao de y e derive.
Somatorio x* [y^(x-1)], com x variando de 0 a infinito = 1/[(1-y)^2]. 
Multiplique por y.
Somatorio x* (y^x), com x variando de 0 a infinito = y/[(1-y)^2]. 
Faça y = 1-p.
Somatorio x* [(1-p)^x], com x variando de 0 a infinito = (1-p)/(p^2). 
Multiplique por p.
Somatorio px* [(1-p)^x], com x variando de 0 a infinito = (1-p)/p. 
==
Mensagem  enviada  pelo  CIP  WebMAIL  - Nova Geração - v. 2.1
CentroIn Internet Provider  http://www.centroin.com.br
Tel: (21) 2542-4849, (21) 2295-3331Fax: (21) 2295-2978
Empresa 100% Brasileira - Desde 1992 prestando servicos online


-- Original Message ---
From: "Henrique Patrício Sant'Anna Branco" <[EMAIL PROTECTED]>
To: <[EMAIL PROTECTED]>
Sent: Thu, 20 May 2004 00:56:56 -0300
Subject: [obm-l] Somatório

> Pessoal,
> 
> Alguém sabe resolver isso ou dar alguma indicação? É uma esperança 
> de uma
> v.a. geométrica.
> 
> Somatório de x*p*(1-p)^x, com x variando entre 0 e infinito.
> 
> Grato,
> Henrique.
> 
> =
> Instruções para entrar na lista, sair da lista e usar a lista em
> http://www.mat.puc-rio.br/~nicolau/olimp/obm-l.html
> =
--- End of Original Message ---

=
Instruções para entrar na lista, sair da lista e usar a lista em
http://www.mat.puc-rio.br/~nicolau/olimp/obm-l.html
=


RE: [obm-l] Exponencial

2004-05-19 Por tôpico Augusto Cesar de Oliveira Morgado
Rogério, o que voce eh? egiptologo?
Parabens por ter decifrado.

==
Mensagem  enviada  pelo  CIP  WebMAIL  - Nova Geração - v. 2.1
CentroIn Internet Provider  http://www.centroin.com.br
Tel: (21) 2542-4849, (21) 2295-3331Fax: (21) 2295-2978
Empresa 100% Brasileira - Desde 1992 prestando servicos online


-- Original Message ---
From: Rogério Moraes de Carvalho <[EMAIL PROTECTED]>
To: <[EMAIL PROTECTED]>
Sent: Wed, 19 May 2004 01:54:32 -0300
Subject: RE: [obm-l] Exponencial

> Olá Fábio,
> 
>   Ficou muito difícil entender a questão com esta explicação da
> notação no meio do enunciado. De qualquer modo, eu já havia 
> resolvido esta questão anteriormente.
> 
>   Segue o enunciado e uma resolução possível.
> 
> ENUNCIADO:
> Resolva no campo dos reais a seguinte equação exponencial:
> 3^(x^2 + 1/x^2) = 81/3^(x + 1/x)
> 
> RESOLUÇÃO:
> Condição de existência: x != 0
> 
> Fazendo y = x + 1/x, teremos:
> y^2 = (x + 1/x)^2 => y^2 = x^2 + 2 + 1/x^2 => x^2 + 1/x^2 = y^2 - 2
> 
> Portanto, representando a equação exponencial em função de y,
>  teremos: 3^(y^2 - 2) = 3^4/3^y <=> 3^(y^2 - 2) = 3^(4 - y) <=> y^2 -
>  2 = 4 - y <=> y^2 + y - 6 = 0 <=> y = -3 ou y = 2
> 
> Para y = -3:
> x + 1/x = -3 <=> x^2 + 3x + 1 = 0 <=> x = [-3-sqr(5)]/2 ou x = [-
> 3+sqr(5)]/2
> 
> Para y = 2:
> x + 1/x = 2 <=> x^2 - 2x + 1 = 0 <=> (x - 1)^2 = 0 <=> x = 1
> 
> Todas as soluções satisfazem a condição de existência.
> 
> Resposta: S = {[-3-sqr(5)]/2, [-3+sqr(5)]/2, 1}
> 
> Atenciosamente,
> 
> Rogério Moraes de Carvalho
> 
> From: [EMAIL PROTECTED] [mailto:[EMAIL PROTECTED] 
> On Behalf Of Fabio Contreiras Sent: sexta-feira, 14 de maio de 2004 23:00
> To: [EMAIL PROTECTED]
> Subject: [obm-l] Exponencial
> 
> Tentei sair dessa equação mas naum deu em nada... alguem tem o bizu 
> aih hehe , Abraços! Fabio     3^x^2 ( 3 elevado à x ao quadrado ) + 
> 1 / x^2 = { 81 / 3^[(x+1/x)] }       Valeu desde já!
> 
> =
> Instruções para entrar na lista, sair da lista e usar a lista em
> http://www.mat.puc-rio.br/~nicolau/olimp/obm-l.html
> =
--- End of Original Message ---

=
Instruções para entrar na lista, sair da lista e usar a lista em
http://www.mat.puc-rio.br/~nicolau/olimp/obm-l.html
=


Re: [obm-l] Geometria de Beltrami-Klein

2004-05-18 Por tôpico Augusto Cesar de Oliveira Morgado



www.cut-the-knot.org/triangle/pythpar/Model.shtml 

== 
Mensagem  enviada  pelo  CIP  WebMAIL  - Nova Geração - v. 2.1 
CentroIn Internet Provider          http://www.centroin.com.br 
Tel: (21) 2542-4849, (21) 2295-3331        Fax: (21) 2295-2978 
Empresa 100% Brasileira - Desde 1992 prestando servicos online 


-- Original Message ---
From: Alan Pellejero <[EMAIL PROTECTED]> 
To: [EMAIL PROTECTED] 
Sent: Tue, 18 May 2004 12:44:00 -0300 (ART) 
Subject: Re: [obm-l] Geometria de Beltrami-Klein 

> Olá, não possuo os artigos, porém me interessei pelos tópicos que 
> vai abordar. Gostaria de saber se você poderia me mandar algo a 
> respeito para que eu possa ficar a par. 
> Você estuda onde? Esta apresentação será para a pós-graduação? 
> Muito obrigado, 
> Alan Pellejero 
>   
> Ps: de uma olhada em http://www.mat.uc.pt/~emsa/Logica/Conjuntos.pdf 
> há algumas coisas interessantes lá. 
>   
> []'s
> 
> Thiago Ferraiol <[EMAIL PROTECTED]> wrote: 
> Pessoal, 
>   
> Preciso preparar um seminário sobre geometrias não Euclidianas... Nada muito profundo, só para o pessoal conhecer um pouco... Vou falar um pouco sobre o modelo de Poincare e sobre o modelo de Beltrami-Klein... 
>   
> Alguem sabe onde posso encontrar algo sobre esse último modelo??? Livros, artigos etc? 
>   
> Obrigado
> 
> 
Yahoo! Messenger - Fale com seus amigos online. Instale agora! 
--- End of Original Message ---






Re: [obm-l] Livro de eq Diferenciais...

2004-05-18 Por tôpico Augusto Cesar de Oliveira Morgado



Boyce - Di Prima

== 
Mensagem  enviada  pelo  CIP  WebMAIL  - Nova Geração - v. 2.1 
CentroIn Internet Provider          http://www.centroin.com.br 
Tel: (21) 2542-4849, (21) 2295-3331        Fax: (21) 2295-2978 
Empresa 100% Brasileira - Desde 1992 prestando servicos online 


-- Original Message ---
From: carlos augusto <[EMAIL PROTECTED]> 
To: obm <[EMAIL PROTECTED]> 
Sent: Tue, 18 May 2004 01:32:03 -0300 (ART) 
Subject: [obm-l] Livro de eq Diferenciais... 

> Alguem poderia me informar sobre um bom livro de eq diferenciais, para quem está começando no assunto 
>   
> Obrigado pela atenção 
>   
> Carlos Augusto
> 
> 
Yahoo! Messenger - Fale com seus amigos online. Instale agora! 
--- End of Original Message ---






Re: [obm-l] Distânciamédiaquadrática

2004-05-17 Por tôpico Augusto Cesar de Oliveira Morgado
Intdupla[(x-1)^2+y^2]dxdy/Intdupla dxdy, a regiao de integraçao sendo o disco.
A integral do denominador eh a area. A do numerador se decompoe em tres:
Int(x^2+y^2), Int(-2x), Int(1). Esta ultima eh a area; a do meio vale 0; a
primeira, calcule em coordenadas polares, dah 1/4.

==
Mensagem  enviada  pelo  CIP  WebMAIL  - Nova Geração - v. 2.1
CentroIn Internet Provider  http://www.centroin.com.br
Tel: (21) 2542-4849, (21) 2295-3331Fax: (21) 2295-2978
Empresa 100% Brasileira - Desde 1992 prestando servicos online


-- Original Message ---
From: "Henrique Patrício Sant'Anna Branco" <[EMAIL PROTECTED]>
To: <[EMAIL PROTECTED]>
Sent: Mon, 17 May 2004 22:51:02 -0300
Subject: [obm-l] Distância média quadrática

> Alguém pode me ajudar nesse? É um problema do livro Cálculo, Volume 
> 2, do George B. Thomas. Tá na parte de integrais múltiplas e o livro 
> não dá nenhuma "dica" sobre isso, apenas propõe o problema.
> 
> Encontre o valor médio do quadrado da disância do ponto P(x,y) no 
> disco x^2 + y^2 <=1 ao ponto de fronteira A(1,0)
> 
> Grato,
> Henrique
> 
> =
> Instruções para entrar na lista, sair da lista e usar a lista em
> http://www.mat.puc-rio.br/~nicolau/olimp/obm-l.html
> =
--- End of Original Message ---

=
Instruções para entrar na lista, sair da lista e usar a lista em
http://www.mat.puc-rio.br/~nicolau/olimp/obm-l.html
=


Re: [obm-l] + funcoes

2004-05-17 Por tôpico Augusto Cesar de Oliveira Morgado
A receita vale R = px = -0,2(x^2)+ 100x.
a) Se p = 60, x=200 e R= 12 000 
b) R sera maximo se x = -100/2(-0,2) = 250 e p=50.

==
Mensagem  enviada  pelo  CIP  WebMAIL  - Nova Geração - v. 2.1
CentroIn Internet Provider  http://www.centroin.com.br
Tel: (21) 2542-4849, (21) 2295-3331Fax: (21) 2295-2978
Empresa 100% Brasileira - Desde 1992 prestando servicos online


-- Original Message ---
From: "aryqueirozq" <[EMAIL PROTECTED]>
To: "[EMAIL PROTECTED]" <[EMAIL PROTECTED]>
Sent: Mon, 17 May 2004 22:28:43 -0300
Subject: [obm-l] + funcoes

> Me desculpem pelas perguntas, mas por que ainda estou 
> na 8ª série.
> 
>  O preço de ingresso numa peça de teatro (p) relaciona-
> se com a quantidade de frequentadores (x) por sessão 
> através da relação;
> 
> p = - 0,2x + 100
> 
> a) Qual a receita arrecadada por sessão, se o preço de 
> ingresso for R$60,00?
> b) Qual o preço que deve ser cobrado para dar a máxima 
> receita por sessão?
> 
>  agradeço
> 
> __
> Acabe com aquelas janelinhas que pulam na sua tela.
> AntiPop-up UOL - É grátis!
> http://antipopup.uol.com.br/
> 
> =
> Instruções para entrar na lista, sair da lista e usar a lista em
> http://www.mat.puc-rio.br/~nicolau/olimp/obm-l.html
> =
--- End of Original Message ---

=
Instruções para entrar na lista, sair da lista e usar a lista em
http://www.mat.puc-rio.br/~nicolau/olimp/obm-l.html
=


Re: [obm-l] Probabilidade

2004-05-17 Por tôpico Augusto Cesar de Oliveira Morgado
Vamos analisar o caso 2 flores em estoque:
Se houver 0 comprador, o lucro eh -40; se houver 1 comprador, o lucro eh 
25-20=5; se houver 2 ou mais compradores, o lucro eh 50.
O lucro esperado eh 0,1(-40)+0,4(5)+0,5(50)= 23.
Analise os outros casos.
==
Mensagem  enviada  pelo  CIP  WebMAIL  - Nova Geração - v. 2.1
CentroIn Internet Provider  http://www.centroin.com.br
Tel: (21) 2542-4849, (21) 2295-3331Fax: (21) 2295-2978
Empresa 100% Brasileira - Desde 1992 prestando servicos online


-- Original Message ---
From: Marcus Alexandre Nunes <[EMAIL PROTECTED]>
To: [EMAIL PROTECTED]
Sent: Mon, 17 May 2004 15:54:47 -0300 (ART)
Subject: [obm-l] Probabilidade

> Oi lista! Nao consigo resolver o seguinte exercicio de
> Probabilidade:
> 
> Um florista faz estoque de uma flor de curta duracao
> que lhe custa 20 reais e que ele vende por 45 reais no
> primeiro dia que esta flor estah na loja. Toda flor
> que nao eh vendida neste dia nao serve mais e eh
> jogada fora. Seja X a variavel aleatoria que denota o
> numero de flores que os fregueses compram em um dia
> casualmente escolhido. O florista sabe que a funcao de
> probabilidade da procura eh dada por
> 
> x| 0   |  1  |  2  |  3
> -
> f(x) | 0,1 | 0,4 | 0,3 | 0,2
> 
> Quantas flores o florista deve ter em estoque a fim de
> maximizar o seu lucro medio?
> 
> -
> Marcus Alexandre Nunes
> [EMAIL PROTECTED]
> UIN 114153703
> 
> __
> 
> Yahoo! Messenger - Fale com seus amigos online. Instale agora! 
> http://br.download.yahoo.com/messenger/
> =
> Instruções para entrar na lista, sair da lista e usar a lista em
> http://www.mat.puc-rio.br/~nicolau/olimp/obm-l.html
> =
--- End of Original Message ---

=
Instruções para entrar na lista, sair da lista e usar a lista em
http://www.mat.puc-rio.br/~nicolau/olimp/obm-l.html
=


Re: [obm-l] porcentagem

2004-05-17 Por tôpico Augusto Cesar de Oliveira Morgado
salario: 100105
aluguel: 1518,9
18,9/105 = 0,18 = 18% que eh a resposta
==
Mensagem  enviada  pelo  CIP  WebMAIL  - Nova Geração - v. 2.1
CentroIn Internet Provider  http://www.centroin.com.br
Tel: (21) 2542-4849, (21) 2295-3331Fax: (21) 2295-2978
Empresa 100% Brasileira - Desde 1992 prestando servicos online


-- Original Message ---
From: "aryqueirozq" <[EMAIL PROTECTED]>
To: "[EMAIL PROTECTED]" <[EMAIL PROTECTED]>
Sent: Mon, 17 May 2004 19:48:24 -0300
Subject: [obm-l] porcentagem

> Alguém poderia me ajudar nesta questão, estou fazendo 
> por modo muito complicado.
> 
>Uma pessoa gasta 15% do seu salário com aluguel. se 
> o aluguel aumenta de 26% e o salário de 5% , que 
> percentagem do salário essa pessoa passará a gastar com 
> aluguel?
> 
> Agradeço desde de já.
> 
> __
> Acabe com aquelas janelinhas que pulam na sua tela.
> AntiPop-up UOL - É grátis!
> http://antipopup.uol.com.br/
> 
> =
> Instruções para entrar na lista, sair da lista e usar a lista em
> http://www.mat.puc-rio.br/~nicolau/olimp/obm-l.html
> =
--- End of Original Message ---

=
Instruções para entrar na lista, sair da lista e usar a lista em
http://www.mat.puc-rio.br/~nicolau/olimp/obm-l.html
=


Re: [obm-l] Demonstracoes no ensino medio

2004-05-12 Por tôpico Augusto Cesar de Oliveira Morgado
Perdão, dão resultados em concursos mal feitos.

==
Mensagem  enviada  pelo  CIP  WebMAIL  - Nova Geração - v. 2.1
CentroIn Internet Provider  http://www.centroin.com.br
Tel: (21) 2542-4849, (21) 2295-3331Fax: (21) 2295-2978
Empresa 100% Brasileira - Desde 1992 prestando servicos online


-- Original Message ---
From: "Fellipe Rossi" <[EMAIL PROTECTED]>
To: <[EMAIL PROTECTED]>
Sent: Wed, 12 May 2004 10:26:25 -0300
Subject: Re: [obm-l] Demonstracoes no ensino medio

> A verdade é que o que vende livros e conquista novos alunos não é o
> crescimento acadêmico, são os resultados.
> As pessoas escolhem fazer esse ou aquele curso ou colégio porque 
> "ele aprova mais do q os outros no vestibular". E os cursos bitolam 
> os alunos com fórmulas e teoremas que estes nem sabem de onde 
> surgiram, mas que dão resultados num concurso.
> 
> - Original Message -
> From: "Augusto Cesar de Oliveira Morgado" <[EMAIL PROTECTED]>
> To: <[EMAIL PROTECTED]>
> Sent: Wednesday, May 12, 2004 7:55 AM
> Subject: Re: [obm-l] Demonstracoes no ensino medio
> 
> > E eu fico mais abismado ainda, porque, como dizem alguns "colegas" e
> alguns
> > autores de livros didáticos, a ausencia das demonstrações é para tornar o
> > ensino objetivo e prático.
> >
> >
> >
> > -- Original Message ---
> > From: "Marcelo Salhab Brogliato" <[EMAIL PROTECTED]>
> > To: <[EMAIL PROTECTED]>
> > Sent: Wed, 12 May 2004 03:18:08 -0300
> > Subject: Re: [obm-l] Demonstracoes no ensino medio
> >
> > > Claudio,
> > > nao participo muito da lista pois pouco sei sobre os assuntos que voces
> > > falam. Mas fiz ETEP, um dos colegios tecnicos mais conceituados da minha
> > > regiao (SJCampos-SP) e nunca tive demonstracao nenhuma. Alias, era
> decorar
> > > formula e aplicar em provas. Agora que estou estudando para ITA/IME e
> > > fazendo cursinho no Poliedro, estou tendo todas as demonstracoes e
> ficando
> > > abismado como as coisas sao simples.
> > > Infelizmente vejo que nenhum de meus amigos/conhecidos jamais viu uma
> > > demonstração.
> > >
> > > Marcelo
> >
> > =
> > Instruções para entrar na lista, sair da lista e usar a lista em
> > http://www.mat.puc-rio.br/~nicolau/olimp/obm-l.html
> > =
> >
> 
> =
> Instruções para entrar na lista, sair da lista e usar a lista em
> http://www.mat.puc-rio.br/~nicolau/olimp/obm-l.html
> =
--- End of Original Message ---

=
Instruções para entrar na lista, sair da lista e usar a lista em
http://www.mat.puc-rio.br/~nicolau/olimp/obm-l.html
=


Re: [obm-l] Demonstracoes no ensino medio

2004-05-12 Por tôpico Augusto Cesar de Oliveira Morgado
E eu fico mais abismado ainda, porque, como dizem alguns "colegas" e alguns 
autores de livros didáticos, a ausencia das demonstrações é para tornar o 
ensino objetivo e prático.



-- Original Message ---
From: "Marcelo Salhab Brogliato" <[EMAIL PROTECTED]>
To: <[EMAIL PROTECTED]>
Sent: Wed, 12 May 2004 03:18:08 -0300
Subject: Re: [obm-l] Demonstracoes no ensino medio

> Claudio,
> nao participo muito da lista pois pouco sei sobre os assuntos que voces
> falam. Mas fiz ETEP, um dos colegios tecnicos mais conceituados da minha
> regiao (SJCampos-SP) e nunca tive demonstracao nenhuma. Alias, era decorar
> formula e aplicar em provas. Agora que estou estudando para ITA/IME e
> fazendo cursinho no Poliedro, estou tendo todas as demonstracoes e ficando
> abismado como as coisas sao simples.
> Infelizmente vejo que nenhum de meus amigos/conhecidos jamais viu uma
> demonstração.
> 
> Marcelo

=
Instruções para entrar na lista, sair da lista e usar a lista em
http://www.mat.puc-rio.br/~nicolau/olimp/obm-l.html
=


Re: [obm-l] Numeros de Bernoulli. melhorando

2004-05-10 Por tôpico Augusto Cesar de Oliveira Morgado
Mostre que  {x/[expx-1]}-B1 *x eh funçao par


==
Mensagem  enviada  pelo  CIP  WebMAIL  - Nova Geração - v. 2.1
CentroIn Internet Provider  http://www.centroin.com.br
Tel: (21) 2542-4849, (21) 2295-3331Fax: (21) 2295-2978
Empresa 100% Brasileira - Desde 1992 prestando servicos online


-- Original Message ---
From: "Augusto Cesar de Oliveira Morgado" <[EMAIL PROTECTED]>
To: [EMAIL PROTECTED]
Sent: Mon, 10 May 2004 15:18:25 -0200
Subject: Re: [obm-l] Numeros de Bernoulli

> Mostre que  x/[expx-1]-B1 *x eh funçao par
> 
> ==
> Mensagem  enviada  pelo  CIP  WebMAIL  - Nova Geração - v. 2.1
> CentroIn Internet Provider  http://www.centroin.com.br
> Tel: (21) 2542-4849, (21) 2295-3331Fax: (21) 2295-2978
> Empresa 100% Brasileira - Desde 1992 prestando servicos online
> 
> -- Original Message ---
> From: Tertuliano Carneiro <[EMAIL PROTECTED]>
> To: [EMAIL PROTECTED]
> Sent: Mon, 10 May 2004 12:23:48 -0300 (ART)
> Subject: [obm-l] Numeros de Bernoulli
> 
> > Olá a todos!
> > 
> > Os números de Bernoulli sao definidos a partir do
> > desenvolvimento em serie de Taylor da funcao
> > x/(exp(x)-1) do seguinte modo:
> > x/(exp(x)-1)=S[B_n*x^n/n!], onde S indica somatorio e
> > os B_n sao os numeros procurados. Fazendo as contas
> > nao é dificil chegar à seguinte formula de
> > recorrência:
> > (n/0)B_0 + (n/1)B_1 + ... + (n/(n-1))B_(n-1) = 0.
> > Tambem vi que todos os B_n sao racionais. No entanto,
> > nao consegui mostrar que B_(2n+1)=0, para todo
> > n=1,2,... Ou seja, com excecao de B_1, todos os termos
> > de ordem impar sao nulos. Alguem tem alguma ideia?
> > Tentei por inducao, mas a expressao ficou monstruosa.
> > 
> > Grato.
> > 
> > __
> > 
> > Yahoo! Messenger - Fale com seus amigos online. Instale agora! 
> > http://br.download.yahoo.com/messenger/
> > =
> > Instruções para entrar na lista, sair da lista e usar a lista em
> > http://www.mat.puc-rio.br/~nicolau/olimp/obm-l.html
> > =
> --- End of Original Message ---
> 
> =
> Instruções para entrar na lista, sair da lista e usar a lista em
> http://www.mat.puc-rio.br/~nicolau/olimp/obm-l.html
> =
--- End of Original Message ---

=
Instruções para entrar na lista, sair da lista e usar a lista em
http://www.mat.puc-rio.br/~nicolau/olimp/obm-l.html
=


Re: [obm-l] Numeros de Bernoulli

2004-05-10 Por tôpico Augusto Cesar de Oliveira Morgado
Mostre que  x/[expx-1]-B1 *x eh funçao par

==
Mensagem  enviada  pelo  CIP  WebMAIL  - Nova Geração - v. 2.1
CentroIn Internet Provider  http://www.centroin.com.br
Tel: (21) 2542-4849, (21) 2295-3331Fax: (21) 2295-2978
Empresa 100% Brasileira - Desde 1992 prestando servicos online


-- Original Message ---
From: Tertuliano Carneiro <[EMAIL PROTECTED]>
To: [EMAIL PROTECTED]
Sent: Mon, 10 May 2004 12:23:48 -0300 (ART)
Subject: [obm-l] Numeros de Bernoulli

> Olá a todos!
> 
> Os números de Bernoulli sao definidos a partir do
> desenvolvimento em serie de Taylor da funcao
> x/(exp(x)-1) do seguinte modo:
> x/(exp(x)-1)=S[B_n*x^n/n!], onde S indica somatorio e
> os B_n sao os numeros procurados. Fazendo as contas
> nao é dificil chegar à seguinte formula de
> recorrência:
> (n/0)B_0 + (n/1)B_1 + ... + (n/(n-1))B_(n-1) = 0.
> Tambem vi que todos os B_n sao racionais. No entanto,
> nao consegui mostrar que B_(2n+1)=0, para todo
> n=1,2,... Ou seja, com excecao de B_1, todos os termos
> de ordem impar sao nulos. Alguem tem alguma ideia?
> Tentei por inducao, mas a expressao ficou monstruosa.
> 
> Grato.
> 
> __
> 
> Yahoo! Messenger - Fale com seus amigos online. Instale agora! 
> http://br.download.yahoo.com/messenger/
> =
> Instruções para entrar na lista, sair da lista e usar a lista em
> http://www.mat.puc-rio.br/~nicolau/olimp/obm-l.html
> =
--- End of Original Message ---

=
Instruções para entrar na lista, sair da lista e usar a lista em
http://www.mat.puc-rio.br/~nicolau/olimp/obm-l.html
=


Re: [obm-l] Demonstracoes no ensino medio

2004-05-09 Por tôpico Augusto Cesar de Oliveira Morgado



E a USP Sao Carlos é conceituadissima!

== 
Mensagem  enviada  pelo  CIP  WebMAIL  - Nova Geração - v. 2.1 
CentroIn Internet Provider          http://www.centroin.com.br 
Tel: (21) 2542-4849, (21) 2295-3331        Fax: (21) 2295-2978 
Empresa 100% Brasileira - Desde 1992 prestando servicos online 


-- Original Message ---
From: Johann Peter Gustav Lejeune Dirichlet <[EMAIL PROTECTED]> 
To: [EMAIL PROTECTED] 
Sent: Sun, 9 May 2004 23:01:26 -0300 (ART) 
Subject: Re: [obm-l] Demonstracoes no ensino medio 

> So uma coisa que talvez seja util voces saberem: na faculdade a turma de matematica aqui da USP-Sao Carlos tem aula de MEB (Matematica do Ensino Basico). Curioso, eu perguntei o que sec aprende nessa matera e a resposta foi um belo de um " Tudo o que se ve no Ensino Medio, com demonstraçoes! ". 
> Isto nao e estupido? 
>   
> 
> Augusto Cesar de Oliveira Morgado <[EMAIL PROTECTED]> wrote: Ah, ah, ah!
> Com esses livros didaticos e esses salarios de professores, a Matematica no 
> ensino medio nao tem mais teoremas, tem, no maximo, observações.
> O Provão mostrou que cerca de 20% dos licenciandos em Matemática (aí 
> incluidos os formandos de universidades conceituadissimas) não sabem deduzir 
> a fórmula que resolve equação do segundo grau.
> Morgado
> 
> ==
> Mensagem enviada pelo CIP WebMAIL - Nova Geração - v. 2.1
> CentroIn Internet Provider http://www.centroin.com.br
> Tel: (21) 2542-4849, (21) 2295-3331 Fax: (21) 2295-2978
> Empresa 100% Brasileira - Desde 1992 prestando servicos online
> 
> -- Original Message ---
> From: Claudio Buffara <[EMAIL PROTECTED] />
> To: Lista OBM <[EMAIL PROTECTED] />
> Sent: Sun, 09 May 2004 21:43:51 -0300
> Subject: ! [obm-l] Demonstracoes no ensino medio
> 
> > Esta eh uma pergunta para quem estah cursando ou lecionando 
> > matematica no ensino medio "normal" (ou seja, excluindo cursos de 
> > preparacao para olimpiadas ou vestibulares muito puxados como IME e 
> > ITA):
> > 
> > Em provas e exercicios propostos exige-se que os alunos apresentem
> > demonstracoes de resultados matematicos ou apenas pede-se que eles calculem
> > coisas?
> > 
> > []s,
> > Claudio.
> > 
> > =
> > Instruções para entrar na lista, sair da lista e usar a lista em
> > http://www.mat.puc-rio.br/~nicolau/olimp/obm-l.html
> > =
> --- End of Original Message ---
> 
> =
> Instruções para entrar na lista, sair da lista e usar a lista em
> http://www.mat.puc-rio.br/~nicolau/olimp/obm-l.html
> =
> 
> TRANSIRE SVVM PECTVS MVNDOQVE POTIRI 
> 
> CONGREGATI EX TOTO ORBE MATHEMATICI OB SCRIPTA INSIGNIA TRIBVERE 
> 
> Fields Medal(John Charles Fields) 
>   
> N.F.C. (Ne Fronti Crede)
> 
> 
Yahoo! Messenger - Fale com seus amigos online. Instale agora! 
--- End of Original Message ---






Re: [obm-l] Re: [obm-l] Re: [obm-l] Questão de 2o. grau

2004-05-09 Por tôpico Augusto Cesar de Oliveira Morgado



A sugestão do Marcio Cohen parece-me a melhor maneira de resolver a questão para alunos do ensino medio.
Para n>4, n! é multiplo de 10, ou seja, termina em 0. Logo, nao ha alteraçao do digito final da soma para n>4.
Para n=1, a soma vale 1; para n=2, vale 3; para n=3, vale9; para n=4, vale 33; a partir daí, todos os valores terminam em 3 e nao ha mais quadrados perfeitos.

== 
Mensagem  enviada  pelo  CIP  WebMAIL  - Nova Geração - v. 2.1 
CentroIn Internet Provider          http://www.centroin.com.br 
Tel: (21) 2542-4849, (21) 2295-3331        Fax: (21) 2295-2978 
Empresa 100% Brasileira - Desde 1992 prestando servicos online 


-- Original Message ---
From: "Fellipe Rossi" <[EMAIL PROTECTED]> 
To: <[EMAIL PROTECTED]> 
Sent: Sun, 9 May 2004 21:11:54 -0300 
Subject: [obm-l] Re: [obm-l] Re: [obm-l] Questão de 2o. grau 

> Sim, mas essas demonstrações exigem uma experiência raramente encontrada em alunos de 2o. grau. 
>   
> Outra questão do tipo seria mostrar que pra n>1, n pertence a N, que (n^2)! > (n!)^2 
>   
> No caso eu faria: 
>   
> (n^2)! = 1*2*3*...*n^2 = 1*2*3...*n*(n+1)*(n+2)*...*n^2 
> (n!)^2= 1*1*2*2*3*3*4*4*...*n*n = 1*2*3...*n*1*2*3...*n 
>   
> como (n+1)(n+2)...n^2 > 1*2*3...*n    pois n+1>1, n+2>2,..., n^2>n provamos 
>   
> Porém é mais um tipo de resolução que raramente entra na cabeça de vestibulandos. 
>   
> Oq é mais normal fazer seria: 
> (2^2)! = 24 
> (2!)^2 = 4 
>   
> (3^2)! = 9! 
> (3!)^2 = 36 
>   
> para valores maiores de n maior será a diferença... 
>   
> O que eu queria saber é se uma Banca de vestibular aceita esses tipos de resolução. 
>   
> Grato pela atenção! 
> Rossi 
> - Original Message - 
> From: Marcio Afonso A. Cohen 
> To: [EMAIL PROTECTED] 
> Sent: Sunday, May 09, 2004 6:14 PM 
> Subject: [obm-l] Re: [obm-l] Questão de 2o. grau 
> 
>     Em parte. Tudo que voce diz eh verdade, mas eu exigiria uma explicacao um pouquinho melhor de pq n! eh maior que n^2. Mas a ideia eh otima e funciona. Eu acho q faria algo como: p/ n>3, 1! + 2! + ... + n! >= n! + (n-1)!+1 > n(n-1) + (n-1) + 1 = n^2. 
>     Uma outra opcao eh olhar mod 10. 
> - Original Message - 
> From: Fellipe Rossi 
> To: [EMAIL PROTECTED] 
> Sent: Sunday, May 09, 2004 5:42 PM 
> Subject: [obm-l] Questão de 2o. grau 
> 
> Como vocês demonstrariam, para 2o. grau, que 
>   
> para n>=1, n pertence a Z. apenas n=1 e n=3 são raízes da equação: 
>   
> 1!+2!+3!+...+n! = n^2 
>   
> Vocês aceitariam uma resolução que mostrasse, com exemplos (4!=24, 4^2=16 ; 5!=120, 5^2=25, e assim por diante...) que para n>=4. n! é maior que n^2 e que como o lado esquerdo da igualdade eh n!+valor positivo, ela vai ser sempre maior que o lado direito para n>=4, e substituindo n por 1, 2 e 3 chegamos q apenas 1 e 3 são raizes? 
>   
> Essa qustão caiu, se não me engano, na prova específica da UFRJ 1992. 
>   
> Abraços! 
> Rossi


--- End of Original Message ---






Re: [obm-l] 8a cone sul

2004-05-09 Por tôpico Augusto Cesar de Oliveira Morgado
10a+b-a^2-b^2 = 25,25 -(a-5)^2 - (b-0,5)^2
a = 5 e b=0 ou b=1
==
Mensagem  enviada  pelo  CIP  WebMAIL  - Nova Geração - v. 2.1
CentroIn Internet Provider  http://www.centroin.com.br
Tel: (21) 2542-4849, (21) 2295-3331Fax: (21) 2295-2978
Empresa 100% Brasileira - Desde 1992 prestando servicos online


-- Original Message ---
From: [EMAIL PROTECTED]
To: [EMAIL PROTECTED]
Sent: Sun, 9 May 2004 21:43:41 -0300
Subject: [obm-l] 8a cone sul

> >>Por favor alguém tem idea de como posso resolver esse problema. De cada 
> número inteiro positivo n,n<=99,subtraímos a soma dos quadrados de 
> seus algarismos.Para que valores de n esta diferença é a maior 
> possível?
> 
> _
> Voce quer um iGMail protegido contra vírus e spams? 
> Clique aqui: http://www.igmailseguro.ig.com.br
> Ofertas imperdíveis! Link: http://www.americanas.com.br/ig/
> 
> =
> Instruções para entrar na lista, sair da lista e usar a lista em
> http://www.mat.puc-rio.br/~nicolau/olimp/obm-l.html
> =
--- End of Original Message ---

=
Instruções para entrar na lista, sair da lista e usar a lista em
http://www.mat.puc-rio.br/~nicolau/olimp/obm-l.html
=


Re: [obm-l] Demonstracoes no ensino medio

2004-05-09 Por tôpico Augusto Cesar de Oliveira Morgado
Ah, ah, ah!
Com esses livros didaticos e esses salarios de professores, a Matematica no 
ensino medio nao tem mais teoremas, tem, no maximo, observações.
O Provão mostrou que cerca de 20% dos licenciandos em Matemática (aí 
incluidos os formandos de universidades conceituadissimas) não sabem deduzir 
a fórmula que resolve equação do segundo grau.
Morgado

==
Mensagem  enviada  pelo  CIP  WebMAIL  - Nova Geração - v. 2.1
CentroIn Internet Provider  http://www.centroin.com.br
Tel: (21) 2542-4849, (21) 2295-3331Fax: (21) 2295-2978
Empresa 100% Brasileira - Desde 1992 prestando servicos online


-- Original Message ---
From: Claudio Buffara <[EMAIL PROTECTED]>
To: Lista OBM <[EMAIL PROTECTED]>
Sent: Sun, 09 May 2004 21:43:51 -0300
Subject: [obm-l] Demonstracoes no ensino medio

> Esta eh uma pergunta para quem estah cursando ou lecionando 
> matematica no ensino medio "normal" (ou seja, excluindo cursos de 
> preparacao para olimpiadas ou vestibulares muito puxados como IME e 
> ITA):
> 
> Em provas e exercicios propostos exige-se que os alunos apresentem
> demonstracoes de resultados matematicos ou apenas pede-se que eles calculem
> coisas?
> 
> []s,
> Claudio.
> 
> =
> Instruções para entrar na lista, sair da lista e usar a lista em
> http://www.mat.puc-rio.br/~nicolau/olimp/obm-l.html
> =
--- End of Original Message ---

=
Instruções para entrar na lista, sair da lista e usar a lista em
http://www.mat.puc-rio.br/~nicolau/olimp/obm-l.html
=


Re: [obm-l] Re: [obm-l] Re: [obm-l] Questão de 2o. grau

2004-05-09 Por tôpico Augusto Cesar de Oliveira Morgado



A melhor maneira de transformar uma pessoa (honesta, por suposto) em defensora incondicional das provas de multipla escolha é faze-la acompanhar de perto a correçao de provas discursivas de um vestibular. O criterio de correçao é nenhum!
Morgado

== 
Mensagem  enviada  pelo  CIP  WebMAIL  - Nova Geração - v. 2.1 
CentroIn Internet Provider          http://www.centroin.com.br 
Tel: (21) 2542-4849, (21) 2295-3331        Fax: (21) 2295-2978 
Empresa 100% Brasileira - Desde 1992 prestando servicos online 


-- Original Message ---
From: "Fellipe Rossi" <[EMAIL PROTECTED]> 
 O que eu queria saber é se uma Banca de vestibular aceita esses tipos de resolução. 





Re: [obm-l] Calcular a soma

2004-05-09 Por tôpico Augusto Cesar de Oliveira Morgado
1/senx = -cotx + cot(x/2)

==
Mensagem  enviada  pelo  CIP  WebMAIL  - Nova Geração - v. 2.1
CentroIn Internet Provider  http://www.centroin.com.br
Tel: (21) 2542-4849, (21) 2295-3331Fax: (21) 2295-2978
Empresa 100% Brasileira - Desde 1992 prestando servicos online


-- Original Message ---
From: [EMAIL PROTECTED]
To: [EMAIL PROTECTED]
Sent: Sun, 9 May 2004 06:15:16 -0300
Subject: [obm-l] Calcular a soma

> Por favor alguém tem alguma idéia para calcular esta soma: 
> S=1/senX + 1/sen2X + 1/sen4X + 1/sen8X + ... + 1/sen(2^n)X
> 
> _
> Voce quer um iGMail protegido contra vírus e spams?
> Clique aqui: http://www.igmailseguro.ig.com.br
> Ofertas imperdíveis! Link: http://www.americanas.com.br/ig/
> 
> =
> Instruções para entrar na lista, sair da lista e usar a lista em
> http://www.mat.puc-rio.br/~nicolau/olimp/obm-l.html
> =
--- End of Original Message ---

=
Instruções para entrar na lista, sair da lista e usar a lista em
http://www.mat.puc-rio.br/~nicolau/olimp/obm-l.html
=


Re: [obm-l] O que eh "AO ACASO"?correçao

2004-05-08 Por tôpico Augusto Cesar de Oliveira Morgado
Corrigindo um erro de digitação do Claudio: (correçao em maiusculas)



> 
> -- Original Message ---
> From: Claudio Buffara <[EMAIL PROTECTED]>
> > E que tal isso aqui?
> > Escolhe-se ao acaso o ponto medio da corda (o qual especifica univocamente
> > essa corda).
> > Se m(corda) > raiz(3), entao o ponto medio estarah a uma distancia 
inferior
> > a 1/2 de centro (trigonometria basica). Logo, os pontos medios permitidos
> > ocupam o interior do circulo de raio 1/2 concentrico com o circulo 
> > original, cuja area eh 1/4 DA AREA DO CIRCULO ORIGINAL. Isso dah uma 
probabilidade de 1/4.
>
=
Instruções para entrar na lista, sair da lista e usar a lista em
http://www.mat.puc-rio.br/~nicolau/olimp/obm-l.html
=


Re: [obm-l] O que eh "AO ACASO"?

2004-05-08 Por tôpico Augusto Cesar de Oliveira Morgado
Só complementando: esse problema é conhecido como Paradoxo de Bertrand. as 
tres soluções apresentadas são as tres apresentadas por Bertrand.
Ha um outro Paradoxo de Bertrand, menos conhecido:
Selecionam-se, ao acaso, dois pontos A e B em uma superficie esferica de raio 
1. Qual é a probabilidade de o menor arco de circulo máximo que os contêm ter 
comptimento menor que a? (0http://www.centroin.com.br
Tel: (21) 2542-4849, (21) 2295-3331Fax: (21) 2295-2978
Empresa 100% Brasileira - Desde 1992 prestando servicos online


-- Original Message ---
From: Claudio Buffara <[EMAIL PROTECTED]>
To: <[EMAIL PROTECTED]>
Sent: Sat, 08 May 2004 09:07:58 -0300
Subject: Re: [obm-l] O que eh "AO ACASO"?

> E que tal isso aqui?
> Escolhe-se ao acaso o ponto medio da corda (o qual especifica univocamente
> essa corda).
> Se m(corda) > raiz(3), entao o ponto medio estarah a uma distancia inferior
> a 1/2 de centro (trigonometria basica). Logo, os pontos medios permitidos
> ocupam o interior do circulo de raio 1/2 concentrico com o circulo 
> original, cuja area eh 1/4. Isso dah uma probabilidade de 1/4.
> 
> on 08.05.04 01:11, niski at [EMAIL PROTECTED] wrote:
> 
> > Tudo depende em definir, com precisão, o que voce quer dizer com
> > "escolher uma corda ao acaso". Acho que o mais natural é pensar que
> > a posicao da corda pode ser escolhida de acordo com a distancia ao
> > centro do circulo. Nesse caso para o comprimento da corda ser maior que
> > sqrt(3) (comprimento do lado de um triangulo equilatero inscrito na
> > circunferencia) é preciso que a distancia entre a corda e o centro seja
> > menor do que 1/2 e a probabilidade pedida sai imediatamente. Mas tambem
> > é valido pensar no seguinte: Desenhe uma corda, por uma de sua
> > extremidade traçe uma tangente a circunferencia. O angulo formado entre
> > a corda e a tangente varia de 0 a 180, mas dai no caso voce esta
> > interessado quando angulo esta entre 60 e 120, o que fornece outro valor
> > para a probabilidade diferente da primeira formulação do problema.
> > A resposta mais apropriada deve ser dada levando em conta as condicoes
> > do experimento em que o problema é proposto. No caso especifico do
> > enunciado que voce propos, ambas as respostas 1/2 e 1/3 sao validas.
> > 
> > 
> >> Esse dah margem pra uma boa discussao:
> >> 
> >> Escolhe-se, AO ACASO, uma corda de uma circunferencia de raio = 1.
> >> Qual a probabilidade dessa corda ter comprimento > raiz(3)?
> >
> 
> =
> Instruções para entrar na lista, sair da lista e usar a lista em
> http://www.mat.puc-rio.br/~nicolau/olimp/obm-l.html
> =
--- End of Original Message ---

=
Instruções para entrar na lista, sair da lista e usar a lista em
http://www.mat.puc-rio.br/~nicolau/olimp/obm-l.html
=


Re: [obm-l] Uma certa confusao

2004-05-07 Por tôpico Augusto Cesar de Oliveira Morgado
f(x,y) = (x^2)*(y)/ (x^4+y^2); f(0,0) = 0
(0,0)
continua ao longo de todas as retas por (0,0)
descontinua ao longo de y = x^2
==
Mensagem  enviada  pelo  CIP  WebMAIL  - Nova Geração - v. 2.1
CentroIn Internet Provider  http://www.centroin.com.br
Tel: (21) 2542-4849, (21) 2295-3331Fax: (21) 2295-2978
Empresa 100% Brasileira - Desde 1992 prestando servicos online


-- Original Message ---
From: Claudio Buffara <[EMAIL PROTECTED]>
To: <[EMAIL PROTECTED]>
Sent: Fri, 07 May 2004 15:42:15 -0300
Subject: Re: [obm-l] Uma certa confusao

> on 07.05.04 12:14, Will at [EMAIL PROTECTED] wrote:
> 
> > 
> > Arthur, se tiver como buscar para nós um destes exemplos ou então, se você
> > lembrar, apontar uma bibliografia, seria ótimo :-)
> > 
> > Me refiro a  funções tal que " restricoes de f a todas as retas que passem
> > por um ponto x sejam continuas mas que f, entretanto, naum seja continua 
em
> > x."
> > 
> > Não estou participando ativamente desta discussão por excesso de 
ignorância
> > no assunto, mas estou tentando aprender Analise no R^n esse periodo e mais
> > conteúdo é sempre bom !
> > 
> > Abraço
> > Will
> > 
> > 
> > - Original Message -
> > From: "Artur Costa Steiner" <[EMAIL PROTECTED]>
> > 
> > Um fato que eu naum
> > acho muito intuitivo: Eh possivel que as restricoes de
> > f a todas as retas que passem por um ponto x sejam
> > continuas mas que f, entretanto, naum seja continua em
> > x. Os exemplos que eu jah vi sao um tanto
> > "patologicos", mas matematicamente perfeitos.
> > Artur
> > 
> >
> 
> =
> Instruções para entrar na lista, sair da lista e usar a lista em
> http://www.mat.puc-rio.br/~nicolau/olimp/obm-l.html
> =
--- End of Original Message ---

=
Instruções para entrar na lista, sair da lista e usar a lista em
http://www.mat.puc-rio.br/~nicolau/olimp/obm-l.html
=


Re: [obm-l] Re:[obm-l] Essa éfácil ???? sim ou não ?? ??

2004-05-07 Por tôpico Augusto Cesar de Oliveira Morgado
Desisto!

==
Mensagem  enviada  pelo  CIP  WebMAIL  - Nova Geração - v. 2.1
CentroIn Internet Provider  http://www.centroin.com.br
Tel: (21) 2542-4849, (21) 2295-3331Fax: (21) 2295-2978
Empresa 100% Brasileira - Desde 1992 prestando servicos online


-- Original Message ---
From: "rickufrj" <[EMAIL PROTECTED]>
To: "obm-l" <[EMAIL PROTECTED]>
Sent: Fri,  7 May 2004 13:29:19 -0300
Subject: [obm-l]  Re:[obm-l] Essa é fácil  sim ou não ??  ??

> Ãm?!Inteiros nao sao reais?
> 
> __
> Acabe com aquelas janelinhas que pulam na sua tela.
> AntiPop-up UOL - É grátis!
> http://antipopup.uol.com.br/
> 
> =
> Instruções para entrar na lista, sair da lista e usar a lista em
> http://www.mat.puc-rio.br/~nicolau/olimp/obm-l.html
> =
--- End of Original Message ---

=
Instruções para entrar na lista, sair da lista e usar a lista em
http://www.mat.puc-rio.br/~nicolau/olimp/obm-l.html
=


Re: [obm-l] DE NOVO COLEGIO NAVAL

2004-05-07 Por tôpico Augusto Cesar de Oliveira Morgado

min (A, B) > C se e somente se A > C  e  B > C. Logo, o que se pede é a 
soluçao do sistema  2x-7>-3x+3  e  8-3x>-3x+3

-- Original Message ---
From: "leandro-epcar" <[EMAIL PROTECTED]>

>   La vai mais uma para gente esquentar!!
> 
>  COLEGIO NAVAL  (2002)
> 
>Se A e B sao dois numeros reais,denotarmos por
>  min(A,B) o menor dos numeros A e B ,isto e ,
> |==|
> min(A,B)|(A, se A = |(A, se A >=B) |
> |==|
> o numeros de solucoes inteiras negativas da inequacao
>  min(2X-7, 8-3X)> -3X+3
>  (A) 0
>  (B) 1
>  (C) 2
>  (D) 3
>  (E) 4
> 
>  O ESTRANHO E ESSE MINIMO
> 
> |==|
> min(A,B)|(A, se A = |(A, se A >=B) |
> |==|
> 
> 
>   VALEU!
> 
> -- Início da mensagem original ---
> 
>   De: [EMAIL PROTECTED]
> Para: [EMAIL PROTECTED]
>   Cc: 
> Data: Fri, 7 May 2004 00:36:20 -0300
>  Assunto: RE: [obm-l] En:COLEGIO NAVAL
> 
> > Olá Leandro,
> > 
> > As questões das provas do Colégio Naval 
> geralmente são muito
> > interessantes, principalmente as questões de 
> Geometria Plana. Se for
> > possível, eu tenho interesse em obter as provas do 
> Colégio Naval de todos os
> > anos que você tiver disponível. Você tem versões 
> digitais destas provas?
> > 
> > Vamos a uma solução para esta bela questão de 
> geometria.
> > 
> > RESOLUÇÃO POSSÍVEL:
> > 
> > A diagonal BD divide o quadrilátero ABCD em dois 
> triângulos, sendo que as
> > medidas de todos os lados destes triângulos foram 
> fornecidas.
> > Triângulo ABD: AB = 42 = 7.6, BD = 56 = 7.8 e DA = 49 
> = 7.7
> > Triângulo BCD: BC = 48 = 8.6, CD = 64 = 8.8 e DB = 56 
> = 8.7
> > 
> > Observe que AB/BC = BD/CD = DA/DB = 7/8, portanto:
> > Triângulo ABD ~ Triângulo BCD, pelo critério LLL.
> > Logo:  triângulos semelhantes)
> > Sendo assim, o segmento DP pertence à reta bissetriz 
> do ângulo ADC.
> > 
> > Aplicando o Teorema da bissetriz interna no segmento 
> DP do triângulo ADC,
> > teremos:
> > PA/PB = DA/DC => PA/PB = 49/64
> > 
> > Resposta: Letra E
> > 
> > Atenciosamente,
> > 
> > Rogério Moraes de Carvalho
> > Consultor e Instrutor de Tecnologias da Informação
> > [EMAIL PROTECTED]
> > 
> > -Original Message-
> > From: [EMAIL PROTECTED] [mailto:owner-obm-
> [EMAIL PROTECTED] On
> > Behalf Of leandro-epcar
> > Sent: quinta-feira, 6 de maio de 2004 17:35
> > To: obm-l
> > Subject: [obm-l] En:COLEGIO NAVAL
> > 
> > -- Início da mensagem original ---
> > 
> >   De: "leandro-epcar" leandro-
> > [EMAIL PROTECTED]
> > Para: "obm-l" [EMAIL PROTECTED]
> >   Cc: 
> > Data: Wed, 28 Apr 2004 10:31:09 -0300
> >  Assunto: COLEGIO NAVAL
> > 
> >   de:leandro geraldo da costa.
> > para:obm-l.mat.puc-rio.com.br
> >  assunto:geometria plana.
> > data: 28 de abril de 2004.
> >   -- 
> > 
> >  PROVA DO COLEGIO NAVAL DE 2003.
> >
> >Num quadrilátero ABCD tem-se:AB=42, BC=48, CD=64,
> >  DA=49 e P é o ponto de interseção entre as diagonais
> >  AC e BD .Qual a razão entre os segmentos PA e PC ,sa-
> >  bendo-se que a diagonal BD é igual a 56.
> >   
> >   (A)7/8.
> >   (B)8/7.
> >   (C)7/6.
> >   (D)6/7.
> >   (E)49/64.
> >   --
> >  OBSERVAÇÃO:
> >
> >Desenhando com escala notei que a diagonal BD é 
> >  bissetriz de ADC, mas não tenho como provar.  
> >   
> >   --
> > 
> >Tenho as provas de admissao do colegio naval de
> >  52 a 66 e 87 a 2003.
> >se houver algum interesado ,entrem em contato .   
> >  
> > 
> 
> > __
> > Acabe com aquelas janelinhas que pulam na sua tela.
> > AntiPop-up UOL - É grátis!
> > http://antipopup.uol.com.br/
> > 
> > 
> >  
> > 
> 
> __
> > Acabe com aquelas janelinhas que pulam na sua tela.
> > AntiPop-up UOL - É grátis!
> > http://antipopup.uol.com.br/
> > 
> > 
> > 
> > 
> 
> =
> > Instruções para entrar na lista, sair da lista e usar 
> a lista em
> > http://www.mat.puc-rio.br/~nicolau/olimp/obm-l.html
> > 
> 
> =
> > 
> > 
> > 
> > 
> > 
> 
> =
> > Instruções para entrar na lista, sair da lista e usar 
> a lista em
> > http://www.mat.puc-rio.br/~nicolau/olimp/obm-l.html
> > 
> 
> =
> >
> 
> __
> Acabe com aquelas janelinhas que pulam na 

Re: [obm-l] Re:[obm-l] Essa éfácil ???? sim ou não ?? ??

2004-05-07 Por tôpico Augusto Cesar de Oliveira Morgado
A e b sao reais, nao inteiros.

==
Mensagem  enviada  pelo  CIP  WebMAIL  - Nova Geração - v. 2.1
CentroIn Internet Provider  http://www.centroin.com.br
Tel: (21) 2542-4849, (21) 2295-3331Fax: (21) 2295-2978
Empresa 100% Brasileira - Desde 1992 prestando servicos online


-- Original Message ---
From: "rickufrj" <[EMAIL PROTECTED]>
To: "obm-l" <[EMAIL PROTECTED]>
Sent: Fri,  7 May 2004 13:04:32 -0300
Subject: [obm-l]  Re:[obm-l] Essa é fácil  sim ou não ?? ??
=
Instruções para entrar na lista, sair da lista e usar a lista em
http://www.mat.puc-rio.br/~nicolau/olimp/obm-l.html
=


Re: [obm-l] corrida...

2004-05-04 Por tôpico Augusto Cesar de Oliveira Morgado
Nao mudo nada do que escrevi na epoca. De FDB so se chega a FDB ou a DBF 
mediante um numero par de transposiçoes. A resposta é E.
Quanto a nova soluçao do Anglo, me lembra homeopatia, que vence a doença por 
cansaço. Nao vou ler um calhamaço desses porque sei que de FDB so se chega a 
FDB ou a DBF mediante um numero par de transposiçoes.

==
Mensagem  enviada  pelo  CIP  WebMAIL  - Nova Geração - v. 2.1
CentroIn Internet Provider  http://www.centroin.com.br
Tel: (21) 2542-4849, (21) 2295-3331Fax: (21) 2295-2978
Empresa 100% Brasileira - Desde 1992 prestando servicos online


-- Original Message ---
From: niski <[EMAIL PROTECTED]>
To: [EMAIL PROTECTED]
Sent: Tue, 04 May 2004 23:52:53 -0300
Subject: Re: [obm-l] corrida...

> Bem lembrado. Inclusive fui eu mesmo quem começou a discussao! Nao 
> tinha nem lembrado. Bom, na epoca a resolucao do Anglo foi 
> considerada errada, porem estive vendo e eles consertaram a 
> argumentacao, mas continuam com a alternativa A. Será que voces 
> conseguiriam (Morgado?) refutar a nova tentativa de solucao do 
> anglo? Nao consigo ver nenhum furo nela. Segue 
http://www.linux.ime.usp.br/~niski/anglo.gif
> 
> Ariel de Silvio wrote:
> > vejam a discussao que citei:
> >  
> > http://www.mail-archive.com/cgi-bin/htsearch?
method=and&format=short&config=obm-l_mat_puc-
rio_br&restrict=&exclude=&words=rubinho 
> > <http://www.mail-archive.com/cgi-bin/htsearch?
method=and&format=short&config=obm-l_mat_puc-
rio_br&restrict=&exclude=&words=rubinho>
> 
> -- 
> Niski - http://www.linux.ime.usp.br/~niski
> 
> [upon losing the use of his right eye]
> "Now I will have less distraction"
> Leonhard Euler
> 
> =
> Instruções para entrar na lista, sair da lista e usar a lista em
> http://www.mat.puc-rio.br/~nicolau/olimp/obm-l.html
> =
--- End of Original Message ---

=
Instruções para entrar na lista, sair da lista e usar a lista em
http://www.mat.puc-rio.br/~nicolau/olimp/obm-l.html
=


[obm-l] Re: Sobre a rpm e Integral. Festival de besteira.

2004-05-03 Por tôpico Augusto Cesar de Oliveira Morgado
O teorema de Cramer diz: Um sistema de equaçoes lineares em que a matriz dos 
coeficientes das incognitas eh quadrada e tem determinante diferente de zero 
possui uma e uma so soluçao.
A regra de Cramer diz: Em um sistema nas condiçoes do teorema de Cramer, o 
valor de cada incognita pode (eh, eh; esse pode eh otimo; talvez seja melhor 
acrescentar para os leitores que nao leem tudo com a devida atençao: pode, 
mas nao deve) ser calculado dividindo-se os valores de dois determinantes ...

teorema de Pitágoras: em um triangulo retangulo, o quadrado do maior lado ...

Se voce aplicar o teorema de Pitagoras a um triangulo que nao seja retangulo, 
nao vai dar certo. Se voce aplicar a regra de Cramer a um sistema que nao 
satisfaça "sistema de equaçoes lineares em que a matriz dos coeficientes das 
incognitas eh quadrada e tem determinante diferente de zero" tambem nao vai 
dar certo.
Alguns ilustres matematicos brasileiros (qua, qua, qua), descritos no Exame 
de Textos, fazem coisas absurdas do tipo:
sistema: x+y+z=1
x+y+z = 2
x+y+z = 3 
det coef = det (1,1,1);(1,1,1);(1,1,1) = 0
Em vez de pararem por aih, porque o sistema nao satisfaz as condiçoes para 
ser resolvido pela regra de Cramer, os imbecis continuam:
det numerador da incognita x = det (1,2,3);(1,1,1);(1,1,1) = 0
Logo, x = 0/0 e o sistema é indeterminado!
Um desses bobalhoes que escrevem livros a respeito de assuntos que nao 
compreenderam e que deveriam ter sido reprovados na oitava serie, em ediçao 
anterior do seu livro, chegou a resumir a discussao de um sistema com a 
inesquecivel frase:
nesse sistema, x eh indeterminado, y eh indeterminado e z eh impossivel.
Aconselho, mais uma vez, que leiam o Exame de Textos: eh divertido e servira 
para aguçar o espirito critico.
Queria ser descendente de Cramer para processar esses imbecis por injuria, 
calunia e difamaçao.






=
Instruções para entrar na lista, sair da lista e usar a lista em
http://www.mat.puc-rio.br/~nicolau/olimp/obm-l.html
=


Re: [obm-l] Sobre a rpm e Integral

2004-05-03 Por tôpico Augusto Cesar de Oliveira Morgado




A regra de Cramer é tão falha quanto o teorema de Pitágoras. Se voce usar o teorema em um triangulo nao-retangulo, nao da certo. Do mesmo modo, se voce usar a regra num sistema cuja matriz de coeficientes das incognitas nao seja quadrada ou tenha determinante igual a 0, tambem nao vai dar certo.

== 

Mensagem  enviada  pelo  CIP  WebMAIL  - Nova Geração - v. 2.1 

CentroIn Internet Provider          http://www.centroin.com.br 

Tel: (21) 2542-4849, (21) 2295-3331        Fax: (21) 2295-2978 

Empresa 100% Brasileira - Desde 1992 prestando servicos online 



-- Original Message ---

From: Alan Pellejero <[EMAIL PROTECTED]> 

To: [EMAIL PROTECTED] 

Sent: Mon, 3 May 2004 12:13:56 -0300 (ART) 

Subject: Re: [obm-l] Sobre a rpm e Integral 


> Então por qual motivo o sistema proposto pela prova é falho?

> Por Cramer tem-se que o sistema é indeterminado, embora, por escalonamento, saiba-se que este é impossível.

>  

> 
> Augusto Cesar de Oliveira Morgado <[EMAIL PROTECTED]> wrote:

O Prof. Elon jamais disse que a técnica é falha, até porque ela não é. O que ele disse é que ela é uma técnica muito ruim para resolver sistemas "grandes", pois exige um número muito grande de operações. 
> 
> == 
> Mensagem  enviada  pelo  CIP  WebMAIL  - Nova Geração - v. 2.1 
> CentroIn Internet Provider          http://www.centroin.com.br 
> Tel: (21) 2542-4849, (21) 2295-3331        Fax: (21) 2295-2978 
> Empresa 100% Brasileira - Desde 1992 prestando servicos online 
> 
> -- Original Message --- 
> From: Alan Pellejero <[EMAIL PROTECTED]> 
> To: [EMAIL PROTECTED] 
> Sent: Mon, 3 May 2004 09:29:49 -0300 (ART) 
> Subject: [obm-l] Sobre a rpm e Integral 
> 
> >
 Pessoal, obrigado pela ajuda! 
> >   
> > A integral é: 
> >   
> > / 
> > | x / (senx) dx 
> > / 
> >   
> > E sobre uma prova que estava na rpm, o Prof. Elon comenta que a técnica de Cramer pra determinação de um sistema é falha. Eu fiz a demonstração que estava no livro, olhei atentamente, analisei, mas, mesmo assim, não achei o motivo pelo qual essa técnica é falha. Seria um erro conceitual? 
> >   
> > Muito obrigado... 
> >   
> > Ps: sobre as "caudas", eu juro que eu as apago antesnão sei o que acontece...mas de qualquer maneira peço desculpas. 
> >   
> > abraços, 
> > Alan Pellejero 
> > 
> > 


Yahoo! Messenger - Fale com seus amigos online. Instale agora! 
> --- End of Original Message --- 
> 
> 
> 
Yahoo! Messenger - Fale com seus amigos online. Instale agora!

--- End of Original Message ---








Re: [obm-l] Sobre a rpm e Integral

2004-05-03 Por tôpico Augusto Cesar de Oliveira Morgado



O Prof. Elon jamais disse que a técnica é falha, até porque ela não é. O que ele disse é que ela é uma técnica muito ruim para resolver sistemas "grandes", pois exige um número muito grande de operações.

== 
Mensagem  enviada  pelo  CIP  WebMAIL  - Nova Geração - v. 2.1 
CentroIn Internet Provider          http://www.centroin.com.br 
Tel: (21) 2542-4849, (21) 2295-3331        Fax: (21) 2295-2978 
Empresa 100% Brasileira - Desde 1992 prestando servicos online 


-- Original Message ---
From: Alan Pellejero <[EMAIL PROTECTED]> 
To: [EMAIL PROTECTED] 
Sent: Mon, 3 May 2004 09:29:49 -0300 (ART) 
Subject: [obm-l] Sobre a rpm e Integral 

> Pessoal, obrigado pela ajuda! 
>   
> A integral é: 
>   
> / 
> | x / (senx) dx 
> / 
>   
> E sobre uma prova que estava na rpm, o Prof. Elon comenta que a técnica de Cramer pra determinação de um sistema é falha. Eu fiz a demonstração que estava no livro, olhei atentamente, analisei, mas, mesmo assim, não achei o motivo pelo qual essa técnica é falha. Seria um erro conceitual? 
>   
> Muito obrigado... 
>   
> Ps: sobre as "caudas", eu juro que eu as apago antesnão sei o que acontece...mas de qualquer maneira peço desculpas. 
>   
> abraços, 
> Alan Pellejero
> 
> 
Yahoo! Messenger - Fale com seus amigos online. Instale agora! 
--- End of Original Message ---






Re: [obm-l] Artigo sobre a OBM no Estadao

2004-05-03 Por tôpico Augusto Cesar de Oliveira Morgado
25

==
Mensagem  enviada  pelo  CIP  WebMAIL  - Nova Geração - v. 2.1
CentroIn Internet Provider  http://www.centroin.com.br
Tel: (21) 2542-4849, (21) 2295-3331Fax: (21) 2295-2978
Empresa 100% Brasileira - Desde 1992 prestando servicos online


-- Original Message ---
From: Claudio Buffara <[EMAIL PROTECTED]>
To: Lista OBM <[EMAIL PROTECTED]>
Sent: Mon, 03 May 2004 08:27:04 -0300
Subject: [obm-l] Artigo sobre a OBM no Estadao

> Oi, pessoal:
> 
> Saiu hoje, no jornal O Estado de Sao Paulo, um artigo falando sobre 
> a OBM e alguns de seus vencedores.
> 
> A parte mais interessante (e triste) do artigo eh uma avaliacao 
> feita pelo Nicolau sobre a matematica que se ensina nas escolas: 
> "...as provas e os exercicios sao chatos e mecanicos, nao fascinam 
> ninguem, e mesmo aqueles que poderiam gostar de matematica acabam se 
> desinteressando."
> 
> Infelizmente, eu sou forcado a concordar com essa avaliacao. E o 
> pior eh que existem varios resultados matematicos extremamente 
> interessantes, bonitos e faceis de se entender (desde que expostos 
> com um minimo de competencia, o que eh cada vez mais raro nesse pais 
> - vide o resultado do provao para licenciatura em matematica) mas 
> que nao sao nem mencionados nos cursos de ensino medio do pais,
>  exceto naqueles cujo objetivo eh a preparacao para olimpiadas de 
matematica.
> 
> Por exemplo, de uma olhada no resultado da enquete sobre beleza matematica
> que foi feita aqui na lista em agosto do ano passado em
> http://www.mail-archive.com/[EMAIL PROTECTED]/msg14668.html
> e responda: Quantos destes resultados voce viu no ensino medio?
> 
> E pra nao deixar de propor um problema, aqui vai: um dos resultados
> mencionados na enquete contem um erro. Qual deles e qual o erro?
> 
> []s,
> Claudio.
> 
> =
> Instruções para entrar na lista, sair da lista e usar a lista em
> http://www.mat.puc-rio.br/~nicolau/olimp/obm-l.html
> =
--- End of Original Message ---

=
Instruções para entrar na lista, sair da lista e usar a lista em
http://www.mat.puc-rio.br/~nicolau/olimp/obm-l.html
=


Re: [obm-l] AJUDA

2004-05-02 Por tôpico Augusto Cesar de Oliveira Morgado
O livro citado é bom.
Meu favorito (não como livro de leitura) como professor, para um primeiro 
curso de Inferencia, é o de Harold Larson.

==
Mensagem  enviada  pelo  CIP  WebMAIL  - Nova Geração - v. 2.1
CentroIn Internet Provider  http://www.centroin.com.br
Tel: (21) 2542-4849, (21) 2295-3331Fax: (21) 2295-2978
Empresa 100% Brasileira - Desde 1992 prestando servicos online


-- Original Message ---
From: "Henrique Patrício Sant'Anna Branco" <[EMAIL PROTECTED]>
To: <[EMAIL PROTECTED]>
Sent: Sat, 1 May 2004 19:50:03 -0300
Subject: Re: [obm-l] AJUDA

> Morgado,
> 
> Devo começar a disciplina Inferência Estatística no próximo semestre 
> e já ouvi falar da falta de boas obras traduzidas sobre o assunto. 
> Não me incomodo de ler livros em inglês, o problema é consegui-los 
> aqui. O livro citado é bom? Existem outros bons?
> 
> Grato,
> Henrique.
> 
> - Original Message - 
> From: "Augusto Cesar de Oliveira Morgado" <[EMAIL PROTECTED]>
> To: <[EMAIL PROTECTED]>
> Sent: Saturday, May 01, 2004 8:11 PM
> Subject: [obm-l] AJUDA
> 
> >
> > Peço adiantadamente desculpas aos colegas pelo off topic, mas tenho um
> > problema. Qual o ano em que foi editado o livro Introdução à Inferência
> > Estatística, SBM, Heleno Bolfarine e Monica Sandoval?
> > Obrigado.
> > Morgado
> >
> >
> > =
> > Instruções para entrar na lista, sair da lista e usar a lista em
> > http://www.mat.puc-rio.br/~nicolau/olimp/obm-l.html
> > =
> >
> 
> =
> Instruções para entrar na lista, sair da lista e usar a lista em
> http://www.mat.puc-rio.br/~nicolau/olimp/obm-l.html
> =
--- End of Original Message ---

=
Instruções para entrar na lista, sair da lista e usar a lista em
http://www.mat.puc-rio.br/~nicolau/olimp/obm-l.html
=


Re: [obm-l] Integral

2004-05-02 Por tôpico Augusto Cesar de Oliveira Morgado



Faça u = x, dv = senx dx e integração por partes.

Obs: Os membros da lista que usam conexão discada amam suas mensagens com essas caudas que não dizem respeito a ela.

== 
Mensagem  enviada  pelo  CIP  WebMAIL  - Nova Geração - v. 2.1 
CentroIn Internet Provider          http://www.centroin.com.br 
Tel: (21) 2542-4849, (21) 2295-3331        Fax: (21) 2295-2978 
Empresa 100% Brasileira - Desde 1992 prestando servicos online 


-- Original Message ---
From: Alan Pellejero <[EMAIL PROTECTED]> 
To: [EMAIL PROTECTED] 
Sent: Sun, 2 May 2004 17:04:43 -0300 (ART) 
Subject: [obm-l] Integral 

> Pessoal, até agora não sei direito o que é off-topic, mas já que não consigo resolver essa aqui, gostaria de saber se alguém tem alguma sugestão para: 
>   
> / 
> | x(senx)dx 
> / 
>   
> Obrigado 
> Alan Pellejero
> 
> [EMAIL PROTECTED] wrote: Seja P(x_0, y_0) um ponto qualquer e r:ax+by+c=0 a equacao da circunferencia, temos que a distancia entre elas eh dada por:
> d_p;r = |a*x_0 + b*y_0 + c| / (sqrt(a^2 + b^2))
> 
> Como sabemos que a origem eh o ponto P(0,0) e r eh 4x + 3y –5 = 0
> 
> d_p;r = |a*x_0 + b*y_0 + c| / (sqrt(a^2 + b^2))
> d_p;r = |4*0 + 3*0 - 5| / (sqrt(4^2 + 3^2))
> d_p;r = 5 / 5 = 1
> 
> Em uma mensagem de 2/5/2004 13:30:14 Hora padrão leste da Am. Sul, [EMAIL PROTECTED] escreveu:
> 
> 
> 
> Pessoal, como calcular origem de reta em geo. analitica ? 
> 
> Ex: Calcule a distância da origem à reta  r: 4x + 3y –5 = 0 
> 
> 
> 
> 
> 
> 
Yahoo! Messenger - Fale com seus amigos online. Instale agora! 
--- End of Original Message ---






Re: [obm-l] Re: [obm-l] Estimação

2004-05-01 Por tôpico Augusto Cesar de Oliveira Morgado
E[a(1)X_1+...+a(n)X_n]=a(1)E[X_1]+...+a(n)E[X_n]= [a(1)+...+a(n)]E[X]



==
Mensagem  enviada  pelo  CIP  WebMAIL  - Nova Geração - v. 2.1
CentroIn Internet Provider  http://www.centroin.com.br
Tel: (21) 2542-4849, (21) 2295-3331Fax: (21) 2295-2978
Empresa 100% Brasileira - Desde 1992 prestando servicos online


-- Original Message ---
From: "Henrique Patrício Sant'Anna Branco" <[EMAIL PROTECTED]>
To: <[EMAIL PROTECTED]>
Sent: Sat, 1 May 2004 19:50:57 -0300
Subject: [obm-l] Re: [obm-l] Estimação

> Morgado,
> 
> Não entendi direito. Eu precisaria provar que a condição necessária e
> suficiente para o estimador ser não-viciado é que a soma dos coeficientes
> seja 1, certo? Como eu procederia com isso?
> 
> Grato,
> Henrique.
> 
> - Original Message - 
> From: "Augusto Cesar de Oliveira Morgado" <[EMAIL PROTECTED]>
> To: <[EMAIL PROTECTED]>
> Sent: Friday, April 30, 2004 8:01 PM
> Subject: Re: [obm-l] Estimação
> 
> > Se os coeficientes são a(1),...,a(n), o estimador será não-viciado sse
> > a(1)+...+a(n)=1 e terá variância mínima sse a(1)^2+...+a(n)^2 for mínimo.
> > Multiplicadores de Lagrange ou desigualdades espertas mostrarão que
> > a(1)=...=a(n)=1/n.
> 
> =
> Instruções para entrar na lista, sair da lista e usar a lista em
> http://www.mat.puc-rio.br/~nicolau/olimp/obm-l.html
> =
--- End of Original Message ---

=
Instruções para entrar na lista, sair da lista e usar a lista em
http://www.mat.puc-rio.br/~nicolau/olimp/obm-l.html
=


  1   2   3   4   5   6   7   8   9   10   >